You are on page 1of 93

1

PEDIATRICS
SICK BABY QUESTIONS:
- Ask what is wrong? Can you tell more about it?
- Fever? Did you give any medications for the fever? Did it help?
- Does he have cough? Noisy breathing? Difficulty breathing? Runny nose?
- ENT: Have you noticed the baby pulling at the ear?
- Is she able to feed? Is she able to keep the feeds down? Any vomiting?
- UTI/dehydration: How wet are the nappies? Any change on the color or smell of nappies?
- Joint: does she cry when you touch her anywhere?
- Any rash or abnormal posturing?
- Contacts: Do you have any other children? Are they sick? Does your child go to childcare?
- BINDS. How has your baby been since birth? Immunizations up to date?
- Physical examination: general appearance, vitals, length, height, weight and anthropometric measurements
- Do not forget ENT, abdomen and genitals!!
- Urine examination

PEDIATRIC EMERGENCIES

Bee Sting

Case: You are an HMO and a 9-year-old girl was brought by a school nurse. She was stung by a bee and developed swollen lips along
with SOB. On examination, she has tachycardia and her BP is 60/40. You are about to see the nurse on duty in the hospital and you will
meet the mother later on.

Task
a. Give the nurse instructions regarding management
b. Explain child’s condition to the mother and answer her questions

RCH Guidelines
- Vasopressor and bronchodilator therapy: adrenaline 10mcg/kg;
- Adrenaline 0.01 ml/kg 1:1000 IM or 0.01 mg/kg 1:10,000
- Oxygen by mask
- IV volume expander: 0.9% NSS 20ml/kg then give repeat boluses of 10-20 ml/kg until BP restored
- Bronchodilator therapy with salbutamol continuous nebulized (0.5%) or IV 5mcg/kg per min for 1 hour, then 1 mcg/kg per minute
thereafter
- Relief of upper airway obstruction: mild to moderate edema may respond to inhalation of nebulized adrenaline 1% adrenaline
(1ml per dose diluted to 4 ml) or 5 ml of nebulised 1:1000 solution
- Anaphylaxis biphasic and may deteriorate again over the next few hours; all patients with anaphylaxis should be observed for at
least 12 hours
- Allergen testing, self-injectable adrenaline, medi-alert bracelet

Immediate Resuscitation
- Can you please check the patient’s level of consciousness? What is her GCS? Can you see a bee sting in the skin? Can you
please take it out with the help of a forcep? Can you please check her airway? Is it patient? Any secretions? Can you please
auscultate the lungs for air entry? I would like you to put her on

oxygen around 6-8 liters/minute via a mask. Can you please give her adrenaline in a concentration of 1:1000 IM 0.01ml/kg or 0.1
ml/kg 1:10,000 IV. Can
you check the blood pressure? If the response is not good, would you kindly repeat it in the same concentration every 5-10
minutes. Can you please put in 2 IV cannula. Please start her on normal saline 300-500 ml depending upon the child’s stature.
Can you please tell me if the child has any wheezing or if she is still SOB? Can you please start her on nebulized salbutamol.
Can you please give her 1mg/kg of hydrocortisone IV. Would you kindly give her cimetidine or other antihistamine that might be
available.

Explanation to Mom
- Let me reassure you that your child is stable at the moment. She is better with the emergency measures we have done. It is
quite understandable that you are very anxious to know what happened to her. Basically, what she had is a condition called an
anaphylactic reaction to the bee sting. It is a kind of an allergic reaction that can happen after insect bite, certain foods, as well
as intake of certain medications. It is a serious condition as there is a risk of choking or circulatory compromise. Therefore, we
need to keep her in the hospital for the next 12-24 hours. There is a chance of rebound reaction within the 1st 6-12 hours. I will
call in the pediatric registrar to come and take a look at her for further assessment. They will assess her condition and they will
do further testing later on that might include skin prick testing or RAST. With these tests we will determine if she is allergic to
other substances as well.
- Can you please tell me if she has any known allergies already? Any history of eczema, asthma, hay fever or drug allergy? How’s
her general health? Does she have family history of similar allergies? Was she stung before?
- Before going home, I will ask the nurse to explain to you what to do in case of another bee sting, but let me explain the
components of the anaphylaxis action plan. This includes the use of a special kit. Basically, it contains an epipen (an injection of
adrenaline) that needs to be given IM in case of anaphylaxis. It also contains a ventolin puffer, a tablet of antihistamine, and
steroids. During another attack, you need to observe her for certain signs such as wheeze, hoarseness of voice, loss of
consciousness, vomiting/diarrhea, and swollen blue lips with or without swelling all over the body. In case she develops these
2

symptoms you will use this injection and give it on the thigh over the clothes. You can repeat it every 5-10 minutes. Make her lie
flat on the ground, elevate the legs, call family for help, but most importantly, call 000 for an ambulance. Please provide these
injections to the school along with written instructions. Avoid places where she is likely to have a bee sting. Avoid wearing
colorful clothes. Avoid using perfumes places especially gardens.
- Once she is discharged, please come for followup with the immunologist for venom immunotherapy.
- Reading material.

Local reactions
- Local ice application, elevation, analgesics and antihistamines
- Removal of bee sting: scrape off skin with fingernail (don’t squeeze)

- Care at home: washing skin, calamine lotion to help with itching, ice packs, elevation, antishistamine (phenergan or zyrtec),
strong steroid creams applied early and regularly onto skin that was bitten;
- See doctor if: reactions in other parts of the body such as hives, or breathing problems; child has a lot of pain where they were
bitten which does not settle down within a few hours; swelling or itching gets worse after 24 hours
- Medications should be carried with the child at all times.

Determination of ETT tube:


- Diameter ETT tube: Age of child/4 + 4
- Length of ETT tube: Age of child/2 + 12

Foreign Body Swallowing

Case: You are an HMO in ED. A 3-year-old girl brought to you by mom. She says that mom swallowed bulletin board pin about half an
hour ago. The child is fine at the moment. She has been examined by another doctor and she is asymptomatic.

Task:
a. Talk to the mother about the management.

- History: can you please describe how it happened? Did you or anyone else see her at the time? What type of pin is it? Can you
show me? Did she have anything to drink afterwards? Any vomiting? Cough? Bleeding? Any abdominal pain? Is it the first time
for this to happen?
- At the moment, your child doesn’t have any symptoms. All examination is normal so we need to find out where the pin is lodged.
So I would like to order xrays of the neck, chest and abdomen. As you can see on the x-ray that the pin has crossed the pylorus
which is one of the narrowest parts of the stomach, so there is a very high chance it will pass out spontaneously. You can take
her home but keep an eye on her. Watch out for symptoms like fever, tummy pain, vomiting or blood in stools. These symptoms
indicate that the pin might be causing damage to the bowel wall. Unless you want to, there is no need to check the stools for the
pin. We will not be doing follow up xrays unless she develops symptoms. Please understand that it is very important to supervise
your child at all times to prevent this from happening again. I will give you written material regarding safety at home for kids.
- For food bolus: allow to drink fizzy drinks then do back tap
- For lead: if passing to stomach then do not do anything about it but if it stays, then take it out and do followup xrays

FOREIGN BODY

Radio-opaque Radiolucent
(pins, batteries, (glass/plastic)
buttons, coins) or
unknown
Asymptomatic Symptomatic
(Observe at (drooling, chest
Xray (neck, chest home) pain, intolerant to
and abdomen) food)

Stomach and Esophagus


beyond (ENT surgeon)

DEVELOPMENT AND PSYCHOSOCIAL CONDITIONS

Temper Tantrums

Case 2: John brought his 4-year-old son Sammy to your GP clinic. He and his wife think that Sammy has ADHD and want you to check
him for that.

Task
a. Further history (cannot control him; annoys parents; parents are frustrated; teachers not having same problem)
b. Examination Findings (eye contact, seems like happy child; can communicate with doctor; vital signs, chest and heart, normal)
3

c. Advise parents about your diagnosis

Features:
- Dramatic reaction of kicking, shouting, screaming, breath-holding, throwing, or banging of the head
- Start 12-28 months and persist until 3-4 years.
- Occur if child is tired or bored
- Perpetuated if the tantrums are inadvertently rewarded by the parents to seek peace and avoid conflict

History
- Detailed history to gain insight into family stresses
- Allows parents to ventilate their feelings
- Ask parents exactly what child does during a tantrum – what they do during and after and what causes the tantrums
- School performance

Physical examination
- Growth charts
- Check hearing and vision
- School performance

Management
- Reassure that that the tantrums are relatively commonplace and not harmful
- Explain reasons for tantrums and include the concept that “temper tantrums need an audience”
- Ignore what is ignorable (parents should pretend to ignore the behavior and leave the child alone without comment, including
moving to a different area, but not locking the child in his room)
- Stay calm and say nothing
- Don’t give in
- Avoid what is avoidable: try to avoid other causes of tantrums (visiting supermarket)
- Distract what is distractable: redirect child’s interest to some other object or activity
- Praise appropriate behavior
- When ignored, the problem will probably get worse for a few days before it starts to improve. Medication has no place in
management of temper tantrums

Headbanging

Case: A mother of a 2-1/2 boy comes to your GP practice complaining that child is banging his head several times a day.

Task
a. Further history
b. Counsel the mother

Features:
- Common (5-15%) in infants and toddlers under 4 especially 3
- Also occurs in developmental disability and severe emotional deprivation
- Usually prior to going to sleep
- Headbanging occurs 60-80x/minute
- Lasts several minutes to 60 minutes or more per episode
- Associated repetitive movements (body rocking, thumb sucking)
- Child usually not distressed and rarely self-injurious
- Consider an autism spectrum disorder
- Management
o Reassure that it is self-limiting and usually settles by 3-5 years
o Avoid reinforcing behavior by excessive attention or punishment
o Advise distraction or actively ignoring behavior
o Place bed or cot in middle of the room away from a wall
o Restrict bed time

Differential diagnosis Headbanging


- Temper tantrum
- Pain (ENT infections, teething)
- Vision and hearing problems
- Autism spectrum disorder
- Normal type of headbanging

History
- I understand you have come to see me because you’re concerned about Martha’s behavior. When did it start? How often does
this episode occur? For how long? Does it happen anytime in particular (day or night)? Apart from headbanging, does he suck
his thumb or rocks his body or hum? Have you noticed any abnormal body movements? Does he have a bad temper? Does he
get upset when you say no?
4

- How was your pregnancy? Delivery? Any complications after delivery like infections or jaundice? How is his general health? Any
concerns about his growth and development? Is he walking upstairs without help? Can he throw a ball? Can he build a tower of 4
blocks? How is his speech? Is he trying to combine two words and making short sentences?
- Does he enjoy your company? Does he play with other children? Any concern about his hearing or vision? Immunization up to
date? Medications? Allergies?
- Who do you live with? Any family or financial problems? Do you have enough support? Any mental or behavioral problem that
run in the family?

Counseling
- According to your child’s history, he has a condition which is called head banging. Have you ever heart about it? It is a behavioral
problem and is very common in this age. In most cases, head banging bears a self-calming effect on children. It is similar to
other activities like thumb sucking or fondness of a toy or blanket.
- To you, it appears like your son is trying to injure himself when in fact he is trying to get rid of stress or tension. Children don’t get
serious injuries while head banging and grows up normally. He will grow out of this condition by the age of 4 or earlier. During
head banging, try to ignore him but make sure he gets plenty of positive attention when he is not banging his

head. Try to move the cot away from the walls or hang a small fabric or quilt in between the wall and cot. Try to set up a relaxing
routine. A warm bath, quiet story or song may help. Rubbing his back or stroking his forehead may have a calming down effect.
- I will review your son frequently. Please let me know if you need any help or support.

Breathholding Spell

Case: You are a GP and a 2-year-old boy was brought to you by his father because an hour ago, the child had a finger jammed by the
door then he stopped breathing and started twitching. Father is concerned about epilepsy.

Task
a. Relevant history
b. Physical examination (normal appearance, normal vital signs, finger has swelling and bluish discoloration, CNS normal)
c. Diagnosis and management

Features
- Usually 6months to 6 years
- 2 types: occurring with a tantrum and other simple faint in response to pain or fright
- Blue attacks: breath-holding with a closed glottis
- White attacks: reflex anoxic seizures often in response to pain
- Become pale, cyanosed jerky movements, unconsciousness or a fit; lasts for 10-60 seconds
- Management: coma position; reassure patients; maintain discipline and resist spoiling the child; avoid incidents that frustrate the
child or precipitate a tantrum by distraction methods

History
- How is he now? Is it very painful? Offer painkiller. Could you please describe what exactly happened? Is it the first time? How
long did it last? did he wet himself? Did he lose consciousness? Or was he drowsy? Did he bite his tongue?
- FHx of epilepsy
- Have you noticed that he cries excessively when he needs something? Does he accept “NO” easily? Behavioral problems such
as headbanging? How is his health in general? Any fever or head injuries? BINDS? How is his growth and development? Is he
growing well? His language? Has he started talking? Does he like playing with other children? Does he emotionally interact with
you? How is the relationship with other siblings?
- Are you a happy family? Any kind of stressors in the family?

Physical examination
- General appearance:
- Vital signs and growth charts
- Examination of injured fingers

Diagnosis and Management


- Your child had a breath-holding attack. It is a common condition in this age group. It is a behavioral disorder that is completely
harmless. It can be precipitated by pain, emotion or frustration. Usually, it disappears by 3 years. Let me reassure you that this is
not epilepsy.
- Try to avoid the situations which are precipitating it. Ignore him and keep him safe by putting him in the lateral position. Distract
him. Do not spoil the child by bribing.

- This will not lead to any epilepsy or brain damage.


- Reading material.
- Red flags: attacks are more than 1 minute, loses consciousness or gets drowsy, bite his tongue or incontinent
- Review

Stuttering
5

Case: You are a GP and a 4-year-old boy was brought because of stuttering for the last 6 months. He was alright before but the parents
think that the problem developed since he joined Kindergarten. It is more prominent when the child is excited. His growth and development
are normal’

Task
a. Relevant history
b. Management

History
- Please tell me more about how and when it started. Have you noticed what type of stuttering it is? Is he able to initiate his
speech? Is he repeating the word often? Are there breaks within the sentence? Do you notice that his stuttering increase during
a particular circumstance/situation? Do you think his behavior remains the same at home or otherwise?
- I understand he started Kindergarten 6 months ago, any problems over there? How is his relationship with his teachers and the
kids? Does he speak the same at kindergarten? Do you think he is under any kind of stress? How is the home situation? How is
the relationship between you and your husband and the kids? Any other family member who stutters? Is this your only child?
What about the other kids?
- Do you have any concerns about his growth and development? Immunization status? Past medical conditions? How is his
appetite, sleep, waterworks and bowel habits? May I ask, what is your response to the child when he stutters?

Management
- Let me reassure you that stuttering is a very common condition. Around half of all kids who attend kindergarten stutter in one
way or another. It is more commonly seen with boys between the age of 2 and 5 years. The usual causes are anxiety, tiredness,
unfamiliar or strange surroundings, reading within groups while using difficult vocabulary, when the child is forced to speak and
when competing against classmates. Please don’t worry. Stuttering is not related to his intelligence. There is no organic
pathology within his brain or anywhere else. 65% of kids grow out of this condition by themselves. What is important at this point
is to identify any stress-related causes to this condition and to rule out any chance of bullying or teasing. You will need to
cooperate with the child throughout this time.
- I will tell you what needs to be done and avoided:
o Listen to your child. Don’t interrupt him in between.
o Be patient if he gets stuck with a word. Please don’t finish his sentences for him. Repeat what he has said
o Reassure him and be supportive.
o Don’t criticize him for not being fluent.
o Don’t allow any family member to tease him.
o Don’t push him to speak
o Try to avoid circumstances where he is likely to stutter.

- We will give him a trial of around 6-12 months with these techniques. You will need to involve his teachers. If at the end of 1
year, he still doesn’t him improve, I will refer him to the speech therapist but usually 80-90% of children improve if treatment is
started before the age of 5 years.
- Reading material. Review.

RCH
- Strong genetic link (50-75%); Developmental anomaly
- Period of time that has lapsed since the onset of stuttering is a strong predictor with little chance of natural recovery in children
>9 year old
- Treatment: if >6 mos refer to speech pathologist trained in Lidcombe programme.

Speech Delay

Case: A 9-month-old baby was brought by parents to your GP clinic. They are concerned that baby could not say any words. The baby
was seen by another GP a few days earlier.

Task
a. Relevant history
b. Management
c. No physical examination required
d. Answer parents’ questions

History
- I know you have come to see me because you are concerned about your son’s speech. Is he babbling? Does he variable
syllables? Does he imitate speech sounds? Any concern about his hearing or vision? Does he enjoy musical toys? Does he turn
to loud sounds? How was your pregnancy? Is he a term baby? What type of delivery did you have? Any complication after
delivery such as fever or yellowing of the skin? Did he have hearing screening after delivery?
- How is his general health? Any serious illnesses or recurrent ear infection or head trauma in the past? Are you still breastfeeding
or bottlefeeding? Any concern about his growth? Immunization up-to-date? Is he your only child? Regarding his development,
can he sit without support? Can he stand holding on? Can he pass object from one hand to another (7months)? Can he try to
grasp small objects between his index finger and thumb (7-11 months)? Does he play peek-a-boo? Does he enjoy cuddles and
eye contact? Has he lost any developed skills recently (regression)?
- What did other doctor say regarding your concerns? How’s the situation at home? Any recent emotional or financial stressors?
Do you have any FHx of hearing problem or speech delay?

Management
6

- According to your son’s history, he has no problem regarding his development. As a parent, you are doing a wonderful job and
he reached all his milestones required at this age group. Usually, we expect baby to say 1-3 clear words between 9 and 15
months. You don’t have any concerns regarding hearing and he is babbling using variable syllables which are all good sings
regarding his language development.
- If you are still very concerned, I can arrange referral to pediatric audiologist for formal hearing assessment. If audiogram is
normal, I just need to review your son when he is 12 months old.

Global Developmental Delay

Case: A mother presents to your GP practice concerned that her 21-month-old child has not started walking yet.

Task
a. History (spoke 1st word at 14 months, sits with support, plays with other children; has another daughter)
b. Physical examination (active but with no dysmorphic features; growth chart height and weight 50 percentile, head circumference
10th to 25th percentile, hypotonia, jerk/reflex brisk)
c. Advise on further management

Features
- Delay in two or more important areas of development
- Causes: genetic or hereditary disorders such as Down syndrome, or other developmental disorders such as CP or spina bifida);
premature birth, infections or various metabolic diseases, neurologic (epilepsy)
- Investigations: metabolic tests and screening, genetic testing, hearing and vision test, lead screening, thyroid screening, EEG,
CT scan, psychologic assessment,

History
- I understand that you are concerned because your child is not yet walking, can I ask a few more questions? When did he lift his
head (2-3 months)? When did he start sitting with support (6months)? Without support (8 months)? Can he stand while holding
on to things? Can he hold things with his hand and pass it from one to the other? Pincer grasp? When did he speak his first
word? Does he turn around when you call his name or to loud sounds? Does he play peek-a-boo? Does he play with other
children? Can he indicate what he wants (15 months)? Can he drink from a cup (17 months)? Do you have other kids? How
would you compare their development?
- Does he get sick often since birth? How was the delivery? Were there any complications? Have you ever been sick while
pregnant? Did they do the heel-prick test? Is the immunization up-to-date? Is he eating well? Any problem with urination or
bowel? How are things at home?
- Case 2 (delayed walking): maybe associated with mild hypotonia; refer to physiotherapist and review child in one month;
reassure; refer if after 1-2 month

Physical Examination
- General appearance and scissoring of legs
- Vital signs
- Growth chart
- Neurologic examination: IT PRC (inspection, tone, power, reflex, coordination)

Management
- From the history and examination, your child has a condition called global developmental delay because it seems that he has
achieved his developmental milestones at a later time. I am also concerned about his head circumference which is lower
compared to his height and weight. At this stage I would like to refer him to a specialist pediatrician who will do a full
developmental assessment. If required, the specialist might do some investigations.
- Review. Referral ASAP.

Fuzzy Eater

Case: You are a GP and a 2-year-old child was brought by his mother. She is worried about her child’s weight and wants your advice.

Case 2: Your next patient in GP practice is a 2-1/2 year old boy brought in by his father John because he has poor appetite and does not
eat properly. Parents are concerned and feel that he has not been gaining weight like other children with the same age (Dr. Wenzel)

Case 3: Mr. Smith brought his son David who is 2-1/2 years old. Mr Smith is worried that David is not eating well. The parents are very
concerned about this and think that he is not gaining weight as other children of his age.

Task
a. Focused History
b. Physical examination (weight 15kg, height 95cm)
7

c. Management

Features:
- 8/10 Australian parents are concerned about their child’s eating habits
- 1/3 of parents worry that their child isn’t eating enough
- Management
o Keep calm and don’t make a fuss of whether your child is eating or not
o Be realistic about the amount of your child’s meals
o Don’t threaten, nag or yell
o Don’t use lollies, chocolates, biscuits, milk or desserts as bribes
o Meal time:
Be a good role model
Ask your child to help prepare a meal
Set up regular habits for eating such as always putting your child in their high chair or eating at the same
table
Offer a range of colorful foods
Encourage self feeding and exploration of food from early age
At the end of meal, take your child’s plate away. If they haven’t eaten much, offer them a healthy snack later
on or wait until next mealtime

History
- What is your concern? Have you brought his growth charts with you? Do you think he is not eating well? Can you describe his
typical daily diet to me? How much milk does he take? What type of milk? Do you think he is picky about his food? Can you
describe his behavior at meal time? Does he eat at the table with his family? Are you concerned about his general health? Does
he have any problems with his bowel habits (diarrhea, constipation)? Does he have N/V/tummy pain? Do you think his diapers
are smelly? Any concerns about waterworks? Does he have fever, cough, SOB? Did he suffer from frequent respiratory tract
infections previously? Do you think he is pale or turns blue at any time?
- Please tell me more about the pregnancy? Any problems or complications? Was he delivered full term? NSVD? What was the
BW? Is this a planned pregnancy? Do you think he has achieved all the developmental milestones on time? Immunization?

May I ask, how is the home situation? Any stress/financial problems? Is he your only child?
- Any family history of anybody on a special diet? Chronic diarrhea or other genetic conditions? Anyone smoking at home? Is he
able to sleep well? Does he go to childcare? Did he have the heelprick test

Physical examination
- General appearance and dysmorphic features; level of consciousness
- Bruises over skin
- Growth charts and observe pattern of growth (height, weight, growth chart)
- Vital signs
- Mouth: evidence of glossitis, cheilitis, or protruding tongue
- Skin: dry, scaly skin
- Measure bulk of triceps/biceps
- ENT, thyroid, chest, heart, abdomen
- Inspection of genital area: signs of abuse or perianal skin changes
- Urine dipstick

Management
- I do not see any real cause for concern. Please don’t worry, your child looks physically fine to me. The only problem might be
that he is a bit lower than the weight percentile. This could be completely normal for him, but what is important is to acknowledge
that his behavior regarding food needs to be reviewed.
- It looks like he is fuzzy about his eating. A fuzzy eater is one who refuses to try a new food at least half of the time when it is
offered. It is very common with this age group. Almost half of all toddlers fit this description. Please understand that this is not a
disease, but a variant of normal behavior.
- It is important to establish healthy eating habits to avoid problems like obesity and eating disorders later in life. I will refer you to
the dietitian to help you, but I do have some suggestions. Generally, show the child a healthy eating pattern by adopting it
yourself. Try to offer a variety of food at different times throughout the day. Please don’t force him and don’t threaten him, but
also don’t bribe him into eating. Don’t be discouraged if they reject a new food initially. You will need to offer it at least 3-4 times
in the beginning. Have a regular routine to eat meals together with the family at the table. Encourage the child to feed himself
and to help you prepare his food. Please understand that a child has appetite equivalent to his fist. You can offer 3 small meals
and 2 snacks in between. Please make sure that he is not tired, ill, or emotionally upset when offering food. Lastly, as long as
your child is putting on weight please do not stress yourself about his health and weight.
- Reading material. Referral to dietitian.

ALLERGY AND IMMUNOLOGY

Atopic Eczema and Egg Allergy

Case: An 8-month-old child comes with his mother to your GP clinic complaining of swelling of the lip after breakfast in the morning that
settled after some time. She also had a skin rash over the elbow for the last 2 month.

Task
a. History (+FHx of eczema and asthma; noted after eating egg; (+) dry scaly itchy rash x 2months in the elbow)
8

b. Explain Diagnosis and Management

Case 2: The next patient in GP practice is a 10-month-old Suzie brought by mother Sarah. She is otherwise a healthy child who suffers
from dry intermittent itchy skin for the last 3 months. It seems worse in warm environments. There is a history of asthma and hay fever in
the family.

Task
a. Focused history (rash on the face, cheek and arms, normal pregnancy, not related to food, gets worse with warm shower)
b. Examination (alert, happy looking child, red, erythematous, non-blanching rash; Vital and growth chart is good)
c. Diagnosis and management

Allergens in children: egg, peanuts, cow’s milk

History
- Can you tell me more about it? Was egg given for the first time? Amount? When did it happen? How long did it last? rash
(urticaria)? Flushing of the face? Swelling of the tongue? DOB and respiratory distress? Hoarseness while crying? Musical
sounds in the chest? N/V/diarrhea? Confused/dizzy/floppy/LOC?
- Rash: What kind of rash is it? Are there discharges? Weeping/Oozing? Crusting? Asthma? Hay fever? Eczema? FHx? Any
change in the soaps/cosmetics/clothings? Is it affected by the season? Does anybody smoke at home?
- BINDS?

Diagnosis and Management


- Your child has 2 conditions: Egg allergy due to exposure to the egg for the first time and eczema. Atopy refers to an allergic
condition that tends to run in the families that includes asthma, hay fever, atopic eczema and skin sensitivities. For atopic
eczema, it is not a dangerous disease. It is a superficial skin disorder and will not cause scarring. Most of the children outgrow it
by late childhood (5 years).
- Can she go for vaccination? Current vaccines do not contain egg. It’s present only in minute amounts in the MMR vaccine. We
can give it in the presence of resuscitation facilities
Egg allergy:
- Avoid eating or coming into contact with allergen-containing food
- Read all food labels
- Ask about probability of allergen occurring in food ingredients and methods of preparation
- Emergency anaphylaxis kit containing adrenaline Epipen, salbutamol, steroids, antihistamines
- Avoid eating products during pregnancy and breastfeeding
- Breastfeeding for the first 6 months believed to help prevent allergy
- Red Flags for anaphylaxis!
Atopic Eczema:
- Avoid soap and perfumed products use a bland bath oil in bath (QV, alpha-keri) and bland cleansing agent (sorbolene cream)
- Apply moisturizing agent to dry, irritated skin TID
- Short tepid showers
- Avoid rubbing and scratching
- Avoid sudden changes in temperature
- Wear light, soft loose clothes (cotton clothing)
- Avoid dusty, dirty conditions and sand especially sand pits
- Dust mite eradication steps
- Medications: antihistamine, corticosteroid creams, wet dressing for weeping lesions and topical antibiotic (mupirocin)

Allergic Rhinitis

Case: A 3-year-old child brought in by mom to your GP clinic because he has been having problems breathing from the nose. He snores at
night and has frequent URTI. The mom wants to know why this is happening again and again

Task
a. History: dad with hay fever; son has eczema
b. Examination: open mouth breathing; nasal crease; mucosa is swollen; throat is mildly congested; (+) nasal discharge
c. Diagnosis
d. Investigation
e. Management

History
- What exactly do you mean when you say he has difficulty breathing? Since when have you noticed this? Does this happen
during day or night as well? Cough? Runny nose? Sneezing? Can you hear loud breathing or wheezing sound from the child? Is
there anything that makes his breathing better or worse (e.g.seasonal change, contact with pets, dust, smoking, stuff toys)? Do
you think he had ear infections in the past? Any hearing problems?
- FHx: allergies? Hay fever? Asthma? Eczema? What about his growth and development? Is he growing well? How was your
pregnancy and delivery with this child? Immunization up to date? Past medical condition I should be aware of?
- Do you know what type of eczema (flexural – arms, legs, neck, face)? Is it there at the moment? What medications do you
usually use? Do they help? Is he allergic to any medications?

Physical examination
9

- General appearance: pallor, jaundice, dehydration, look for mouth breathing? Adenoid facies? Nasal crease? Bad odor from the
mouth?
- ENT: check for the condition of nasal mucosa bilaterally. Pale? Swollen? Boggy in appearance? Check patency of air passages
within the nose. Any polyps or foreign body, ulcers? Look for nasal crease. Inspect the throat for signs of inflammation over the
tonsils? Visualize the adenoids (at posterior nares)? Palpate LN especially of the neck.
- Chest, and heart
- Skin for presence of eczema
- Palpate abdomen for tenderness, or organ enlargement

Diagnosis and Management


- Most likely what your son has is a very common condition called allergic rhinitis. It is a condition where the inner lining of the
nose becomes inflamed and engorged as a result of sensitivity to an allergen. These allergens are usually present in the
environment in the form of: a. dust, b. pollen, c. cigarette smoke, d. pets, and e. fluffy toys.
- Usual symptoms are watery nasal discharge, stuffy nose, sneezing, and itchy nose. It is commonly associated with a FHx of
allergic phenomena. I need to refer you to an allergy specialist to confirm the allergen. There are two types of tests available:
RAST and skin prick test. The RAST is a bit expensive and the results are available within a few weeks, but this is very sensitive.
The second one is a skin prick test. It is cheap and quick but not very sensitive. The choice will depend on the specialist.

- Please avoid all possible allergen at the moment.


o If possible, avoid pets in the house
o Vacuum all carpets at least once a week. If possible, remove the carpets.
o Avoid smoking around the child
o Don’t put fluffy toys around the child
o Don’t let the child go outside while grass is being cut
- I can prescribe antihistamine given in the form of nasal drops. The second option is a nasal decongestant. I will prescribe topical
steroid (beclamethasone) also in the form of a spray. We usually recommend to give it for 7-10 days. Otherwise, prolonging the
treatment may cause rebound rhinorrhea that is difficult to treat. If it is not controlled with topical steroids and a specific allergen
is known, we might recommend oral antihistamine, especially for seasonal allergies.
- Usual choice is oral non-sedating: astemizole, cetirizine, fexofenadine, loratadine, terfenadine
- Reading material and follow up after she has seen the specialist.

David is 7 months old. He presented to you in your GP practice with coughing and wheezy chest.

Task:
a. History
b. Examination findings
c. Differential diagnosis
d. Diagnosis and management

Differential Diagnosis:
- Asthma
- Bronchiolitis
- Pneumonia
- Foreign body aspiration
- URTI
- Cystic fibrosis

History:
- How many days? More during the day? Or night time? Continuous? Like a dog? Wheezing since when? Is it the first time?
When? Runny nose? Fever? Vomiting? Diarrhea? Rash? How bad? What are you using? Do you have –cough cough cough
and oooo-
- History of atopy: dad with childhood asthma, mum with asthma and hay fever, brother with asthma and eczema
- Smoking in the family; feeding history
- History of having had previous episodes? Details? How long did it stay? What’s the diagnosis of them?

Physical examination
- GROWTH chart!
- Signs of respiratory distress (tracheal tug, tachypnea, supracostal, intercostals, subcostal retractions, alar flaring)

Diagnosis and management:


- This may be asthma because of the strong family history of atopy and because of the recurrent wheezy chest.
- I will need to call an ambulance and arrange for hospital admission because his wheezing is severe and he has signs of
respiratory distress. I will start him with oxygen via nasal prong at 2L/m (up to 4L/m). If it does not work or will need more than
4L/m then use mask. I will also give a trial of ventolin (efficacy of spacer is the same as nebulizer) of up to 6 puffs (3puffs
q20mins for 3 doses) or nebulizer with salbutamol 2.5mg q20 x 3 doses).

- Prednisolone (Predmix 5mg/ml): 1mg/kg


10

- If mild: we can put him with a trial of salbutamol. Side effects may include fast heart rate and he may be hyperactive but
otherwise, it is relatively safe. Salbutamol will be given via a pressured metered-dose inhaler with spacer for 5 days 3 puffs every
4-6 hours and review for 2 days.

Acute Severe Exacerbation of Asthma

Case: You are an HMO at a small rural country hospital ED. Nina aged 8 years, arrives at the hospital in a car driven by Margaret, her
mother. She says that Nina, who has a past history of asthma had a runny nose, cold and cough for 3 days. Her asthma has been worse
over the last 2 days, and she has been requiring frequent ventolin (12 puffs via spacer hourly for the last2 hours). Despite ventolin, Nina
has deteriorated. Margaret noticed on the way to the hospital that Nina stopped talking and her breathing became more labored. Margaret
is worried about Nina and seeks your help.

Nina was born at 40 weeks gestation and delivery was uneventful. She was first diagnosed with asthma at the age of 3 years. She usually
takes flixotide twice a day and for her exacerbation of asthma she needs ventolin and oral prednisolone. On average, she has two to three
exacerbations per year and has had five ward admissions since she was 3 years of age. Nina’s immunizations are up to date. She is
allergic to penicillin.

Task
a. Focused history
b. Examination and investigation findings from examiner
c. Diagnosis and management advise

History
- Triggers? Medications? Smoker? Pets? Carpets? FHx of asthma? Other medical problems? Other sick kids at home?

Physical examination
- General appearance: Level of consciousness
- Vital signs: oxygen saturation
- ENT/Lungs: tracheal tug, retractions, alar flaring, etc.

Assessment of Severity

Mild Moderate Severe Critical


Mental Normal Normal Agitated Confused/
State Drowsy
Work of Normal Mildly Markedly/ Maximally
Breathing increased moderately increased
increased or
exhausted
- Include oxygen saturation, ability to talk and heart rate

Management
- Severe: Oxygen <92%; give salbutamol 6-12 puffs pMDI/spacer q20 x 3 doses; review 10 minutes after 3rd dose; ipratropium
bromide 2 or 4 puffs pMDI/spacer, q20 x 3 doses; oral prednisolone 1mg/kg x dose (3 days) or IV if with vomiting; admit to
hospital with frequent review; no aminophylline or IV salbutamol
- Critical Asthma: oxygen; ICU admission; nebulized continuous salbutamol q20 x 3 doses; nebulized ipratropium bromide
(25mcg) q20 x 3 doses; IV methylprednosolone/hydrocortisone 1mg/kg/dose q6 or 8 on day 1; aminophylline and IV beta-agonist
and MgSO4 if with poor initial response

Pattern of Asthma

Feature Preventer Symptom Reliever


Controller
Infrequent Ep. 6-8 Nil Nil SABA prn
Episodic weeks
apart; sxs
rare
between
attacks;
normal
examination
and lung fxn
bet eps
Frequent Attacks <6 ICS (100- Nil SABA prn
Episodic wks apart; 200 mcq
more fluticasone)
troublesome or LRT ant.
attacks; inc.
sx bet
11

attacks;
normal
examination
and lung
function bet
eps
Persistent Daytime sx LRT ant. or If on SABA prn
>2d/wk; ICS (100- 250mcg/day
nocturnal sx 200 mc add LABA
>1n/wk; fluticasone) (fluticasone
attacks <6 up to
wks apart; 500mcg/day);
may have consider
abnormal referral to
lung pulmo
function;
may have
multiple ED
visits

Heart failure secondary to congenital heart disease (Coarctation of the Aorta) not all causes of wheezy chest are pulmonary in
origin. Be aware of CHD!!!

Case: You are an HMO working in the ED. Your next patient is David, 7 weeks old who has been having cough and wheezy chest.
a. History
b. Physical examination
c. Differential diagnosis
d. Management

PE: unwell, no signs of dehydration, no skin rash.


T: 37, HR: 68, RR:70, BP:126/86 (request BP in all limbs if you suspect that it is elevated); O2: 96%
BP: LA 126/86, RA: 152/92, LL: 69/41, RL: 63/59
Chest examination: diffuse bilateral wheezy chest and crackles with intercostal retractions; difficult to auscultate the heart due to noisy
breathing,
Abdomen: hepatomegaly 4cm below right costal margin
Palpate for femoral pulses! Part of baby check!!!!

Differential diagnosis:
a. Bronchiolitis
b. Pneumonia
c. Congenital heart disease
d. Septicemia

Expectation:
- Admission to hospital for treatment of heart failure at PICU
- Request urgent blood test, ECG and CXR
- Arrange urgent pediatric cardiologist review and 2d-echo
- Start IV furosemide (lasix)
- Plan is for long-term followup.

Acute Asthma

Case: The mother of a 3-year-old child comes to ED complaining of cough, having difficulty breathing, and wheezing for the last 2 days.

Task
a. History (has 2episodes of DOB and cough waking him up at night overrthe last weak. His chest also has muscial sounds, has
had similar attacks for the last 1 year but not very serious; gets symptoms in the morning; had urti last week which resolved)
b. Physical examination
c. Diagnosis and management

Case 2: A 6-year-old boy was brought to the ED because of severe asthma not responding to home treatment.

Task
a. Assess severity (DO history and PE)
b. Manage the case

Case: A 3-year-old child is coming to your child with runny nose for 3-4 days. She suddenly developed SOB and was brought to ED and
diagnosed with asthma.
12

Task
a. History- had this attack for the first time; had eczema but resolved; father had asthma that is controlled on ventolin and smokes
at home but stopped.
b. Management: What is asthma; Medications: ventolin only; Asthma action plan and regular reviews

Features
- SOB, chest tightness, cough, expiratory wheeze, early morning symptoms, waking up at night

Mild Mod Severe Critical


Work of Normal Mild Moderate Exhausted
breathing
Mental Normal Normal Agitated Drowsy
state
Chest Silent
exam Oxygen
<92%
tachycardic

History
- Is my patient hemodynamically stable? Can you tell me more about it? did it happen for the first time? Does he get the symptoms
early in the morning? How SOB is he? Was he using neck muscles or indrawing of the chest? Did he get agitated or confused
during these episodes? Was he able to speak? Any bluish discoloration? Any precipitating factor?
- Risk factors: Is she allergic to anything like seasonal/pollen, pets, carpets, smoking, exercise, cold air? Does she get frequent
URTI? Does anyone smoke at home?

- PMHx: does she have any other allergic condition such as eczema, hay fever, asthma? Medical illness? BINDS (development
and immunization) - growth retardation if asthma is poorly controlled? FHx of asthma or other condition? Is the child using
another medications apart from the asthma medications? Any other known allergies?

Physical examination
- General appearance: pallor, dehydration, jaundice, LOC, cyanosis, work of breathing, accessory muscle use, able to speak
- Vital signs
- ENT and LAD
- Chest: chest wall abnormalities, expiratory wheeze or silent on auscultation
- Cardiac and abdomen

Treatment.
- Mild 6puffs/12 puffs wait for 20 minutes(1 puff = 100 mcg)
- Moderate 6/12 puffs q 20 x 3 doses + steroids (1mg/kg prednisolone x 3 days - short bursts)
- Severe - ventolin + ipratropium bromide (2/4 puffs) + steroid (IV/oral)
- Critical - oxygen, continuous nebulized salbutamol (0.5%) + ipratropium + IV steroids (Others: MgSO4, IV salbutamol,
aminophylline if indicated)

- What is asthma? Asthma is a condition that affects the small airway passages ofthe lungs. These become narrow in this
condition making it harder for air to flow in and out ofthe lungs,but the good news is prognosis is good. Many children improve
when they get older.

- Risk factors: No smoking inside the home. Remove soft toys from the child's room. Wash the bed linens under high temperature.
Try to vacuum the carpets everyday. Avoid any other allergens and precipitating factors. Avoid pets inside the home.

- I will refer you to asthma educator to educate you on the proper techqnique of giving medications. If you have further questions,
you can also be educated by your GP.
You will need to be seen by GP in next 48 hours.
Reading material.

Asthma Education (Action Plan)

Case: Your next patient in GP practice is a 3-year-old girl Samantha who was brought in by her father. She had cold and developed
breathing problems which necessitated for her to be brought to the local hospital. She was diagnosed with asthma and was prescribed
salbutamol puffer via face mask and spacer. Samantha has no signs of asthma today.

Task
a. Further history
b. Review prescription and administration technique
c. Asthma action plan

History
13

- Since when did the cold start? Since when did she develop SOB? Was the chest wheezy? Does she have cough often? In the
day? At night?
- How bad was her attack? Is this the first time you had to take her to the hospital? What happened in the hospital yesterday? How
long did you stay in the hospital for?

- Precipitating factors: URTI, smoking, pets, change in weather, exercise, inhaled allergen, cold weather, food allergen

Prescription and Administration Technique:


- More efficient delivery of medication
- Decreased side effects
- Make sure you have 2 ventolin puffers or check if puffer floats in water or shake it
- Clean in warm water with detergent and allow to air dry at least once a month
- Technique: Remove cap and shake the puffer. Insert puffer into the spacer. Place lips around mouthpiece. Press puffer. Breathe
in and out normally through the mouth 4 times.

Asthma Management Plan


When Well Be free of nighttime cough/wheeze/chest tightness
Have no regular wheeze/cough/chest tightness on
waking or during the day
Be able to take part in normal physical activity
without getting asthma symptoms
Need reliever medications less than 3x/week
(except if used before exercise)
Treatment: Continue usual medication
When Not Well Increasing night-time wheeze/cough/chest
tightness
Symptoms regularly in morning when you wake up
Have need for extra doses of reliever medications
(more than 3x a week)
Have symptoms which interfere with exercise
Treatment: Use preventer and Increase
medication
Acute Attack Will have one or more of the following: increasing
wheeze, cough, chest tightness or SOB
Difficulty with normal activity
Waking each night and most mornings with
wheezing, coughing or chest tightness
Feel that asthma is out of control
Need reliever puffer every 3 hours or more often
Treatment (Acute Asthma Plan): Sit upright and
stay calm; Take 4 separate puffs then breathe in
and out 4x wait 4 minutes then may start again)
and start oral prednisolone
Severe Attack Symptoms get worse quickly
Wheeze or chest tightness or SOB continue after
using reliever medication or return within minutes
of taking reliever medication
Severe SOB, inability to speak comfortably,
blueness of lips
Treatment as acute asthma plan
If ambulance delayed get someone to drive you to
the nearest hospital emergency department

Asthma First Aid Plan


- 4 x 4 x 4 <6 years old
- 6 x 6 x 6 >6 years old
- Repeat x 2 doses then call 000

Explain, demonstrate and check the use of MDI

Case: A mother of an 18-month-old girl who was recently diagnosed with asthma is in your GP clinic. She was treated in ED yesterday and
was advised to use MDI with spacer and mask

Task
a. Relevant history
b. Check and demonstrate use of space and puffer
14

c. Write asthma action plan and explain further management to the mother

History
- I understand that you are quite worried about your daughter because of her hospital stay. May I know a bit more about this
episode?

- Attack: Can you tell me more about what happened? How SOB was she? Was she using her neck muscles? Was she alert,
agitated or confused? Was she able to talk or was it getting difficult for her to speak? Did she turn blue? Was she able to feed?
What was the Precipitating factor?Did you give any medications? For how long did she stay in the hospital? What treatment was
given in the hospital? What medications were given after discharge? Is it the first episode that required hospitalization?

- Pattern: When was she diagnosed with asthma? What medications were she on before the most recent attack? ? Do you know
how to give the medication. I will check your technique and demonstrate the proper technique later. How frequent are the
attacks? How does she feel in between the episodes? Does she get any symptoms before exercise or any activity?

- Risk factors: Is she allergic to anything like seasonal/pollen, pets, carpets, smoking, exercise, cold air? Does she get frequent
URTI? Does anyone smoke at home?

- PMHx: does she have any other allergic condition such as eczema, hay fever, asthma? Medical illness? BINDS (development
and immunization) - growth retardation if asthma is poorly controlled? FHx of asthma or other condition? Is the child using
another medications apart from the asthma medications? Any other known allergies?

- Technique
Can you show me how you do it? It is very important to use the inhaler correctly so that medication can reach deep into the lungs
to treat the asthma. It is important to know that it is the inhalation technique and not the pressure that is important. There are 2
techniques: open and closed-mouth which is the preferred technique.

MDI
- Remove the cup. Hold it upright and shake it for 1-2 seconds. Breathe out gently. Place mouthpiece in between your teeth and
seal lips around it. Tilt your head back slightly with your chin up. Start to breathe in and squeeze the canister and continue to
breathe in as deep as you can over 3-5 seconds through the mouth. Remove puffer from your mouth and hold breath for 10
seconds and breathe out gently. Replace the cap on. Wait for 1 minute and repeat inhalation if you need to. Rinse mouth after
use of inhaled steroids.

MDI + spacer and mask


- If you're using puffer with space and mask. Remove inhaler cap and shake it well. Attach the puffer upright to the end of the
spacer. Place the spacer mouthpiece in between the teeth and close lips around it. If using

with a face mask, place face mask over the mouth and nose to ensure a good seal. Press the inhaler once and breathe in and
out normally for about 4-5 seconds or take 4 normal size breaths. Remove inhaler from spacer and replace the cap and repeat
the steps if extra puffs are needed. Spacers should be cleaned regularly with warm soapy water or ordinary household
detergents and dried in sunlight every 10 days or monthly. Don't rinse or wipe.

- How to find out if MDI is empty? Always keep a spare one. You can use water floatation test. It floats if it is empty. You can
shake it and hear the sound but this is not accurate. Counting the dose with a gadget called a doser is the best way. If symptoms
are occasional, it is best to replace after every 6 months even if it has medications. Check the expiry date and replace if expired.
Do not press it to confirm if it is empty because it can still give some mist.

- I will write for you the Asthma action plan.

Chronic Asthma

Case: A 3-year-old girl was discharged from the hospital 2 days ago because of asthma attack. A father comes to see you in your GP clinic
for followup.

Task
a. Relevant history (she was SOB but attack was mild; had asthma for the last 1 year on ventolin PRN; triggers: URTI 3 days ago;
b. Further management and explain to father

Attacks Medications
Infrequently >6-8 weeks SABA PRN
Episodic
Frequently <6 weeks SABA + steroid
Episodic
Persistent Most of the days SABA +
LABA/steroid

0-3 MDI + spacer (mask + small volume spacer


3-5 MDI + small-volume spacer
5-8 MDI + large-volume spacer
>8 MDI

History
15

- I understand that you are quite worried about your daughter because of her hospital stay. May I know a bit more about this
episode?

- Attack: Can you tell me more about what happened? How SOB was she? Was she using her neck muscles? Was she alert,
agitated or confused? Was she able to talk or was it getting difficult for her to speak? Did she turn blue? Was she able to feed?
What was the Precipitating factor?Did you give any medications? For how long did she stay in the hospital? What treatment was
given in the hospital? What medications were given after discharge? Is it the first episode that required hospitalization?

- Pattern: When was she diagnosed with asthma? What medications were she on before the most recent attack? ? Do you know
how to give the medication. I will check your technique and demonstrate the proper technique later. How frequent are the
attacks? How does she feel in between the episodes? Does she get any symptoms before exercise or any activity?

- Risk factors: Is she allergic to anything like seasonal/pollen, pets, carpets, smoking, exercise, cold air? Does she get frequent
URTI? Does anyone smoke at home?

- PMHx: does she have any other allergic condition such as eczema, hay fever, asthma? Medical illness? BINDS (development
and immunization) - growth retardation if asthma is poorly controlled? FHx of asthma or other condition? Is the child using
another medications apart from the asthma medications? Any other known allergies?

Management
- I will talk about what asthma is, how to manage symptoms, risk factors, how to use medications and give you the asthma actiong
plan.
Asthma: it is a condition that affects the small airway passages ofthe lungs. These become narrow in this condition making it
harder for air to flow in and out ofthe lungs,but the good news is prognosis is good. Many children improve when they get older.
- Risk factors: No smoking inside the home. Remove soft toys from the child's room. Wash the bed linens under high temperature.
Try to vacuum the carpets everyday. Avoid any other allergens and precipitating factors. Avoid pets inside the home.
- Exercise: If she has symptoms before exercise, give 2 puffs ventolin 5-10mins before exercise
Medication use: Can you show me how to deliver the medications?
- Asthma action plan: I will give you an action plan 1 copy for me, for you and your child and at childcare. This discusses what
symptoms she has when the child is well, unwell, and what to do when she is having acute attack and severe attack. When
having an acute attack, sit the child in a comfortable position and follow the 4x4x4 rule. Give salbutamol 1 puffthen 4 breaths in a
spacer then take 4 puffs in 4 minutes wait 4 minutes. If with no improvement, repeat another4 puffs. If with no Improvement or
with danger signs like DOB, child is drowsy, unable to speak, turning blue call 000 and say "my child ishavingasevere asthma
attack". Keep giving 4 puffs every 4 minutes until the ambulance arrives.
- She needs regular followups for review of her symptoms, medications, and growth and development. Reading materials.
Advice annual flu and 5-yearly pneumococcal vaccine.

Criteria for Discharge:


- Ventolin requirement more than 3-4 hours or less frequently
- No oxygen requirement for 24 hours

CARDIOLOGY AND PERIPHERAL VASCULAR DISEASE

Innocent Murmur

Case: Your next patient in GP practice is a 4-year-old boy seen by your colleague who had a soft heart murmur. Parents are here to
discuss with you because you are their family doctor and to decide if further investigations are required for the child.

Task
a. Relevant history (no recurrent infections, cousin with “hole in heart”, pregnancy was good, immunization up to date, growth okay)

b. Examination (growth chart normal, VS normal as well as BP on all 4 limbs,apex beat not displaced, musical soft vibrating
murmur, midsystolic 2-3 at the base of the heart which changes with position and respiration; no palpable thrill)
c. Diagnosis and management

Features of organic murmur


- Loud
- Associated with palpable thrill
- Diastolic component
- Radiates
- Associated with cyanosis, significant symptoms like SOB, recurrent infections, not growing well
- Does not change with position and respiration

History
- What made you come today? Do his lips turn blue? Is he able to play or exercise well? Any SOB or difficulty breathing? Any
recurrent chest infection? Is he growing well? Any concerns about his growth and development?
16

- BINDS (immunization)
- FHx of heart disease

Physical examination
- General appearance: pallor or cyanosis, dysmorphic features of Down syndrome, marfan, etc.
- Growth chart
- Vital signs: BP on all limbs, radiofemoral/radio-radial delay
- Heart: inspection: precordial bulge, apex beat, palpable heaves/thrill, auscultation: character of murmur

Diagnosis and Management


- Let me reassure you that it is an innocent murmur. It is a very common condition in young children and caused by hyperdynamic
circulation when child is growing. The good news is your baby is thriving well. There is no history of recurrent chest infections
and he is not getting blue. He has good exercise tolerance and the murmur is soft and musical and systolic which does not go
anywhere and changes with posture or respiration. These are suggestive of an innocent murmur. Usually, they settle down at
around 5-7 years but I would like to review him frequently.
- Does he need to see the pediatrician? Not at this stage. But if you are concerned, I am happy to give you a referral.
- Should investigations be done for him? At this point, I don’t think it is required, but again, if you are concerned, I can organize
basic investigation including ECG and CXR.
- Can he continue to play? Absolutely!
- Review. Reading material.
- Red flags: breathing difficulty, development problems, cyanosis, recurrent infections

Ventricular Septal Defect

Case 1: You are a GP and a 3-month old boy was brought by his mom who is concerned because the pediatrician said that her son has a
“hole in the heart”

Task
a. History (generally tired)
b. Examination (+Grade III/VI systolic murmur)
c. Diagnosis and management

Case 2: You are a GP and a 5-year-old girl was brought in by her mother. The family has recently moved to your area. Mom brought a
letter from the previous GP that says the child has a heart murmur and should be reviewed from time to time. The child is undergoing a
dental checkup within 1 week time.

Task
a. History
b. Physical examination findings
c. Management

History
- Please tell me how your baby is at the moment? Does he have any episodes of bluish discoloration of his face? Does he have
breathing problems during feeding? Does he sweat a lot during feeding? Have you noticed him to be pale at any time? Do you
think he is thriving well? Has he had a respiratory tract infection recently? Any cough? Have you noticed any swelling around the
eyes or other parts? Fever or frothing of the mouth?
- Is this the first child? Any problems during the pregnancy (Rubella, down syndrome, GDM)? How was delivery? Did he require a
lot of resuscitation at birth? Do you remember his apgar score? Was he blue at birth? Is he feeding well? Are you concerned
about his growth?
- FHx of heart problems?

Task
- General appearance: pallor, cyanosis, dehydration; alert and active
- Vital signs and growth chart
- Pulse: radioradial or radiofemoral delay
- Dysmorphic features of down syndrome
- Chest:
o Inspection: increased work of breathing? accessory muscle use? Intercostal, subcostal retractions? Alar flaring?
o Equal air entry? Any added sounds? Crackles at the base of the lungs?
- Cardiac
o Inspection: visible pulsations; apex beat (tapping, heaving)
o Heart sounds, and character of murmur; associated thrill? Press on the liver to increase venous return and accentuate
the murmur
- Abdomen: soft, tenderness, visible distention of abdomen, palpate the liver? Is the liver pulsatile? Sacral edema?
- Urine dipstick

Management (Case 1)
- As you know, the heart has four chambers. The right side of the heart receives blood from all over the body and the left side
pumps out this blood after getting the oxygen from the lungs. Most probably, your child has a hole within the septum separating
17

the right and the left lower half. It causes mixing of good and bad blood. Usually, it doesn’t give any symptoms to the child.
Depending upon the size of the hole, complications may develop.
- VSDs can be divided into:
o Small <0.5cm2 ; asymptomatic
o Moderate 0.5-1.0 cm2; recurrent URTIs

o Large >1.0 cm2; heart failure and recurrent chest infections (pneumonia)

- 75% of VSDs are present in the membranous part of the septum;


- At the moment, we need to send your child for an echo of the heart to determine the location of the hole, the size of the hole, and
the overall function of his heart. Majority of small to moderate VSDs close spontaneously by the age of 5 years. We need to do
regular follow-ups to check if your child is thriving well and if he is developing heart failure or not.
- If it doesn’t close by the end of 5 years, I will send you to a cardiac surgeon and they will probably discuss with you surgical
treatment to close the hole. There are 2 options depending upon your child’s condition: either the surgery will be done through
catheterization or open heart surgery.
- Once the results of the echo are back, I will discuss with you regarding antibiotic prophylaxis or prevention for any dental
procedures that he might undergo in the future. Usually, half an hour before the procedure, we will give him amoxicillin syrup
according to his weight to prevent any bugs reaching his heart via the blood. These holes are particularly susceptible to
developing infections and further complications.
- Meanwhile, if he develops any symptoms: not feeding well, poor weight gain, breathing problems, repeated respiratory infections
please come back or go to hospital ASAP because they might indicate cardiac failure which needs urgent treatment.

Management (Case 2)
- As you know, 50% of small VSDs close spontaneously, mostly before the age of 5 years. For your daughter, it is best for her to
see a pediatric cardiologist. They will decide if any intervention is needed at this point. I will write some tests for you which you
will take to the cardiologist including FBE, UEC, ECG, CXR, and Transesophageal Echo.
- As you know there are bugs within the mouth that can reach the cardiac hole and attack the tissue especially during a dental
procedure. It is important for her to have a single dose of amoxicillin at least half an hour before the procedure to prevent any
such complications.
- Prophylaxis:
o High risk: Prosthetic cardiac valves, previous infective endocarditis, cyanotic CHD, CHD with completely repaired
defects, cardiac, RHD within indigenous Australians
o Extraction, periodontal procedure, replanting of avulsed teeth must give antibiotics
- RCH Guidelines:
o Low risk: all children with structural CHD (except isolated ASD secundum), OPERATED heart defects (except repaired
ASD and secundum and PDA), acquired valve disease, endocardial pacemakers no longer recommended
o High risk: unrepaired CHD, previous endocarditis, prosthetic valves, conduits, shunts ampicillin/amoxicillin 50mg/kg
IV pre-op 30 minutes before

If sensitive to penicillin: vancomycin 20mg/kg IV infused over 1 hour pre-op


o Procedures:
Low risk Local anesthesia (Dental procedure or oral/naso/pharyngeal) amoxicillin 50mg/kg oral 1 hour
preop/clindamycin 20mg/kg oral 1 hour preop or vancomycin 20 mg/kg IV
Low risk General anesthesia (dental procedures or ONP): Ampicillin 50mg/kg IV at induction, clindamycin
20mg/kg orally or by IV infusion or vancomycine 20mg/kg IV infused
High risk: also for gastro of genitourinary procedure: ampicillin/amoxicillin 50mg/kg IV + Gentamycin
1.5mg/kg IV at induction
- Can she play sports? Yes but if she experiences symptoms such as breathing problems, repeated infections, not feeding well,
poor weight gain

VSD Counseling

Case: A 6 month old baby Suzie diagnosed with VSD by echo. Baby is thriving well. Weight and height on the 50th percentile and has no
trouble with breast feeding.

Task
a. Explain condition to father
b. Answer his questions
18

Counseling
- Hello Mark, I have Suzie’s report and I know you have come to see me to discuss it. As you know, echocardiogram is a test that
uses sound waves to create a moving picture of the heart to assess his function and structures. During Suzie’s
echocardiography, the doctor identified Ventricular Septal Defect. Have you ever heard about it?
- Please try not to worry. It’s not very serious and it’s good we diagnosed it. VSD is the most common congenital heart defect. The
heart has 4 chambers. And the 2 low pumping chambers of the heart are called the ventricles. And normally, there is no
connection between them. In case of VSD, there is a hole in the wall between right and left ventricles. Draw! As a result, blood
flows from left ventricle to right ventricle at high pressure often producing a loud murmur with each heartbeat (Pansystolic
murmur). The smaller the VSD the louder the murmur.
- A small VSD causes no problems and often heal/close on their own. Suzie is asymptomatic and she is feeding and growing well.
They are all good signs that VSD is small and does not cause significant strain on her heart. She needs to have regular follow
up. Most likely, this hole will close with time or it might stay the same. It will never increase in size. In case it doesn’t close with
time, if it doesn’t affect your general health and growth and doesn’t place strain on her heart, it can be left alone with regular
follow ups. Plus she will require antibiotic prophylaxis before dental extraction or any surgery for prevention of bacterial
endocarditis.
- Why did it happen? It is a congenital defect. That means it occurs before a baby is born. The heart begins like as a hollow tube
and the wall between left

and right parts of the heart forms later. If it doesn’t occur properly, it can lead to an opening being left within ventricular
wall/septum.
- The risk factors for cardiac abnormalities include:
o Family history of congenital heart disease (1st degree relative: risk 3%)
o Drug exposure (Anticonvulsant, lithium)
o Maternal diabetes
o Chromosomal abnormality (Down’s syndrome)
- In most cases, we cannot identify possible cause.
- Does this increase chance of having another baby with heart defect? The chances are increased very slightly. Your wife will
need to have a more detailed scan during next pregnancy. Best time to have a look at the heart is at 20weeks.
- Can my child live normal life? Yes, prognosis for small VSD is very good and it will not affect the quality of life.
- Will she need surgery? It’s very unlikely. Surgery is usually performed for large VSD. In most cases first 3 -4 months of life.
However, even large VSD sometimes can be left for observation if it doesn’t cause much trouble. Because we hope it will get
smaller with time. And surgery will be performed only if it remains large or associated with clinical problems.
- Can she be immunized? Yes, immunization is an important part of preventive medicine. By immunizing her you will protect Suzie
against many serious infections. Having a heart defect is not a contraindication for immunization
- Suzie will have regular follow ups with me and the cardiologist. We will assess her general condition, growth and development.
Later on, the cardiologist will repeat echo to see if there is an improvement in the size of VSD.

Kawasaki Disease/Mucocutaneous Lymph Node Syndrome

Case: You are a GP and a 3-years-old boy was brought to by his mom because the child has fever and a rash.

Task
a. History (fever x3-4 days, runny nose, rash x 1day ago, poor appetite)
b. Physical examination (irritable, mildly dehydrated, RR inc. PR130, T:40C; dry parched lips, tonsils erythematous, flat reddish
irregular that blanches on pressure on the hands and feet with desquamation, swelling of hands and feet)
c. Differential diagnosis to examiner
d. Investigation and Management

KAWASAKI DISEASE
- Mucosal involvement (dry cracked lips, strawberry tongue)
- Hands (edema/desquamation)
- Eyes (bilateral nonpurulent conjunctivitis)
- Adenopathy (unilateral >1.5cm)
- Rash (pleiomorphic)
- Temperature >5days *
- Management:
o Admit
o Aspirin
o Intravenous immunoglobulin
o Echocardiogram – do every 5 years to rule out coronary aneurysm

History
19

- Can you tell me more about his fever? Since when? Is it continuous or intermittent? Does he have shakes/chills? Did you give
medications? Did it help? Is this the first time for him to have high fever? When did the rash appear? Where did it start from? Can
you describe the nature of the rash? Is it itchy? Scaly? Do you think it is progressing? Any history of similar rash before? Any
other symptoms like N/V/diarrhea/problems passing water? Does he complain of headache? Cough? How is his appetite? Is he
eating and drinking well? Did he have a fit with the fever at any time? Do you think he came into contact with someone with
similar symptoms (home/childcare/kindergarten)? Is his immunization up to date? How was your pregnancy and delivery with this
child? Any concerns with his growth and development up to now? How is the home situation? Any stress? Any problems? Who
takes care of the child most of the time? Do you know if she has any allergies?
- Any family history of bleeding disorders, rheumatoid arthritis?

Physical examination
- General appearance: pallor, jaundice, dehydration; is the child active and alert
- Vital signs
- Neck stiffness
- Palpate lymph nodes
- ENT: lips (dry red cracked), eyes (redness/discharge),
- Observe the rash (where is it located, what type of rash is it) maculopapular polymorphous desquamating rash that includes
the palms and soles sometimes associated with small joint swelling
- Auscultate the chest and heart: Murmur
- Abdomen: distention, tenderness, masses, bowel sounds
- Inguinal orifices
- Urine dipstick

Differential Diagnosis
- Kawasaki disease/Mucocutaneous Lymph Node Syndrome
- Hand-foot and mouth disease (punctuate)
- Drug allergy
- Scarlet Fever
- Juvenile Rheumatoid Arthritis
- Meningococcemia

Investigations
- FBE (normocytic, normochromic anemia), ESR/CRP, ECG, LFTs, UEC, Echo, MSU

Management
- Most likely what your child has is a condition called Kawasaki disease. It is a fairly common condition where the body’s immune
system starts acting against its own tissues mainly the skin and blood vessels (especially coronary vessels supplying the heart).
Sometimes the liver and spleen may be involved. In majority of cases, it is a benign and self-limiting condition. The exact cause
is not known but certain viral infections are thought to provoke the immune system.
- We need to admit him to the hospital because he is dehydrated and we need to run some tests on him. He needs to be seen by
a pediatrician and we will start him on IV fluids. We will need to give him oral aspirin (3-5mg/kg/day). This reduces the risk of
thrombocytosis.

- I have heard that ASA is not good for kids. WE usually give .low-dose aspirin that do not cause Reye syndrome. Sometimes,
higher doses have been used when required without any complications.
- We will also give him IVIg to counteract his body’s immune system (2g/kg/Single slow infusion).
- Kawasaki carries a risk of dilatation of coronary vessels which we call as aneurysms. It is seen in as many as 30% of patients so
it is recommended to do an echo of the heart within 2 weeks of development of symptoms and we will repeat it in 6-8 weeks to
make sure the heart is alright and once every 5 years.
- High-dose steroids and plasmapheresis reserved for non-responders

DERMATOLOGY

Head Lice

Case: You are a GP and 6-year-old boy comes with his mom because he was found to have lead lice.

Task
a. Relevant history
b. Management

History
- Can you tell me since when you noticed the problem? Does he go to school regularly? Are you aware of anyone at school having
head lice? Any note from the school? Is he itching a lot? How is his general health? How many people are there at home?
Anybody else at home itching?

Management
- First of all, let me reassure you that this condition is NOT related to hygiene or cleanliness. Medically, it is called Pediculosis
hominis. This is an insect that lives in the hairy areas of the body such as the head, eyebrows, eyelashes, rarely, within the pubic
area. It is spread by direct transmission from head to head. This insect irritates the head causing itching. It lays eggs that are
called “nits” that usually hatch within 7 days and around 10-12 days, they mature and lay eggs. The nits can be seen at the base
of the hairs where they are glued to the hair.
20

- The treatment fortunately is simple, but needs to be done regularly for complete eradication. I will write for you permethrin
solution that you can easily find OTC. You need to wet the hair, apply the solution to the base of the hair, leave it on for 10
minutes, wash the head with plain water, and may apply shampoo afterwards. This treatment is quite effective and kills both the
insect and the eggs. However, you need to remove the eggs with the help of a fine-toothed comb after the treatment. After 7
days, you need to repeat the same procedure one more time. Simultaneously you need to wash his beddings with hot water.
Don’t allow anyone to share his towels and combs. You need to soak the combs in hot water for 10 minutes after the treatment.
Usually, in pharmacies, you will find anti-lice kits that contain permethrin in the form of shampoo, a fine-toothed comb and a
conditioner.
- There is an alternative home treatment as well that says you mix 1 part vinegar with 1 part water and wash the head with that
solution followed later on by combing.
- Is it alright to send him to school? Once you have given him the treatment to the child, you can send him to school the next day,
but you need to inform them so that other kids get checked as well.

- If not effective, add coamoxiclav.

Tinea Corporis
Case: Your next patient in the GP practice is a 12-year-old boy, Trent. with his father john. Trent has developed an itchy spot under his
chin. It has been there from last 2 weeks and its growing in diameter

Task
a. Take history
b. Examiner will show you a photo
c. Diagnosis
d. Investigation if you think is necessary

Differential Diagnosis
a. Tinea Corporis
b. Erythema Multiforme
c. Psoriasis
d. Tinea versicolor
e. Dermatitis (coin like)
f. Secondary syphilis

History
- When did it start (One week ago)? Where? Is it itchy? Is it the fist time(Yes)? Do you have any pets? Any change in the soap?
Anyone in the family having similar condition (No)? Has he started on any drugs/medication (No)? Any fever or joint pain?
Constitutional symptoms? Any lumps and bumps in the body? Any family history of skin conditions? How’s the bowel and
bladder? How’s the health in general? Loss of appetite? Sleep is good (Yes)? Do you think there’s any incidence of unprotected
sex? Any travel? No travel

Diagnosis and Management


- From the history and looking at the lesion it is most likely Tinea Corporos caused by Tinea Rubrum or Microsporum Canis. It’s a
very classical lesion where there is central clearing as it expands. It can involve anywhere in the body, hair, nail. Most likely it
spread through pets, could be human to human also. I will take the skin scraping and send for KOH (potassium hydroxide)
application to demonstrate hyphae. At this stage I will give you anti fungal to apply. Topical Terbinafine cream twice a day for one
week. Clotrimazole, econazole, imidazole cream 2-4 times per day for 4 weeks.
- If it is not responding to the cream, or involves the hair and nails: give Oral tervinafine or Griseofulvin. (Tinea capitus)
- Advice the hygiene, as it can spread to other where in the body.
- Review in 1 week.

Scabies
21

Case: A 3-year-old baby is brought in by parent complaining of itching and rash over the right hand since the last week.

Task
a. Take history from mother
b. Physical examination
c. Diagnosis
d. Management

History
- History of rash: Since when? How did it start? Where? Is it itchy or scaly? Is it oozing? What color is the rash is it blue or red?
Has the rash been spreading? Any aggravating or relieving factors? Been worsening? Is the itchy more severe at the night
(yes)? Is there any trigger factors for the itch? Temperature (Wash with hot water make the itching worse)? Is it the 1 st time? Has
he been in contact with people with similar condition? Interfere with soap? Any associated fever runny nose or flu like
symptoms?
- BINDS? Allergies? History of asthma eczema and hay fever? Does he go to child care? Are you aware of anyone at child care or
at home with similar complaint? Did he travel anywhere recently? How are his waterworks, bowel habit and general health?

Physical Examination
- General Apperance
- Vital Signs (If fever check neck stiffness)
- Growth chart
- ENT examination
- Lymphadenopathy
- CVS and abdomen examination
- Local examination of the hand: Use both hands to compare; Check the site, colour of the rash, blanching or not blanching,
macular or popular of the rash. Any discharge from the rash? In the web-space? Can I see any burrows? I would like to take the
skin scraping from the lesion and look under the microscope: Sarcoptes scabiei

Diagnosis and Management


- Most likely Alex is suffering from a skin condition called scabies. It’s caused by mites called sarcoptes scabie where the
characteristic lesion is burrows. There can also be papules and vesicles and that’s very itchy. It’s a common condition anywhere
below the neck. The usual location is the palm, feet, and axilla. The female scabies mite burrows underneath the skin to lay
eggs. These eggs hatch into mites. There’s an antigen in the excreta of these mite which causes hypersensitivity reaction.
Itching is worse at night and the condition is highly contagious. Spread is primarily through close contacts. However, it can also
spread through sexual contact in adult. Itching can cause a bacterial infection. We can treat that with antibiotics. This condition is
not serious although it’s very uncomfortable. 5% Permethrin cream needs to be applied to the entire body from jaw line below
including the nails the flexural area and genitals. Leave it overnight and wash off thoroughly the next day. At the same time, the
family needs to be treated even if asymptomatic. Dose can be repeated after a week for severe cases. Avoid hot bath and
scratching. Maintaining a good hygiene is important. Wash the clothes bed linen in hot water. It’s a highly contagious condition.
- Reading material. Review

Chickenpox

Case: You have a 13-years-old presenting to your GP clinic complaining of fever and skin rash.

Task
a. History (fever 38, intermittent, started 3 days ago, usually in the evening, relieved by Paracetamol, no chills; rash started on
anterior chest and spread on the back, red rash turning pimply and vesicles, no crusting, extremely itchy; no conjunctivitis, runny
nose, pain; decreased appetite, no problems with waterworks and bowel movements; immunization up to date; classmate had
chickenpox)
b. Physical examination (not unwell, maculopapular vesicular rash on face, lips and trunks; vital signs: T38, PR 100, BP and RR
normal, ENT normal, chest and heart normal, abdomen normal)
c. Diagnosis and management

History
- I understand you have come to see me because you have fever and rash, when did it start? Which came first, the fever of rash?
Did you measure your temperature? Is it up and down or constantly high? Do you have chills and shivers? Can you describe the
rash for me? Is it red or blue? Is it itchy? Where in your body did it start? Did it spread anywhere from there? Is it the first
episode? Do you have runny nose, cough, SOB? Do you have headaches, neck stiffness, any change in the way you walk? Do
you feel nauseous or have you vomited? Any abdominal pain? How are your waterworks? Do you have regular BM?
- How is your general health? Medication? Allergies? Immunization? Whom do you live with? Is anyone at home or in school sick?

Physical appearance
- General appearance: pallor and dehydration
- Rash morphology: maculopapular vesicular rash on face, lips and trunk; scratch marks
- Vital signs
- ENT
- Neck stiffness and lymph nodes
- Chest and heart
- Abdomen
22

Diagnosis and Management


- According to history and physical examination, you have a condition called chickenpox. Do you know what it is? Chickenpox is
an infection caused by a virus called Varicella zoster. It is an infectious disease and you most likely caught it from your
classmate.
- Treatment is symptomatic. Make sure you drink ample fluids, have a nutritious diet, rest and use panadol for fever. I know the
rash is itchy but try to avoid scratching. You can also use calamine lotion to relieve itching. After bath, it is better to pat dry the
skin with a soft clean towel. Do not RUB. If severe, I can give antihistamine. It is advisable to take a bath everyday. Half a cup of
sodium bicarbonate could be added to the bath to prevent secondary infection. The rash will heal within 7-10 days leaving
normal skin unless it becomes infected. It is an infectious disease and you shouldn’t go back to school unless the rashes are fully
crusted.
- Have you been in contact with any pregnant woman, newborn babies or immunocompromised people (steroids, chemotherapy,
HIV)? Give immunoglobulin.

- Varicella vaccine is effective in preventing chickenpox in those who are already exposed if used within 3-5 days. It will not be
100% effective and can still have breakthrough infection. If they do, usually milder. Red flags: Please come back to see me if the
rash gets infected, you have severe headache, nausea, vomiting or neck pain, trouble walking or have any general concerns.
- Otherwise, I will see you in a few days to check on your improvement
- Complications: secondary bacterial infection leaving pitted scar, viral pneumonia, and thrombocytopenia, acute cerebellitis,
meningoencephalitis
- (Infectivity 1-2 days before and until all rash crusted)

Erythema Infectiosum (Fifth Disease)

Case: You are GP and parents Jen and John came to you because they are concerned the babysitter is mishandling their 5 years old son,
Matthew. They left the boy and went to a party and when they returned the baby was asleep. The babysitter said everything was fine. In
the morning, Matthew had marks on his cheeks, which looks like he has been slapped by somebody. They immediately brought the baby
to you.

Task
a. History
b. Physical examination
c. Management

History
- I understand that you have come to see me because you’re concerned about your baby sitter slapping your son. How often does
she look after matthew? Is this the first time you’re concerned about her behavior? How is her relationship with Matthew? Has
matthew ever complained about her? Apart from the marks on his face, have you noticed anything else, bruising, etc? did you try
to contact the baby sitter?
- Does Mathew have a fever? How is his appetite? Is he as active as usual? Have you noticed any lumps and bumps in his
body? Have you noticed a rash anywhere in his body? How is his waterworks? Does he have regular bowel movements? Is he
generally healthy? Is he on any medications or does he have any allergies? Any concerns about his growth and development?
Are his immunizations up to date? Does he go to school? Do you know if anyone else in school sick?

Physical examination:
- General appearance: could you please describe the marks on matthew’s face (bright red flushed cheeks) with perioral pallor.
Signs of dehydration?
- Vital signs: BP, PR, T, RR, growth chart?
- ENT and LN: enlarged cervical lymph nodes
- Chest, heart and abdomen

Management
- According to the history and PE, he most likely has a condition called Erythema Infectiosum. It is AKA Fifth disease or “Slapped
cheek” syndrome. It’s common in young school-aged children and is caused by a virus (Parvovirus B19). Some children will have
mild fever and malaise. The rash is typical. It first appears on the face and it looks like he has been slapped on the cheek. After a
day or so, small red rash appears on the limbs and body. It fades over the next few days and centrally clears forming a reticular
or lace-like

pattern. This reticular rash may reappear again with heat, cold or friction in the next few weeks and even months. It is a clinical
diagnosis. If you are very concerned, I can order a blood test, a serology to confirm my diagnosis.
- Treatment is symptomatic. Make sure matthew drinks ample fluids, has rest and a nutritious diet. Panadol can be given for the
fever. Calamine lotion should be used if the rash is itchy. Matthew should wear a broad-brimmed hat when he is outside.
- It is a mild infection. Complications are very rare, but it can seriously affect unborn babies (hydrops fetalis). Has Matthew been in
contact with any pregnant woman? It is a contagious disease transmitted through droplets and direct contact similar to flu. But
the good thing is Matthew is not longer infectious because of the appearance of the rash. He can go back to school anytime.
23

Roseola Infantum(sixth disease)

Case: Your next patient in GP practice is an 18-month-old child brought in by mother due to high fever for 3 days. He has been seen by 2
other doctors about his persistent fever. He then developed a rash this morning hence he was brought for consultation.

Task
a. History (fever x 3 days then got better after rash disappeared; started on body then spread to extremities; good appetite)
b. Physical examination (blanching red rash)
c. Diagnosis and management

Differential Diagnosis
- Roseola infantum
- Rubella (usually low-grade; small, discrete pink rash; face down; postauricular and occipital LAD)
- Rubeola (Measles): conjunctivitis, coryza, cough and Koplik spots (white spot in bright red mucosa); blotch erythematous rash
from hairline then goes to the body
- Chickenpox
- EBV
- Scarlet fever
- Meningococcemia
- Kawasaki disease
- Allergic reaction

History
- Hello. I understand that you have come to see me today because your child is feeling unwell. Can you tell me more about his
fever? Did you measure his temperature? Is it up or down or constantly high? Does he feel miserable when he has the fever?
Did you give him anything for his fever? How often and has it been effective? What did the other doctors say? Did they prescribe
any medications? When did you notice the rash? Where on his body did it start? Did it spread anywhere from there? Can you
describe the rash for me? Is it red or blue? Is it itchy? Did his fever improve when the rash appeared? Is he active or drowsy? Is
he eating or drinking well? Have you noticed watery red eyes, runny nose, or cough? Has he been pulling his ears? Vomiting?
How’s his waterworks? Does he have regular bowel movements? Is he generally healthy? Is he on any medications? Any
concerns about his growth and development? Is his immunization up to date? Do you have other children? Is he attending
childcare? Do you know if anyone has been sick there? Is anyone sick at home?

Physical examination
- General appearance: LOC (active, alert, irritable, lethargic); signs of dehydration; description of rash (blanching red
maculopapular rash on the trunk, arms and face, non-scaly)
- Vital signs
- ENT and neck: conjunctivitis, erythema or pharyngitis, lymphadenoapthy, neck stiffness
- Lung, heart and abdomen

Diagnosis and Management


- According to your son’s history and PE, he has a condition called Roseola infantum. Have you ever heard about it? It’s a viral
infection which is common in children under the age of 3 (Human herpres virus 6). 95% of children are infected by this virus at
the age of 2 and around 30% will develop symptoms. Typically, it starts with a very high fever for about 3 days and then the fever
goes away but a widespread red rash appears first on the body and neck, then later on the face, arms and legs. The rash will last
1-2 days and it will fade without leaving marks.
- Does he need antibiotics? He doesn’t need antibiotics. Treatment is symptomatic. Make sure he drinks ample fluids, rest, and
nutritious diet.
- Children with roseola recover fully usually within a week. It’s a contagious disease. Transmission is through direct contact or
droplets like flu. The child is most infective during the period of rash. Bryan should have his own utensils, glasses, plates and
toys and it is better to isolate him from other children.

Acne Vulgaris

Case: Your next patient in GP practice is a 14-year-old female who came in because of a rash on the skin of the face.

Task
a. History
b. Diagnosis based on picture
c. Management

History
- Please tell me more about the skin rash? When did it start? Where did it start first? Can you describe the rash to me? Is it red,
itchy, scaly? Do you feel it is hot to touch? Do you think it is getting worse? Is this the first time for you to have this rash?
Anything that is making it better (creams, sleeping)? Does anything make it worse like scratching, sunlight, with your periods?
Have you applied any creams up to now? Which ones? How many times? Any side effects? Does it help? What kind of
cosmetics do you use? Are they oil-based or water-based? How do you feel about yourself? How is your mood? Do you go out
often? Do you socialize much? Are you sexually active? Are you in a stable relationship? Are you on any type of contraceptives
like the pill? When was LMP? Are your cycles regular? Is there a FHx of similar problems? SADMA? Stressors?

Diagnosis and Management


24

- Your condition is medically called acne vulgaris. It is a very common problem. Studies show that around 95% of adolescents
suffer from some degree of acne at a certain period of their life. As you know, our skin has some glands within the superficial
layer, called sebaceous glands that produce or secrete sebum that moistens and protects the skin. Under the influence of

hormones, especially around puberty, the functioning of these glands is increased. Sometimes, these secretions block the
draining duct. The gland then becomes swollen and gets infected (Propionobacterium acne). With the infection, there is pain,
redness, feeling of warmth all over the face.
- It is important to treat acne to prevent the risk of scarring, superinfections, cosmetic defects, depression, and social isolation and
avoidance.
- Please understand, up till now, none of the studies have proven any relationship between acne and diet.
- First line medications:
o Benzyl peroxide OTC gel/creams/face wash (2.5% - 10%) once daily preferably in the morning
o Topical Retinoids (Vitamin A) Retin A/Stieva A applied to skin at night; use a sunscreen during the day; SE:
erythema, skin pigmentation, and hypersensitivity to the sun; CI in pregnancy
o Topical isotretinoin (gel) antibacterial and anti-inflammatory; SE: itching, burning, erythema; less irritant; CI in
pregnancy
o Azeleic acid (Skinoren cream) antibacterial and anti-inflammatory less irritating
o Topical antibiotic (Clindamycin/Erythromycin) used for 3 months to see efficacy
- We can use a combined therapy of benzoyl peroxide in the morning and either of the 4 at night.
- Second line medications:
o Oral Antiobiotics: Doxycycline 50mg/day x 3-6months; azithromycin, tetracyclines, trimethoprim
o OCP especially if with history of irregular periods to remove the androgenic effects on the sebaceous glands
- Third line medications: Refer to dermatologist where he will prescribe oral retinoids. It is usually reserved for cases of severe
acne, acne that is resistant to oral antibiotics, acne where extensive scarring has already begun. SE: secondary dermatitis,
fragile skin, myalgias/arthralgias, high levels of TG, dryness of skin and mucous membranes leading to conjunctivitis and
nosebleeds, teratogenicity
- I would like you to still try out a combination of benzoyl peroxide gel in the morning. You can use it once a day. It is better if you
wear sunscreen during the day. At night, I want you to apply a cream that contains vitamin A, but please remember to wash your
face before going out in the morning, because this cream makes you more prone to develop sunburns and skin pigmentation.
You need to use these for the next 3 months.
- Come back to me after that. If your condition still does not improve, I might switch to topical or oral antibiotics depending on the
severity. Please remember to avoid excessive washing of the face because it will make your skin dry and more prone to develop
infections. Avoid picking on the rash. You can use any soap but remember to choose those that are mild on your skin.
- Can I use cosmetics? It is advised to use water-based cosmetics because they will not clog pores.

Nappy Rash

Case: Your next patient in GP practice is a 5-month-old John because of rash in his buttocks. She is worried about her son because this is
the first time she noticed the rash (Pic given).

Task
a. History (started as red spots 2 days ago and increasing in size; 1st time; put some soothing creams but did not work; changes
nappy whenever baby cries, no smelly urine, irritable, diarrhea last 2 days)
b. Physical examination (irritable, red, erythematous rash; urine dipstick)
c. Diagnosis and management

Features
- Common Causes:
o Irritant dermatitis
Due to overhydration loss of epidermal barrier and direct exposure to fecal enzymes
o Candidiasis
o Seborrheic dermatitis
o Psoriasis
o Atopic dermatitis
- Uncommon:
o Staphylococcal
o Streptococcal
o Herpes simplex
- Rare:
o Tinea
o Zinc deficiency
- Virtually ALL nappy rash become colonized with candida
- Management:
o Use highly absorbent disposable nappies
o Often change cloth nappies every 2 hours
o Do not use wipes/soaps if with active rash
25

o Apply barrier creams after every nappy change


o Meds:
Hydrocortisone ointment + antifungal creams
If w/ staphylococcal infection mupirocin ointment or flucloxacillin (oral)

History
- Could you describe what kind of rash it is and where exactly? Is John having rash anywhere else? Do you think it is itchy? Did
you put any creams or medication? Is he your first baby? Any similar history of rash before? Any other problem in the past? Have
you changed any new products (shampoos/shower gels)? Is it affecting his sleep and feeding? Do you use nappies or cloth?
How often do you change it?
- BINDS: Who is taking care of him? Are you a happy family? Any FHx of eczema, asthma, hay fever?
- Any change in the color of urine? Do you think the urine is smelly?

Physical Examination
- General appearance
- Vital signs and growth chart
- Morphology of rash (red erythematous rash on the gluteal and perianal region)
- Lymph nodes
- Chest and heart/abdomen

Diagnosis and Management


- He has a nappy rash. This typically occurs in the area of the nappy. It is a very common condition in babies. The most common
reason is contact dermatitis which results from overhydration.
- General advice:
o Use highly absorbent disposable nappies
o Often change cloth nappies every 2 hours

o Do not use baby wipes or soaps (cetaphil) if with active rash


o Let region be exposed to air for some time
o Wash area gently with water. No soaps.
- There can be a lot of reason for the rash. It could be allergic (atopic dermatitis), or bacterial infection, but most likely it is due to
the contact with the enzymes in the baby’s feces. All nappy rash has a high chance of getting fungal infection. You can use
antifungal creams to apply on the area and apply barrier creams (Zinc) after every nappy change.
- TG: Hydrocortisone for sensitive areas (face, perineum) in combination with antifungal.
- I would like to review him in a few days. Reading material. Red flags: irritable, rash getting more florid, fever, signs of
superinfection

ENT HEAD AND NECK

Chronic Otitis Media

Case: Your next patient in GP practice is a 5-year-old child brought in by mom. She has recurrent otitis media and was treated with
amoxicillin 3x in 6 weeks. He has snoring and mouth breathing. On examination, there is bulging TM, red and child is febrile. Mother is
concerned and wanted to know alternative treatment.

Task
a. Discuss about alternative treatments

Risk factors
- Child care
- Smoking
- Recurrent URTI

Complications
- Hearing concerns
- Speech problems

Counseling
- Your daughter has another recurrence of otitis media which is an infection of the middle ear. She currently has a temperature
and the ear examination shows infection. Your daughter will need antibiotics this time as well but for a longer duration that
before. We need to treat the infection because if left untreated, it can lead to some complications: infection of the nearby
structures, brain, hearing and speech problems. Cefaclor or amoxicillin for 6 weeks (4mos in JM pg. 532).
- Ear toilet: 20 ml of 5% povidone iodine for a few days.
- Why does it happen? There are a few reasons why they may have recurrent infection. One is child care. We don’t recommend
children with this infection to be in group childcare. Is there any possibility to keep her at home? The second risk factor is
smoking. Are any of you smokers? If you like, I can book another appointment for you and your partner for smoking cessation.
- We need to take a swab of the ear for culture before we give antibiotics. Chronic infections can lead to hearing problems so I
would like to refer your daughter for formal hearing tests and if there are speech/language concerns, we can also refer to speech
pathologist as well.
26

- Also, the child is snoring so I am concerned that her tonsils might be enlarged. They are lymphatic tissues around the oral cavity
that help fight against infections. I will refer her to an ENT specialist to have a look and also I would refer her to the pediatrician
as well.

- If infection does not resolve with antibiotics, there are some surgical options: they can put a tube (grommet) to make connection
between inner and outer ear for better drainage. If tonsils or adenoids are enlarged, they may go for tonsillectomy.
- Review. Reading materials. Red flags.

Critical errors:
- Not referring for hearing tests
- Failure to assess speech and language
- Failure to check tonsils and adenoids

Recurrent Otitis Media

Case: Michael, aged 5 years is brought by his mother Suzie to your GP clinic. Michael had discharge from his left ear. He had a history of
URTI last week and was crying at night a couple of days ago. This is Michael’s 3rd visit in the couple of months with similar complaints. He
had started school this year and had several colds and gastroenteritis since starting school. Suzie is worried of Michael’s recurrent
episodes of ear problems and seeks your help.

Task
a. Further focused history
b. Examination findings
c. Diagnosis and treatment advise

History:
- snoring, mouth-breathing, waking up at night and daytime sleepiness, triggers: smoking at home, pets, carpets, swimming,
anyone else sick at home or in school, fever, rash, hearing problems?
- Correlate with school

Physical examination
- General appearance; VS
- ENT: inspection, otoscopic (check for whitish discharge, yellowish, effusion, light reflex, bulging TM, perforation of TM) check
both ears! Throat

Diagnosis and Treatment:


- Recurrent Otitis Media: 3 episodes in 6 months or 4 episodes in 1 year
- Keep clean ear and dry (do not push anything in the ear; clean with cotton pad/tissue)
- Antibiotics: Amoxicillin/Cefaclor/Cefuroxime; Ciprofloxacin if TM is ruptured
- Pain relief
- Review in 2 days and again in 2 weeks;
- Indications for Ventilation tubes or Grommet (Refer to ENT)
o Recurrent otitis media
o Hearing problems
o Repeated ear infections with effusion for 4mos.
- Advice: No air travel and swimming at least 2 weeks; showering with ear plugs; avoid smoking, pets, and carpets as much as
possible;
- Advise mother to come back if she notices changes in hearing. If there are concerns, do hearing tests – audiometry

Tongue Tie

Case: You are an HMO in pediatrics and you are asked to see Kevin who was just born and has a tongue tie.

Task
a. Counsel mother about condition
b. Answer her question

- Congratulations. Is he your first baby? How are you feeling? How was delivery? The midwife told me that your baby has a
problem with your tongue. When did you notice it (I noticed it yesterday when he opened his mouth. What is it)? I understand that
you are worried about Kevin but let me reassure you that it is not a serious condition. It is known as tongue tie wherein the
tongue is attached to floor tightly by a small muscle called frenulum (short). The symptoms are: baby may not be able to protrude
the tongue fully, cannot touch the roof of the mouth with the tongue, and may not be able to move the tongue sideways. But don’t
worry, this usually does not affect the feeding, but may cause some problems with the speech.
- Why did it happen doctor? The cause is unknown. It is a congenital abnormality, but I would also like to examine Kevin for other
congenital abnormalities.
- What can be done about it? As it is not a progressive disease, we will wait and see. If everything goes well, we do not need to do
anything. If there is any concern or if there is a complication or problem with feeding or speech, then surgical intervention can be
done.
27

- How will the surgery be done? It is a very quick procedure. It may or may not need local anesthesia. Usually, for kids less than
12 weeks, we don’t do local anesthesia but for more than 12, local anesthesia. It will be cut by a scissor and usually there is no
complication.
- If you find any feeding difficulties or any speech problem, you can go to the GP who will refer you to a pediatric surgeon for
appropriate management. However, if you are concerned now, I can organize for the pediatric registrar to come and have a look.
- Can there be speech problems after surgery? Not usually. But still, we can refer you to a speech pathologist if that happens.

Herpes stomatitis

Case: 2-year-old son who has been refusing to eat all day. He is irritable, crying a lot and she noticed some ulcers in the mouth.

Task:
a. History
b. Physical examination
c. Diagnosis
d. Management

Differential Diagnosis:
- HFMD (fever, throat, ulcers in mouth, palm and soles of the foot and nappy areas)
- Herpes stomatitis: most common
- Herpangina
- Traumatic ulceration
- Drug-induced

History
- I know you son is not feeling well. When did it start? When did you notice the ulcers? Before you noticed them, did he have any
hot foods or drink? Is it the first episode? Anyone else in the family with same condition?
- Does he have fever? Up and down or constant? Did you give him any medications? Did he complain of headache? Have you
noticed drooling? Has he vomited? Have you noticed rashes on his palms, fingers, sole?
- I know he refused his food, but has he been drinking?
- How is peter’s waterworks? Has it reduced? How many nappies? Are they heavy? Is peter still active or does he appear drowsy?

- Is peter generally healthy? Is he on any medications? Allergies? Any concerns about peter’s growth and development? Is
vaccination up-to-date?
- Do you have other children? Is he attending childcare? Has anyone been sick there?

Physical examination:
- General appearance: active/lethargic, irritable, signs of dehydration (CRT, skin, turgor, sunken eyes, mucous membranes)
- Skin: check for rash while undressed
- ENT: conjunctivitis; mouth and throat; LAD; neck stiffness

Management:
- Helen according to your son’s Hx and PE, your son has a condition called herpes stomatitis. Have you ever heard about it? It’s a
common condition caused by HSV. 90% of the general population has been exposed to this virus. In childhood, the primary
attack can present with fever, mouth ulcers and enlarged LN. You don’t need to give him antibiotics because it is a viral infection.
Like majority of viral condition, it is self-limited.
- Healing occurs in 2 weeks. But during this time, it may be difficult to eat and drink. I will try to apply lignocaine gel and see if he
can drink. If it works, you will need to continue applying the gel at home every 3 hours and make sure he drinks ample fluid. I
would also recommend liquid diet of cool to cold and no acidic drinks.
- To control fever and pain, you can also give him panadol. If he can’t drink, I will need to admit him to the hospital for IV
rehydration
- If you go home, you need to keep an eye on symptoms. If peter is less active, cannot drink, is vomiting, or stops urinating, you
need to go to the hospital.
- It is a contagious disease. He should be isolated from other children (should not go to childcare until after ulcers are healed). He
should have his own utensils, glasses, plates and toys.
- Severe primary attack or immunocomprised acyclovir
- Often virus will become silent and can be reactivated later in his life. Usually it happens when the immune system is down due to
other illness or stress. The commonest presentation of recurrent herpes simplex attack is cold sores on the lips, although it is
rare.

Laryngomalacia

Case: Your next patient in GP practice is a 3-month-old boy brought in because of noisy breathing.

Task
a. History (2-3 weeks, sleeping and crying,musical, bottle-fed; FHx of SIDS )
b. Physical examination (HR 120, RR 35, BP and growth chart normal,
c. Diagnosis and management

Differential Diagnosis
- Croup
- Laryngeal web
- Foreign body or external compression from outside
- Vascular rings
28

Clinical features
- Most common cause of stridor in infants

- Usually presents as inspiratory stridor within the first 6 months of life


- 90% of patients will have spontaneous resolution of symptoms usually by 12 months of age (18months – 2 years)
- Stridor is intermittent and variable in intensity
- Symptoms are worse during sleep and positional variations occur (worse when supine, improved when patient is prone)
- delay in maturation of laryngeal cartilage
- “floppy larynx”
- Diagnosis
o Confirmed upon flexible laryngoscopy with findings of supraglottic tissue into the laryngeal inlet during inspiration
o Laryngotracheobronchoscopy to rule out any synchronous airway pathology
o Polysomnography to detect episodes of hypoxia or hypercapnia
- Management
o Most do not require intervention due to a high rate of spontaneous improvement
o 10-20% of synchronous airway lesions (patients with more severe symptoms should undergo direct laryngoscopy &
bronchoscopy to assess the entire tracheobronchial tree)
o Gastro-oesophageal reflux can exacerbate laryngomalacia & should be treated medically
o Surgical intervention is indicated for approx. 10% of patients – indications are severe stridor, apnea, failure to thrive,
pulmonary hypertension, and cor pulmonale
- 99% recover completely by age 2
- Management: reassure

History
- What exactly do you mean by noisy breathing? When did it start? Is it continuous? When does it happen? Any kind of cough?
Does he go blue? Does he have rapid breathing or stops breathing at any time? Do you think his breathing is labored? Any
vomiting?
- BINDS? Does anybody smoke at home? Any family history of similar condition? Do you think he might have swallowed
something

Physical examination:
- General appearance and rash
- Vital signs
- Lumps and bumps
- ENT
- Chest for signs of respiratory distress, abnormal asymmetrical breathing, adventitious sounds on auscultation (mild stridor when
baby cries)

Diagnosis and management


- Laryngomalacia is the weakness of the larynx. The wall of the larynx is immature, weak and floppy. It is a congenital condition of
unknown cause. In mild cases it is not risky but can have some complications: children can aspirate easily and can have
repetitive chest infections.
- We will wait and see since it is mild. Usually, kids improve by 6 months of age. If it doesn’t get better by 12 months, we need to
investigate further and the baby might need to undergo surgery.
- Does he need tracheostomy: NO.
- Review. Reading material.
- Red flags: respiratory distress, failure to thrive, recurrent chest infections

Acute Tonsillitis

Case: A 4-year-old boy was brought by his mother to your GP clinic because she noticed that his child is not well, less and has whitish
material in his throat.

Task
a. History (fever 38)
b. Physical examination (alert, flushed, normal growth chart, ENT congested tympanic membrane, throat as in page 450, urine
dipstick normal)
c. Diagnosis and management

Differential Diagnosis
- Tonsillitis
- Quinsy
- Scarlet fever
- EBV
- Meningitis

History
29

- I understand you have come because your son is feeling unwell? When did it start? Did he complain of a sore throat? Has he
had difficulty swallowing? Does he complain of earache? Does he have a runny nose or watery eyes? Does he have cough or
difficulty breathing? Does he have fever? Is it up and down or constantly high? Does he have chills? Did you notice any rash
anywhere in his body? How is his appetite? Is he still drinking well? Has he vomited? Any abdominal pain? How’s his
waterworks? Any diarrhea? Is he active or appear sleepy? Is he generally healthy? Medication? Allergy? Concerns about his
growth and development? Immunization? Do you have other children? Does he go to childcare? Has anyone been sick at home
or at childcare?

Physical examination
- General appearance
- Vital signs and growth chart
- ENT: conjunctivitis, tympanic membrane, swab
- Neck stiffness and enlarged tender LAD (submandibular and anterior cervical)

Diagnosis and Management


- According to your son’s history and physical examination, he has a condition called tonsillitis. Have you ever heard about it? The
tonsils are two small buds of tissue located at the back of the mouth. They are part of the immune system and are the first line of
defense against bacteria in food or air. Tonsillitis occurs when the tonsils become infected and can be caused by either bacteria
(20-30%) or viruses.
- In your son’s case, I suspect bacterial tonsillitis because he’s markedly unwell, and has tender, enlarged tonsillar lymph glands.
That is why I took a throat swab for culture. The most common bacteria causing tonsillitis is streptococcus (GABHS).
- I will prescribe antibiotic (phenoxymethylpenicillin) for 10 days (or erythromycin if allergic). You can also give him paracetamol to
relieve pain and fever. Most children with tonsillitis don’t feel well and it hurts them to swallow. Try cool drink, ice block, and ice
cream. Don’t worry if he stops eating for a day or two. He would pick up quickly after infection.
- If he is more lethargic, can’t drink, doesn’t wee or starts vomiting, you need to go to the hospital. Otherwise, I will see you in 2-3
days to check up on improvement. You need to continue antibiotic for a

total of 10 days to prevent possible complications: can spread to sinus, nose or ears, quinsy (peritonsillar abscess), PSGN and
rheumatic fever.
- If not improving after 2-3 days consider EBV serology

Peritonsillar Abscess (Quinsy)

Case: Your next patient in GP practice is a 14 year-old girl complaining of sore throat.

Task
a. History (started 3 days ago and progressively worse, + fever around 39C and chills, no runny nose, + dysphagia and
odynophagia, +nausea, no vomiting, menarche 12, regular, not sexually active, no medications apart from vitamins, no allergies,
+ previous Hx of tonsillitis)
b. Physical examination (mildly dehydrated, pale and flushed, growth chart normal, T40, RR 24, HR 105, BP 90/75 with postural
drop no ulcers, unilateral tonsillar swelling with marked swelling of peritonsillar area, tender cervical LN)
c. Management

Features:
- Quinsy – peritonsillar abscess marked swelling of peritonsillar area with medial displacement of tonsillar tissue
- Cause: GABHS or anaerobes; occasionally H. influenza and S. aureus
- Increasing difficulty in swallowing and trismus
- Treatment: antibiotics (penicillin IM or clindamycin) + drainage under local anesthetic if it is pointing

History
- I understand you have come because you have a sore throat, when did it start? Can you describe the pain? Does the pain travel
anywhere else? How severe is the pain on a scale 1-10? Does anything make it better or worse? Is it the first time? Do you have
any fever? Is it up and down or constantly high? Did you measure your temperature? Do you have uncontrollable shaking or
shivers? Do you have a headache? How is your appetite? Do you have difficulty swallowing? Are you still drinking a lot? Do you
have runny nose or cough? (In younger child: Do you have drooling of saliva?) Do you have difficulty opening of the mouth? Do
you have difficulty breathing? Have you noticed a change in your voice (“hot potato voice”)? Do you have any neck pain? How
are your waterworks?
- Are you generally healthy? Have you ever had tonsillitis in the past? Medication? Allergy?
- Who do you live with? Does anyone in school have any infection?

Physical Examination
- General appearance
- Vital signs
- Rash
- ENT: appearance of tonsils; is uvula displaced from midline
- Lymph nodes

Diagnosis and Management


30

- According to your history and physical examination, you have a condition called quinsy or peritonsillar abscess. Have you ever
heard about it? Quinsy is a collection of pus beside the tonsil. It is a well-

recognized complication of tonsillitis and caused by bacterial infection. It is a surgical emergency. For this reason, I need to admit
you to the hospital immediately. Meanwhile, I will give you a painkiller (spray). I will put an IV line and give you fluids to rehydrate
you. At the hospital, you will be assessed by an ENT specialist who will do incision and drainage of abscess under anesthesia.
Prior the procedure, the nurse will collect blood samples for FBE, U&E, ESR/CRP, and blood cultures. You need to receive IV
antibiotic usually with benzylpenicillin and metronidazole.
- At the hospital, they will also monitor your inflammatory markers and kidney function to monitor your progress
- If with breathing problems give IV corticosteroids!
- Recurrent quinsy or history of tonsillitis complicated with quinsy tonsillectomy

Posterior Triangle Lump (Postviral [Reactive] Lymphadenitis)

Case: A mother of a 3-years-old boy is in your GP clinic as he found a lump on his neck. She is worried about it because her nephew was
recently diagnosed with lymphoma.

Task:
a. History
b. Physical examination (well looking child, posterior triangle of neck 1x1.5cm, soft, mobile, non-tender, movable; mild redness of
the throat)
c. Diagnosis and management

Neck lumps in children


- 80% benign and 20% malignant
- If <2cm in diameter + hard + soft + mobile postviral
- If >2.5 cm in diameter + fixed + hard/firm suspicious may do biopsy and further examination
- Posterior triangle: bronchial sinuses/cyst, cystic hygroma, tuberculosis, lymphoma,

History
- Where exactly is the lump? When did you notice it? Is it painful? Did he have a recent URTI or other infection? Did you notice
any change in size or shape of the lump? Change in color of overlying skin? Is it hard or soft? Is it fixed or mobile? Any other
lumps and bumps in the body?
- Any previous history of fever, night sweats? Is he pale or lethargic? Any symptoms of tiredness, recurrent infections, bleeding,
bruising? Any change in his appetite? Did you notice any mass in his tummy? Waterworks and bowel motions? Is he in
childcare/schooling? Contact history? Anyone else at home having a similar problem? Any recent travel history interstate or
outside Australia?
- BINDS
- FHx: other history of lymphoma or cancers in the family?

Physical Examination
- General appearance: pallor, jaundice, dehydration
- Vital signs and growth chart
- Skin: bruising, pallor or signs of infection
- ENT with LAD
- Chest and heart
- Abdomen

Investigations: FBE, ESR/CRP

Diagnosis and Management


- Most likely it is postviral lymphadenitis and it is a common condition after a viral infection. It is a benign condition and not a
cancer. It is self-limiting and no treatment is needed.
- Antibiotics? No.
- If concerned may do investigations but if persistent, refer to pediatrician.
- Review after 1 week to see if the lump is decreasing.
- Red flags: lump is increasing in size, becoming hard/fixed/immobile/painful and if he is getting pale or lethargic, weight loss etc.

OPHTHALMOLOGY

Strabismus/Squint

Case: Little Johnny is brought by her mother Suzy to your GP clinic. Johnny is referred by maternal and child health care nurse with
concerns about misalignment of his eye. Johnny is aged five months and is first child to suzy who is 35 years of age. Johnny was born
term by NSVD. Suzy had mild vaginal bleeding in first trimester but no further problems in pregnancy. Suzy and husband Michael had no
31

concerns about Johnny eyes alignment or visual function. Jonny always responds to his mom’s presence and he seems to see well. There
is no other concern by Suzy.

Task
a. Further focused history
b. Examination findings from examiner
c. Probable diagnosis and management advise

Differential Diagnosis
- Strabismus
- Retinoblastoma
- 6th nerve palsy

History
- Since when? Is it in one or both eyes? Is it constant or does it come and go?
- BINDS? Any trauma? Was he term or preterm?

Physical examination
- General appearance
- Vital signs and growth chart
- Eye: red reflex

Diagnosis and Management


- What your son has is a condition we call strabismus (squint, cross-eye, wondering eyes). This is a condition where there’s a lack
of coordination between the muscles controlling the eye movements. Sometimes the eyes are mal-aligned especially while
focusing on an object. If the eye is turned towards the nose we call it esotropia, if away from the nose we call it exotropia. As you
know we have 6 muscles on each side of the eye that work together to help the eye to focus on a particular object. Sometimes
due to certain causes eg cranial nerve paralysis, lesion in the brain, increase fluid around the brain (hydrocephalus), in
premature babies, in babies with Down’s syndrome these muscles do not function properly. Rarely certain tumors of the eye can
cause this condition. Sometimes there’s a co-existing family history of squint. In adults squinting is usually associated with
cataracts, DM, and far sightedness, as well as trauma to the brain. If squinting is presenting in kids, 50% usually recognisable
during infancy. The male to

female ratio is equal and there’s a very high risk that the child might develop a condition called amblyopia where the brain
suppresses the image transmitted by the affected eye over a period of time the visual ability from that eye is lost. There’s also a
risk of problems with the perception of depth. Sometimes the peripheral vision is affected even without Amblyopia.
- However in adults the usual symptoms is double vision/diplopia. There are many types of squint eg unilateral vs bilateral,
congenital vs acquired, constant vs intermittent, Esotropia vs exotropia. Esotropia is the most common type of squint usually
seen in baby <1yo. for which there’s usually no cause can be found. Unfortunately this type of squint worsens with age with or
without glasses. Pseudo-strabismus: usually there’s a flat wide nasal bridge, there’s a thick fold of skin on lateral side of eye that
gives squint. This type is commonly seen in chubby babies disappears by 6-9months old.
- Can squint get better by itself? There are chances that certain type of squint, especially the intermittent squint and the pseudo-
squint to disappear before the age of 5. However, a true squint or esotropia rarely corrects itself and needs to be treated as soon
as possible. Ill refer you to the ophthalmologist they need to check your visual acuity, visual field, light reflex and any coexisting
disease of the eye.
- There are many options regarding management
o Eye-patch to be placed over the normal eye. This will force the squinted eye to focus properly
o Special glasses or lenses
o Eye drops or injections to control the activity of the overactive eye muscle. Eg. Inject the inner side of the eye to the
medial rectus, it affect the transmission of acetylcholine (botux)
o Glasses with prism lenses: prevent double vision and to provide comfort to the eye by adjusting the level of light
o Ophthalmologist will teach your child certain exercise to strengthen his visual skills as well as to make the eye muscles
stronger for better alignment of the eyes.
o Surgery where the muscle’s length or position is altered. Reposition/Resection. The surgery produces the best result
especially done within the first 2 years of life. However there are certain complication: infection, bleeding, scaring,
retinal detachment.
- Once your child has been seen by ophthalmologist ill refer you to specialist eye surgeon who would decide which type of
treatment is best for your baby.
- The output is not good. Even with surgery 100% correction is almost always impossible. (Up to you to tell the mom).

Orbital Cellulitis

Case: Your next patient in GP practice is a 13-year-old Ryan brought in by mother because of swelling on the right eye.

Task
a. History (started since yesterday, no trauma, no vision problem, painful, red and hot, no fever, first time, discharge, cough and
colds 7 days, no discharge or throat)
b. Physical examination (distressed, in pain, orbit is swollen and red, ophthalmoplegia and restricted eye movements)
c. Diagnosis and management
32

Differential Diagnosis
- Trauma
- Orbital/Periorbital cellulitis
- Severe conjunctivitis
- Insect bite/allergy

PERIORBITAL ORBITAL
Cause Purulent Purulent
conjunctivitis conjunctivitis
Trauma Trauma
Insect bites Insect bites
URTI URTI
Clinical features Fever, tender, Same + proptosis,
swollen soft tissue, ophthalmoplegia,
difficulty in eye chemosis,
opening decreased VA
Diagnosis Clinical CT – r/o abscess,
sinusitis and
intracranial
extension
Management IV flucloxacillin/ 3rd IV flucloxacillin until
generation symptoms resolve
cephalosporins then completely
oral augmentin x5-7
days if symptoms
controlled
If <5 years or if
severe: treat as
orbital cellulitis

Complications
- Meningitis
- Orbital or subperiosteal abscess
- Cavernous sinus thrombosis
- Optic neuropathy
- Sinusitis

History
- Since when did it start? Was it gradual or sudden? Is there any pain? Any sticky eye or discharge? Any visual disturbances?
Redness? Do you feel pain when you move your eyes? Any trauma? Do you think your eye is bulging out? Any drooping of
eyelids? Do you have excessive tearing? Any chance of insect bite? Do you find it difficult to open the eye? Do you have fever?
- Is there any swelling on any other part of the body? Any joint pain? Any previous history of URTI recently? Any headache or
vomiting?
- BINDS
- FHx of any eye disorders?

Physical examination
- General appearance and growth chart; rash
- Vital signs
- Lymph nodes and neck rigidity
- Eye: proptosis, chemosis, signs of trauma such as bruising, insect bite, swelling involving what, redness of conjunctiva, corneal
problems, visual acuity, PEARL, extraocular movement, temperature, visual fields, funduscopy

Diagnosis and Management


- Your son has a condition called orbital cellulitis. Have you heard about it? It is an emergency situation. Basically it is the infection
and inflammation of the soft tissue around and behind the eye. There could be a lot of reason for it: URTI, trauma, or insect bite.
- At this stage, I would like to call the ambulance and admit your son because this condition carries the risk

of blindness and can become nasty if it spreads to the brain leading to meningitis. In the hospital, they will do an urgent CT scan
and start you on antibiotics IV flucloxacillin + Ceftriaxone. You will be reviewed by a pediatrician, eye and ENT specialist. They
will give you some painkillers.
- But don’t worry, you are in safe hands.
- Prognosis? The prognosis is good if treatment is started.
- How long will I be admitted? You will need to stay for about 7-10 days until all your symptoms resolve completely.

ENDOCRINOLOGY

Congenital Adrenal Hyperplasia

Case: You are a GP and Nadia, 5-days-old was brought by her mother Mrs. Smith due to vomiting.
33

Task
a. History (born at home, vomiting x 10eps)
b. Physical examination (unwell, lethargic, dehydrated with sunken fontanelle, T36, HR 162,RR 38, O2 saturation 93%, BP 60/35,
clitoromegaly and labioscrotal fusion)
c. Differential Diagnosis
d. Investigation
e. Diagnosis and Management

Differential Diagnosis
- GOR
- Cow Milk Protein Intolerance (vomiting/constipation/diarrhea/rash) ask about eating habits of mom – try to remove dairy; if
bottlefed stop and shift to soya milk pepti-junior or neocate
- UTI /Sepsis
- Pyloric stenosis (hypokalemic, hypochloremic metabolic alkalosis)
- Overfeeding
- TEF
- Duodenal atresia
- AGE
- CAH (hyperkalemia and hyponatremia)

History
- Vomiting: when? Number? Color (green)? Projectile? Related to the feeding? How long after the feeding? Fever? Rash? Flu-like
symptoms? Crying always? Stool (diarrhea/ Watery/foul-smelling/mucus, green/frequency)? Urine
(smelly/color/amount/nappies)? Lumps in the abdomen? Mass in hernia orifice? Jaundice or change of color of skin
- Breastfeeding (how often; feeding well);
- Birth: Preterm, FT, PROM, infection in pregnancy, birth where?
- FHx:

Physical Examination
- General appearance: hyperpigmentation of skin folds;
- Vital signs
- Chest, Heart and Abdomen
- Genital examination is needed for baby check!!!!

Investigation:
- FBE, MSU, Urine
- Serum electrolytes (hyponatremia, hyperkalemia)

Diagnosis and Management


- Your daughter has CAH wherein there is a disturbance of enzymes and hormones that are produced by a gland on top of the
kidney called the adrenals. It is not common and as a result of the enzyme abnormalities, this results to disturbance on the
female sex hormones.
- I would like to refer her to the emergency department and resuscitate with her fluids and IV corticosteroids. She needs to be
reviewed by a pediatric endocrinologist and seen by a plastic surgeon to correct her genitalia.
- In the future, she may need to be screened again in case she wants to get pregnant because of the risk of developing CAH in
her baby.
- She will need life-long corticosteroid treatment and might suffer from the side effects of steroids

Diabetes Education

Case: You are an HMO and a 9-year-old boy was admitted to the pediatric unit because of first presentation of diabetes type I. He is in
good condition now.

Task
a. Counsel the mother regarding diabetes type I
b. Answer mom’s questions

- I completely understand that it might be a stressing situation for you but let me assure you that we can manage it and we aim to
give John a normal lifestyle.
- Diabetes is a common metabolic condition that results from the lack of a hormone called insulin produced by the pancreas. This
hormone is important in regulating the BSL, and when it is absent, the blood sugar level goes up. Because of that, a lot of
complications can happen. For this reason, we need to replace this hormone which unfortunately, is lifelong.
- How do we give insulin? Insulin is usually given by injection under the skin. There are different regimens. At this age, we start
with twice daily which is a combination of short-acting and intermediate-acting. We give 1 injection 30 mins. before breakfast and
30 minutes in the evening. The other regimen is 4xa day which we will do when he is >12 years old. (3-SA before meals and 1 IA
at night). Preferred sight of injection is thigh and abdomen and it is used in a rotating manner. You will be educated by the
pediatric unit staff or diabetic educators and they will make sure that you will be confident in giving injections.
- Monitoring. You have to monitor the BSL by glucometer initially before each meal and bedtime (4x) and later on, twice a day.
Ideal levels are 4-6mmol/L before meals and 6-8 mmol/L after meals. It is advisable to maintain a diary (Blood sugar – insulin –
meal).
34

- Cautions:
o Do not miss the meal after insulin
o Do not miss insulin is child is sick
- Dietary modifications: nutritious diet. Child must have 3 meals and 2 snacks. Avoid junk or fatty food. Try to avoid sweets.
- Diabetes Action Plan: red flags for signs of hypoglycemia (tremors, sweating, palpitations, more hungry than usual, drowsy or
loss of consciousness,) measure BSL and give 6 jellybeans, one teaspoon honey, glass of lemonade, 2 barley sugar
repeat if symptoms don’t improve give snack call ambulance
- I will send a letter to school about his condition and diabetes educator will visit the school.

- Can he go to school camps? We aim to give him a normal life. If you can volunteer in the camp that would be better. Otherwise,
there are trained teachers at the camp or special staff to administer injections. If going far for camping, we give a letter to the
local GP.
- Can he go to sleepovers? Yes. That should be encouraged. It would be better that you are close with the family and the place is
near.
- Sports? Yes. There are no restrictions but the dose will be adjusted accordingly. He cannot pilot a plane.
- Do not change insulin dose!
- Support Groups and dietitian.
- When patients are sick: try to check urine

Tiredness in a Young Patient (Diabetic Ketoacidosis)

Case: You are an HMO in a regional hospital and a young male comes in complaining of tiredness.

Case: 18-year-old male working in farm complaining of tiredness and unable to help his parents.

Task
a. History (tiredness x 2 weeks, increased sleep, increased breathing)
b. Physical examination findings (looks ill, dehydrated, breathing a bit fast, no LAD, postural drop, PR: 110, RR: 30,
c. Investigations
d. Management with examiner

History
- When exactly did you start feeling tired? Did it come gradually or after a recent infection? Do you have any other symptoms like
N/V/change in bowel habits/tummy pain? Any fever/cough/breathing problems? Do you feel palpitations/dizziness/aches and
pains/joint pains? Have you noticed any skin changes? Have you noticed weight loss recently? Any lumps or bumps around the
body? Do you think you are more thirsty or passing more water these days? Do you feel hungry all the time? Have you noticed
any infections of the skin especially fungal infections?
- Are you sexually active? Are you in a stable relationship? Have you noticed a discharge or rash or itching within the genital area?
How is your general health? Any medical or surgical condition that I should be aware? SADMA?
- FHx: DM (at what age? Does he use insulin), Thyroid problems, Cancers,
- How is your mood? Do you work or study?

Physical examination
- General appearance: pallor, dehydration, jaundice, observe respiratory pattern (acidotic/kussmaul: rapid shallow breathing)
- Vital signs: PR, BP (postural drop), RR, Temperature, O2
- ENT and LN
- Chest/heart/abdomen
- Urine dipstick/BSL

Diagnosis
- Blood Sugar Level 11.0 blood ketones Bedside Instrument (OPTIUM) >0.6 (acidosis) ABG
- Diagnosis: pH <7.3 OR HCO3- <15.0mmol PLUS KETONES in blood or urine

Assessment
- Degree of Dehydration:
o Mild (<4%): asymptomatic
o Moderate (4-7%): symptomatic
o Severe (>7%): shock
- Level of Consciousness
- Send for investigations: FBE, UEC, BUN, BSL, Infection screening (Urine MSU, Blood Culture, CXR), Insulin Antibody testing,
celiac screening (Antigliandin Ab, IgA, Tissue Transglutaminase Ab), GAD,TFTs, HbA1c

Differential Diagnosis
- Post-infectious Fatigue Syndrome (mononucleosis, influenza)
- Anemia
- Endocrine: Hypothyroidism, Adrenal, Diabetes Mellitus
35

- Depression
- Drugs
- Chronic fatigue syndrome

Management
- Urgent admission
- NPO, weigh patient
- IV fluids:
o NS 10-20ml/kg IV bolus (CRT <2secs)
o 15-30 ml/kg/hour for next 2 hours
o 7.5 ml/kg/hour for the next 6 hours
- Insulin
o 50U clear rapid acting insulin + 50ml NS
o 0.1 u/kg/hour
o Switch to subcut
- Potassium
o <3mmol/L 40mmol/L
o 3-4 mmol/L 30 mmol/L
o >4 mmol/L 10 mmol/L
o If anuric or K+ >5.5: do not replace potassium
- Bicarbonate only if pH <7.0 (0.15 x weight x base deficit)
- Insert foley catheter and monitor urine output
- Monitor GCS, BSL, Ketones, ABGs, UEC q 2 hours
- Add glucose (D5W) once BSL <12mmol/L @100ml/hr.
- ICU admission: age <2 years, coma, cardiovascular compromise, seizures, severe acidosis <7 or HCO3- <5mmol/L.
GASTROINTESTINAL

Gastroesophageal Reflux of Newborn

Case: You are a GP and a mother brought his 4-weeks-old baby boy. She says the child vomits after feeding. He has put on 1 kg since
birth. There are no other symptoms. Mom thinks that bottlefeeding will be better for him.

Task
a. History
b. No further exam required
c. Diagnosis
d. Management

Differential Diagnosis
- Pyloric stenosis: 1st born male babies, test feeding (lump); projectile vomiting; baby hungry; irritable; crying; decreased BW; 4-6
weeks (not immediate as muscles; take time to develop)
- UTI: (fever, <4 weeks, septic look, dipstick)
- Reflux: vomitus remains the same; happy, gains weight; milk only without changes in consistency)

- Overfeeding
- Duodenal atresia
- AGE
- CAH

History
- Please tell me more about the vomiting. How frequent is it? Does it happen after every feed? What is the color?
- Milk: is it digested at all? Have you noticed that the vomiting is forceful? Is he hungry after vomiting? Once you finish feeding, do
you notice a lump in his tummy? When does he feel hungry again? Does he have any associated fever, irritability, lethargy?
- How are his wet nappies? Any changes? Is he passing stool normally?
- Do you think he is a happy child? Is he able to sleep well at night? Any complication with your pregnancy/delivery? Do you think
he is growing well? Is this the 1st baby in the family?
- FHx: bowel related problems? Have you tried bottlefeeding up to now?

Diagnosis
- Your child most likely has GER. The lower part of the food pipe where it connects with the stomach is poorly developed in
neonates/babies. It is the reason why the milk in the stomach flows back into his mouth when you put him down. It is a very
common condition where the baby vomits after each feed but otherwise the baby thrives well. This condition gradually improves
with time especially after the introduction of solids (4-6mos of age). Majority of cases clear up by 1 year of age.
- At the moment, we will do an ultrasound of the tummy to rule out any serious condition such as pyloric stenosis. Most likely the
ultrasound will be normal. I will recommend to give him frequent, smaller feeds, decrease duration of feeding along with time in
between (normal 20 mins on each breast). You can thicken the feed with thickeners called gaviscon. The way to prepare it is to
take 1-1/2 tsp of gaviscon, add a little amount of boiled cold water and little of expressed breast milk (20ml). Mix them in a cup
and feed the child with a spoon immediately before feeding time. This helps the breast milk stay in his stomach. Please elevate
head of cot by 10-20 degrees. After each feed, don’t put the baby immediately to sleep. Carry him for a while and put him in cot
with head elevated.
- We highly recommend breastmilk because it is best for baby.
36

- Reassure.
- If persistent:
o Ranitidine (Zantac) 1ml/day or omeprazole (Losec): <10kg 5mg/day)
o If not improving: consider GORD refer to gastroenterologist do manometry (measure pH and peristaltic
movement of the esophagus)
- Signs of reflux: arching back + flattening of back + screaming + irritable + drawing of legs
- Complications: aspiration, ulcers, inflammation, dehydration,
- Improve at 6-9 mos: maturation of LES, introduction of solids, change in position

Weight:
- 1st 4 months: 750 g/week
- Next 4 months: 500g/week
- 1 year: 250 g/week

Recurrent abdominal pain

Case: Your next patient is 10-year-old Brian who is suffering from abdominal pain for last 2 years. The growth charts are normal. Palpation
of abdomen: mild tenderness.

Task
a. Take history from anxious mother
b. Talk about further management plan

Case 2: Ronnie aged 6 years is brought by her mother Julie to your GP clinic. She tells you that Ronnie had abdominal pain for the last few
months and she is quite concerned as her usual GP think that it’s not serious and seeks your opinion. On further questioning, Julie
describes the pain as intermittent and mainly around the umbilicus sometimes severe enough that Ronnie had to miss school. Ronnie is
otherwise well and had no significant medical or surgical problems. Ronnie lives with his parents at home and had started school this year.

Task
a. Further history
b. Physical examination
c. Differential diagnosis and management

Features
- 3 distinct episodes of abdominal pain over >3 months occur in 10% of school-aged children
- 5-10% organic cause can be found
o Pain is other than periumbilical
o Pain radiates rather than remains localized
o Pain wakes child from sleep
o Accompanied by N/V
o Not well between attacks
o Associated weight loss
o FTT
- Possible causes:
o Constipation
o Abdominal migraine
o Lactose intolerance
o Non-ulcer dyspepsia
- Investigations: urine MCS, FBE and ESR, Xray
- Non-organic
o Acute and frequent colicky abdominal pain
o Pain localized or just above umbilicus
o No radiation of pain
o Pain lasts <60 minutes
o Nausea frequent; vomiting rare
o Diurnal
o Minimal umbilical tenderness
o Anxious child
o Obsessive or perfectionist personality
- Management
o Give explanation, reassurance and support
o Avoid investigations
o Acknowledge that the child has pain
o Emphasize that disorder is COMMON and usually traverses childhood without ill effects
o Simple measures: local warmth, brief rest
o Review: if episodes change in nature, pain persists for hours, or there are new symptoms
o Identify any life stresses and enquiry about family structures and school performance
o Discourage identification with sick role
o Refer for psychological assessment and counseling if necessary
37

Differential Diagnosis
- Migraine
- Travel History: Giardiasis
- Lactose Intolerance

History
- Pain: Where: paraumbilical? Since when? How many episodes (3 episodes in 3 months)? How long do they last (<60mins)? In
between the episodes the child is fine.Severity: Acute/severe. Character: (Colicky pain). Radiation: It doesn’t radiation/ Anything
increasing the pain (No)? Is the child constipated? Emotional factors: Family situation, personality of the child. Anything
decreasing the pain (pain killers)?
- Did he use any medications? Does he wake up at night (No)? Does the child have any fever? Infection? Intussusception?
Volvulus? GORD? Gastroenteritis? Any nausea and vomiting? Any bloating? Any heartburn? Regurgitation(if younger age)?
Any flatulence/distention (lactose intolerance)? Did you notice any mass in the tummy? Bowel motion: Is he toilet trained? Any
previous history of constipation? Did you notice any blood? Offensive? Hard to flush? Urine: Any urinary infection? Any dysuria?
Any blood in the urine? Smelly urine?
- BINDS? Any medical condition? Surgery? Is the child pale/ pallor? Is the child complaining of headaches? Family history of
migraine? Growth Chart: normal. Nutrition: Does the child drinking milk products? Is it associated with milk products? Family
history? Happy family? Any financial issue? Any relationship problems? How do you cope? School performance? Any bullying?
Does this pain happen when the child is going to school? Does it happen in the weekend also? Can you describe his
personality? Perfectionist? Anxious child? Intestinal worm: pain when sleeping? Any recent travel anywhere?

Physical Examination
- General Appearance: well/ playful, no pale, no dehydration, no jaundiced, no LN enlargement?
- Growth Chart: Normal. No FTT
- Abdominal: Any mass? Any scar marks? Palpation: is the abdomen soft? Yes, but mild tenderness. No organomegaly? Fecal
impaction? Hernia orifices? Bowel sound? Normal.
- Inspection: Any excoriation, skin tags, fissure, mucous discharge, soiling

Investigations:
- FBE, ESR, Urine analysis, urine mic/cul, Plain X-ray if required (+ travel history or if suspecting constipation).

Diagnosis and Management


- Most likely its non-organic recurrent abdominal pain, it’s a common disorder it doesn’t have any ill effect on the child’s health.
Reassurance: 8% of school aged children can have this problem. If there are any factors: constipation/emotional factors need to
be addressed. During the painful period: take rest, local warm (heat pack). Rest during painful periods. Make a diary. Involve the
child in discussion. Refer to the pediatric psychologist for the insight therapy. Arrange family meeting to address family issue.
Contact teachers to find out causes in the school.
- Red Flags: If there’s any change in nature of the pain. Any new symptoms, N/V diarrhea, child turn pale, FTT, if the pain wakes
up the child at night, or poor school performance.

Pyloric Stenosis

Case: You are an HMO in ED and your next patient is a 5-week-old boy brought by mom because of vomiting

Task
a. History
b. Examination
c. Diagnosis and management

Features:
- Risk factors
o Male, firstborn child
o Caucasian
o Parental history of HPS
- History
o Vomiting (progressively more forceful, non-bilious, blood stained in up to 10% of cases)
o Usually don’t have diarrhea
o Often hungry afterwards
o Weight loss or inadequate weight gain
- Examination:
o Assess degree of dehydration
o Growth chart
o Jaundice
o Gastric peristalsis (left to right)
o Pyloric mass (olive in LUQ to RLQ)
38

- Investigations: Alkalosis-hypochloremic/hypokalemic
o FBE, U&E, Acid Base (hypochloremic metabolic acidosis), Glucose
o USD: if not established
- Management
o Fluid resuscitation (10-20ml/kg NSS)
o Commence IV fluids (0.45% NS + 5% or 10% dextrose + 10mm KCl) at 100 ml/kg/daily
o Stop oral feeds
o Insert NGT
o Repeat U&E, acid base q4-6
o KCl for significant hypokalemia
- Notes:
o Correction is not an emergency undertaken only until normal electrolytes and pH and infant is rehydrated
(Ramstedt’s pyloromyotomy)
o Feeding started 6 hours after
o Re-stenosis: uncommon 1-2%

History
- Is my patient hemodynamically stable?
- When did it start? How long after feeding? Is it getting worse? What is the vomitus? Is it forceful? What is the color? Is it
greenish? How is the appetite? Is there any distention? Any change in bowel motion? Is there any change in color of urine? Is
the urine smelly? Does he have fever? Any cough or shortness of breath? Is this your first child? BINDS?
- FHx

Physical examination
- General appearance: LOC, signs of dehydration (sunken fontanelle, skin turgor, mucus membranes, CRT) pallor or jaundice
- Vital signs and GROWTH CHART!!!
- ENT: LAD
- Chest and heart
- Abdomen: distention, palpable mass, tenderness, feeding test
- Hernia orifices

Diagnosis and management


- Your child has a condition called hypertrophic pyloric stenosis. It is due to the thickening of the muscles of the pylorus. The
cause is unknown but there are some risk factors such as male child, family history, first baby,
- I am afraid that we need to admit him because he seems to be dehydrated. I will call the pediatric registrar. I will first secure 2 IV
lines and take blood sample for FBE, Urea and electrolytes, acid-base and blood glucose. I will also organize the ultrasound as
well. We will review Kevin every 4-6 hours to see whether he has improved. We aim to correct his electrolyte imbalance until 48
hours. Once he is stable, the surgeon might consider doing a surgery (Pyloromyotomy) which can be done as a keyhole surgery.
They will cut these muscles and make it wider. Once the surgery is done, you can start feeding your child within 6 hours and we
will discharge him in about 2-3 days if without complications.
- Can it happen again? Restenosis can happen but it is uncommon.
- Reassure!!!

Management
History suggestive of Pyloric Stenosis

YES

YES
Exam: Pyloric Mass Discuss with senior doctor
Visible Peristalsis Refer for surgery

NO

USG of abdomen and/or


acid/base abnormal (pH
POSITIVE
>7.45, Cl <98, BE >+3

Negative
Discuss with senior DR
Consider other Dx
Arrange review in 1 week

Functional Constipation

Case: .You are a GP and a 7-year-old girl was brought by her mom because of tummy pain for 4 weeks.

Task:
39

a. Focused history
b. Physical examination
c. Differential diagnosis
d. Ask for relevant investigations from examiner
e. Management

Differential:
- Constipation
- Lactose intolerance
- Abdominal colic: infants
- Functional abdominal pain: psychosocial factors
- UTIs
- Abdominal migraines (+) family history
- Mesenteric adenitis
- Subacute appendicitis

Causes of constipation:
- Lactose intolerance
- Dietary factors
- Psychogenic factors (fear)
- Anal fissure/proctitis
- Thyroid disease (hypothyroidism)
- Medications (codeine-containing)
- Congenital hirschsprung disease
- Spinal cord problems

History:
- How the child is doing? Is she in a lot of pain at the moment? What type of pain? Sharp or dull? Where exactly is it (point to one
finger)? Does it go anywhere else, like the back or the inguinal area? Is it associated with n/v/d/constipation? Do you think it is
related to her diet? How is her waterworks? Does she complain of burning while passing water? has she ever passed hard
stools? How often does she open her bowels? Any history of constipation? Is she soiling her undies (encopresis)? Does the pain
affect her sleep? Any particular time for the pain? Is it there on the weekends? Do you think her growth and development has
been affected? Has she ever passed blood in the stool? Does she complain of pain on passing stools? Fever? Suffered from
viral illness before the pain?
- FHx: Thyroid? Food allergies? Migraines?
- Do you think she is suffering from any kind of stress at school? Bullying? Does she go to childcare? Any problems? Whom does
she live with him at home? Relationship with members?

Organic Functional
Sleep Affects sleep Sleeps well
N/V + May have nausea
but otherwise
asymptomatic
Radiation + Around belly-button;
well-localized
Weight/Growth + normal
affected
Cause + May elicit
psychosocial factor

Physical examination:
- Ask for growth chart?
- General appearance: pallor, jaundice, dehydration,
- VS
- Palpate all LN of the body, chest and heart
- Abdominal exam: any visible distention or mass on inspection; tenderness on palpation; visceromegaly; can I feel fecal masses
w/in abdomen? Can I listen to the bowel sounds?
- No PR unless aim is to empty the bowel. Inspection around anal area looking for any anal fissure, bleeding, soiling of underwear,
- Urine dipstick

Investigation:
- Abdominal xray – erect and supine
- Thyroid function test; FBE;

Management:
- Most likely what your daughter has is a case of constipation. It is very common esp in school going kids (28%).
40

- Constipation can come from a number of causes including a thyroid disease, any local disease around the backpassage,
improper dietary habits, and certain medications
- For your child, we have done an xray that shows her bowels are loaded with fecal matter. At the moment, we need to empty the
bowels. There are a few options available. The best one for her at the moment is lactulose available in the form of sachets that
you can add to any drink that she takes or water. The effect of lactulose is to soften the stool making it easier to pass. We need
to keep the stools soft for a longer period of time so please keep using them until she can develop regular bowel movements
without them
- It is also important to make some changes to her diet including improved water intake and fiber (green leafy vegetables).
- It is good that she is active and playful. Once the pain goes away, you will need to toilet-train her once more. She needs to go to
the toilet and sit for at least 3-5 minutes 3x a day after meals. The aim is to improve the connection between the brain and the
gut. Please understand this treatment will take a few weeks to work. If during this time she develops severe pain or very hard
stools that are difficult to pass please go the emergency, they might give her an enema (microlax – Na citrate).
an enema (microlax – Na citrate)
- Chronic
Other options: liquid paraffin oil, sennosides granules, macrogol sachets, lactulose sachets Diarrhea
(2-3x/day)
- Reading material for healthy eating habits, followup. (>2-3 weeks)
actulose sachets (2-3x/day)
Toddler’s Diarrhea Failure to Thrive

Case: You are a GP and a father brings his 18-month-old son because of diarrhea for 4 weeks. Initially, he had vomiting and diarrhea
which settled. Now, only the diarrhea persisted. The parents are concerned.
YES No
Task
Stool Culture Toddler’s Diarrhea
a. History (had gastro 4 weeks ago, diarrhea 2-3x/day, had undigested food, introduced solid at 6 months of age, drinks a lot of fruit
juice)
b. Physical examination (looks well and has no signs of dehydration; growth chart)
c. Investigation if necessary No Pus Pus/Mucous
d. Diagnosis and management Inflammatory Bowel
Disease
History
- When did it start? How many times a day? What is the consistency? What is the consistency? Is it smelly? Any mucous or
blood? Any undigested food? Do you think that the stool is hard to flush? Any associated symptoms like tummy pain? Is he going
Non-watery:
Watery:
to childcare? Anyone with similar complaints? Does he take aOsmotic Diarrhea
lot of juices? (fruit in his appetite? Has he lost weight? Any
Any change Positive reducing Sugars
juice)
concerns about his development? Is the immunization up to date? Anyone at home having diarrhea? Did you travelIntolerance
anywhere
Lactose
recently? Protein-losing enteropathy
- FHx of any bowel disease?

Physical examination
- General appearance: dehydration and muscle wasting
- Vital signs
- Growth chart
- Abdomen and hernia orifices
- Anal excoriations

Diagnosis and Management


- From history and PE, he has a non-specific chronic diarrhea. Usually we call it chronic diarrhea if is more than 2-3 weeks. In his
case, it is called toddler diarrhea. It is common in children 12-24 months and is characterized by a runny, sometimes, smelly
diarrhea with undigested food. Let me reassure you that it is not risky and most likely it is not infectious. Still, I would like to do
some investigations including stool microscopy and culture and reducing substances in stool.
- For the toddler’s diarrhea, I would advise to reduce the intake of fruit juice, increase dietary fiber, and consider reduction of milk.
We will wait for the results to exclude any infection and then I will write a letter to the child care.
- Reading materials.
- Red flags: increasing diarrhea, weight loss or poor weight gain.
41

Gastroenteritis in an Unimmunized Child

Case: A mother brings her 6-month-old baby to your GP clinic. She had diarrhea (5-6x) and vomiting 3-4x.

Task
a. History (diarrhea and vomiting x 2 days; decreased nappies; irritable; poor feeding; no immunization)
b. Examination (alert, reduced skin turgor, CRT normal, mildly sunken eyeballs and fontanelle, tachycardic)
c. Diagnosis and management

If less than 6 months: ADMIT!

Differential Diagnosis
- Gastroenteritis
- Celiac disease
- Lactose intolerance
- Surgical: intusussception, appendicitis, strangulated hernia
- Meningitis
- UTI
- Hemolytic-uremic syndrome

History
- Which started first, diarrhea or vomiting? When? Since yesterday? Frequency? Color? Mucus/blood? Foul-smelling? Amount?
- Vomiting: frequency? Amount? Content? Color? Blood? Forceful or projectile?
- Associated symptoms: fever or rash?
- Dehydration: Are the number of wet nappies as usual or decreased? Is the baby active or is he sleeping always? Feeding? Does
he cry while passing urine? Change in color of urine? Smelly urine? Is he drawing up his legs while crying or does he get pale
while crying? Anyone else in the family having similar symptoms?
- BINDS:
o Immunization is very important for children as it prevents them from many serious diseases. I would recommend to
keep the vaccinations up to date. I would like to organize another appointment and we can talk about immunization in
details if you’re happy.

Physical examination:
- General appearance: dehydration
- Vital signs and growth chart
- Signs of dehydration: skin turgor, sunken fontanelles/eyes, CRT, buccal mucosa (mouth)
- Skin: rash and neck stiffness
- Abdomen: Hernial orifices and signs of buttock excoriation (lactose intolerance and celiac disease)
- Urine (UTI and level of dehydration ketones) and BSL

Diagnosis and Management


- Your child has a condition called gastroenteritis, commonly known as gastro. It is an infection of the gut mainly caused by viruses
(rotavirus). I would like to assure you that it is a self-limiting condition and will resolve by itself. As it is most likely viral, John does
need antibiotics.
- Most important here is to replace the fluids the child is losing. He is mildly dehydrated so you have 2 options: if you are confident
and you have enough support, you can take him home, continue breastfeeding, give oral rehydration solutions, few mouthfuls
every 15 minutes and I would like to see John in 6 hours or if you want, I can arrange for the child’s admission. At this stage, they
would start rehydrating him via oral route.
- If formula feeding stop for 12 hours, give ORS every 15 minutes then reintroduce in small amounts and feed more frequently
- Lemonades and other juices are not ORS
- This is contagious so keep John away from other children. Wash your hands before and after feeding or changing nappies or
handling him.
- Can I visit my sister? Since this is contagious, keep John away from other kids, take care of hygiene, fluids, and see the doctor
there in 6 hours.
42

- Closure: Reading material. Red flags: drowsy/frequent sleeping, more vomiting and diarrhea, less wet nappies, or development
of rash or fever. Review in 6 hours.

Assessment of Dehydration Management


Mild: <5% of BW loss Rehydrate over 6 hours with
Dry mucous membrane* ORS/water
Thirsty, alert and restless ORS only is preferred in high-risk
patients (infants <6mos)
Moderate: 5-9% BW loss May require rehydration via the
All of above signs PLUS rapid pulse, nasogastric route
decreased peripheral perfusion*, Reassessment including BW and
sunken eyes and fontanelle, deep clinical examination is required at 6
acidotic breathing, pinched skin hours
retracts slowly (1-2 secs)*, If fluids replete, maintenance age-
appropriate fluids can then be used
Weigh patient every 6 hours during
the first 24 hours of admission
Introduce food intake after
rehydration if dehydration has been
corrected
Severe: >9% BW loss Can start ORS once initial
All of above signs PLUS if shock is resuscitation is completed – give
present (NSS 20 ml/kg/IV over 6 hours
repeated IV bolus until organ Following rehydration, management
perfusion is restored) same as above
In infants drowsy, limp, cold,
sweaty, cyanotic limbs and altered
conscious level * these are the only signs proven to
In children apprehension, cold, discriminate between hydration and
sweaty, cyanotic limbs, rapid feeble dehydration 4% or greater
pulse and low blood pressure

Chronic Diarrhea (Giardiasis)

Case: Your next patient is a 3-year-old child who came in to your GP clinic with his mother because of diarrhea for 3 weeks.

Task
a. History (diarrhea, profuse and explosive, foul-smelling, 4-5 episodes per day, no travel, brother also had diarrhea)
b. Physical examination (+ pallor, no signs of dehydration)
c. Provisional Diagnosis and Management

Differential Diagnosis for chronic diarrhea


- Giardiasis
- Coeliac Disease
- Lactose Intolerace
- CMPI
- Infection from Childcare
- Travel (Amebiasis)
- Inflammatory Bowel Disease
- Antibiotic-associated diarrhea

Features
- Characterized by abdominal cramps, flatulence and bubbly, foul-smelling diarrhea persisting beyond 2-4 days
- Causes atrophy of duodenum

History
- How often does he go to the toilet? What is the consistency of the stool? What is the character of the stool? Any blood or
mucous (“peas and carrot”)? Is it difficult to flush or does it float in water? What is the color of the stool? Any nausea, vomiting,
fever or tummy pain? Any change in his diet? Do you think diarrhea is associated with any particular food? Does
43

he drink too much milk or juice? Does he go to kinder of childcare? Anyone in childcare having similar complaints? Any other
family member having similar problem? Did you travel anywhere recently? Is he taking any sort of antibiotics for some reason? Is
he playing well? Does he vomit? Any loss of weight or appetite? Any fever? How about his wet nappies?
- BINDS?
- FHx

Physical Examination
- General appearance: pallor, dehydration, jaundice
- Growth chart
- Vital signs
- Abdomen
- PR: anal excoriations and muscle wasting of the buttock
- Urine dipstick

Diagnosis and Management


- From the history and examination, Kevin has a condition called chronic diarrhea. There could be a few reasons for this. One is
infectious and the most common organism causing chronic diarrhea in this age group and in children going to childcare is
giardiasis which is a kind of protozoa. However, I cannot be certain until we do some investigations. There could be other
reasons such as lactose intolerance, toddler’s diarrhea, celiac disease although they are less likely because I have not found any
signs of anal excoriation or frothy stools, steatorrhea and blood or mucus suggestive of these conditions . SO I would like to run
some investigations for Kevin. I would like to you to submit a sample of his stool and we will do stool microscopy and culture to
confirm this. We will be looking for cysts or trophozoites in the stool. I will also recommend to do celiac screening and test the
stool for reducing substances.
- What is if Giardia? How would you treat it? For Giardia, we will give antibiotics like Metronidazole (30mg/kg x 3days) or
(tinidazole 30 mg/kg SD). The main transmission of Giardia is through contaminated water. It is less likely transmitted from food.
Therefore, it is very important to maintain hygiene. Everybody should wash their hands after toilet and before handling food.
Clean toilet everyday with disinfectant.
- I would like to see Kevin in 3 days time and if the diarrhea persists even after antibiotics we will need to treat the whole family. I
would advise to see the brother as well and do some investigations.
- Reading material. Review.
- Red flags on dehydration and infections.

Salmonella Counseling

Case: You are an HMO and a child diagnosed with Salmonella is now ready for discharge.

Task
a. Talk to parent before discharge
b. Answer questions

Case: Your next patient in GP practice is a 5-year-old Matthew brought in by his mother Flora complaining of colicky abdominal pain with
diarrhea for the last 24 hours. She is very concerned because another child from childcare center has developed the same symptoms.

Task
a. History (diarrhea 8-10x, not-blood stained, and vomiting 5 episodes of water and food particles, + fever, decreased feeding and
appetite, passing water, had Christmas party yesterday and 2 kids started to have diarrhea; immunization up to date)
b. Examination findings (miserable looking boy with moderate dehydration, not pale, vital signs: T: 37.9, 105, BP 80/60, oxygen
saturation 98%; mild generalized tenderness, no organomegaly, no inguinal masses or lumps)
c. Investigations (stool microscopy for salmonella; )
d. Diagnosis and management
e. Answer her questions

- Mary I am happy to hear that your son is going home. Is there any diarrhea, tummy pain, hot to touch?
- Your son had a contagious bug called salmonella which caused gastro in him. Salmonella can affect any age but mostly in
childhood and young adults. The severity depends on the age, general health of the person, and on the number of bacteria
ingested
- From where did he get the infection? How do you prepare your food? (we have a takeaway shop and have food from there). He
can catch infection from people around him even if they don’t have symptoms. They are called carriers but they still have some
bug in their tummies. We can say that if you go to the toilet and not wash your hands properly, you can transmit the infection.
Also the knife, kitchen tools and towels used for raw food. When you deal with raw food, use a separate knife. Dealing with raw
eggs can also transmit infection.
- Any pets at home? They can also transmit the infection through their feces. So whenever you handle pets, you should wash your
hands.
- I have notify the DHS. Salmonella is a reportable infection and I am obliged to do it but it does not mean that they will close your
shop. They will take samples from the food in your shop and they will do stool culture for the whole family. They will take actions
based on the results.
- I would like to followup your son. We will need to do a series of stool cultures: 10 days after discharge and then every 2-3 weeks.
We have to confirm 3 negative cultures to say that he is free from bugs.
44

- When can he go to school? As soon as he improves from the symptoms, he can go to school but take care of hygiene.
- Antibiotics? No. they are not usually indicated. Indications for antibiotics are:
o Immunocompromised child, including malignancy
o Infant is < 3 mos.
o Malnutrition
- Critical issues: notification and prevention advice

Iron-deficiency Anemia (Coeliac Disease)

Case: You are a GP and a 15-month-old boy was brought in by father because the child had flu 1 week ago. You did some blood test.
Results showed hypochromic microcytic anemia with low iron levels.

Cases with anemia: cow-milk protein allergy, celiac disease, nutritional anemia

Task
a. Further history
b. Explain results to father
c. Discuss management

Differential
- Infectious diarrhea
- Protein losing enteropathy
- Toddler’s diarrhea
- CMPI
- Nutritional anemia
- IBD/IBS

Features
- Diarrhea + weight loss + iron/folate deficiency and abdominal bloating
- Malaise, lethargy, flatulence, mouth ulceration, alternating diarrhea and constipation, pale and thin patient, no subcutaneous fat
- Associations: IDA, type I DM, pernicious anemia, PBC, subfertility, lymphoma, dermatitis herpetiformis, IgA deficiency,
autoimmune thyroid disease, osteoporosis, neurological (seizures, ataxia, peripheral neuropathy), Down syndrome

History
- Please tell me more about your son’s condition? Has he recovered from the flu? Any fever, cough, runny nose?
- I need to ask you a few more questions that are related to the blood tests that we did. Can you please tell me his typical daily
diet? Does he take enough fruits, veggies, and meat? What type of milk does he take? How much milk does he take in a day (>1
year: 2 bottles a day; >2years: 1 glass at night)? When did you start giving him solids? Any problems after the introduction of
solids, like diarrhea or constipation? Do you remember your wife’s pregnancy and delivery? Any complications with that (preterm
or LBW are risk factors)? What was his weight after delivery? Was he breastfed? Do you think he has been gaining weight
appropriately since birth? Did you bring his blue book? Any concerns about his development? Immunization? Any problem with
waterworks? How are his bowel habits? Do you think he passes bulky, hard to flush stools? Any diarrhea in between? Are the
stools offensive? What is the color? Do you have a family history of anyone on a special diet?

Management
- Let me explain to you the results of the tests we did. It shows that your son has a low level of hemoglobin. One of the causes of
low Hb is low iron which is essential for the development of Hb. That is why he looks a bit pale. From the history, it seems like he
has a healthy balanced diet, so that is not contributing to his anemia. So my main concern is that he might be suffering from a
condition we call as celiac disease or gluten-sensitive enteropathy. Have you heard about this?
- It is a condition where there is poor absorption of certain food from the gut. As a result, the child develops diarrhea with or
without vomiting, tummy pain, distention, and sometimes loss of weight. The exact cause is not known, but research shows that
it has been linked with certain autoimmune conditions as well as genes. This condition is associated with a higher risk to develop
diabetes later on in life, thyroid problems, skin problems (dermatitis herpetiformis), and sometimes cancers of the bowel
(lymphoma). Please don’t worry treatment is quite simple, but he will require long-term followup. We still need to confirm the
diagnosis by doing some tests that we call as celiac screening test. We will be checking certain

antibodies in the blood (anti-gliadin, anti-endomysial, tissue transglutaminase antibody) as well as the level of proteins. We will
keep on doing regular TFTs for him.
- The definitive diagnosis will be done by endoscopic biopsy wherein we take a small piece of tissue from the lining of the small
bowel and we will check it on the microscope. Once the diagnosis is confirmed, you will need to have regular follow-ups with the
pediatric gastroenterologist. The treatment requires avoidance of gluten-containing foods (BROW barley, rye, oats, wheat).
There are special food aisles in the supermarkets with food labeled as gluten-free for your child. Please check all food items
before giving it to the child. You will need to inform the childcare or his school.
- I will refer you to the dietitian as well if you need a diet chart for him. There is a celiac society of Australia that you can contact.

Intusussception
45

Case: You are an HMO in the hospital and your next patient is a 5-month-old boy presented with screaming and pallor. A lump is felt on
the right side of the umbilicus.

Task
a. Focused History (diarrhea x 2 days before, noted lump on the right side in AM, poor feeding and vomiting during breastfeeding,
screams and draws up his legs, no dirty nappies since last night)
b. Explain to mom what is the condition and management

Differential Diagnosis
- Intusussception
- Volvulus
- Hirschsprung disease
- Impacted feces
- Gastroenteritis
- Hernia

Features
- Telescoping of bowel loops
- Infant cries and raises legs up and is noted to be pale
- (+) currant jelly stool – late sign
- Diagnostics: USD
- Treatment:
o Air enema 2x observe for 24 hours
o Surgery for relapse or unsuccessful attempts
- associated with HSP
- recurrence of 20-30% during the 1st 24 hours

History
- Can you tell me more about what happened? Is he breastfeeding? Is he hot to touch? Did he vomit? Does he draw up his legs
when he’s crying? Is it continuous or episodic? Are these episodes frequent since yesterday night? Was he able to sleep? How
about the bowel? Has he passed any stool? BINDS? Past medical history?

Physical examination
- General appearance: irritable, pallor
- Vital signs

- Abdomen: sausage-shaped mass RUQ or crossing midline in epigastrium or behind umbilicus; distention; auscultation (some
areas with high-pitched bowel sounds and some area silent)
- Hernia orifices

Investigation:
- AXR: rule out perforation
o Target sign: 2 concentric circular radiolucent lines usually in RUQ
o Crescent sign – crescent shaped lucency usually in the LUQ with a soft tissue mass
- Ultrasound: diagnostic and therapeutic
- Blood glucose, FBE, UEC, Blood group

Intussusception: history and examination


IV access/NPO
Analgesia
Notify ED/consultant and surgical registrar

Signs of shock IV fluid resuscitation


OR hypovolemia (20ml/kg NS bolus

AXR with Laparotomy


perforation? NGT on free drainage
IV fluids
IV cefazolin, metronidazole
AXR
demonstrates
NGT on free drainage
SBO?
IV fluids

High suspicion of
USD
intussusception?
Notify surgeon
Consider medical escort

Discuss with surgeons,


consider USD, DDx (self-
reduced intussusception) & Reassess
admission IV cefazolin/metronidazole
46

Diagnosis and Management


- Most likely your son has a condition called intussusception. It is a problem with the intestine in which one part of the bowel slides
into the next part. When it happens, it creates an obstruction in the bowel and leads to swelling, inflammation, and decreased
blood flow to the part involved. It can cause serious complications such as necrosis and perforation. It is a relatively common
condition seen in children 3 months to 3 years.
- Children present with colicky abdominal pain, pallor, draws up his legs when crying and a mass is palpated in the abdomen.
- All children suspected with intussusception require urgent surgical assessment and radiological investigations. That is why we
need to admit your child to the pediatric surgical ward. He will be assessed by a pediatric surgeon. I will organize Xray and
ultrasound, secure IV access, and we will not give anything by mouth. I will call the pediatric registrar.

They might consider starting IV antibiotics.


- The radiologist and pediatric surgeon will perform air enema which is both diagnostic and therapeutic. If it resolves, we will keep
him under observation for a day or two because there are chances of recurrences during the 1 st day. If it happens again, the
specialist may consider performing surgery.
- Complications of air enema: small risk of bowel perforation and bacteremia
- The procedure will be done by experienced hands. The prognosis is excellent.

Intestinal Obstruction

Case: You are an HMO in a rural hospital and you are about to see a young woman who had NSVD with a 1 day old baby. She was
discharged home but came back because the baby has greenish vomiting afterwards. The baby did not pass meconium. The Xray of
abdomen was given.

Task
a. Explain Xray finding to the mother (planned pregnancy, no resuscitation, started getting vomiting right away, no urine, no
meconium, 2-3x green vomiting)
b. Focused History
c. Management

Differential Diagnosis
- Duodenal atresia
- Hirschsprung disease
- Malrotation
- Volvolus
- Meconium ileus

History
- How many vomiting did he have? Do you know if he passed flatus? Is the tummy distended? Did you try to feed him? What
about your pregnancy? Was it planned? Any problems with antenatal care or during birth? Do you have any FHx of cystic fibrosis
(chronic infection of both lung/gut)?
- Social history: Did you come here alone? Do you have a lot of support? Do you have other kids at home?

Diagnosis
- From the xray and the symptoms, this is most likely due to a small bowel obstruction which could be because of duodenal
atresia. This is a medical emergency and it can go serious if not taken cared of now. Because we are in a rural hospital, I will
refer your child ASAP to the tertiary hospital. I will contact NETS/PETS and they will organize air ambulance for the baby.
- Once there, a small tube will be passed through the nose (mouth?). This is put to stop the secretions from the stomach and
decompress it. They will also put an IV line for the fluids that will secure his feeding. Your baby will be assessed by a pediatric
surgeon and anesthetist.
- Please do not feed him now and after surgery until you are advised to do so.
- Do you want me to call anybody for you? Would you like me to organize a social worker for your kids?
- My baby is only one day old, are you sure he’s safe for surgery? Yes, I am sure your child will be handled by expert doctors.
47

- Can it be serious? Yes. After, surgery, there could be adhesions or obstruction. So you have to be careful and anytime he gets
painful or distended tummy, you need to go back to the hospital immediately.
- Doctor my breast is leaking, what do I do? The lactation nurse will help you on how to express your breastmilk and store it for
future use.
- Is it my fault? There is nothing wrong with your caring or nurturing. In most cases, the cause is unknown.
- I will contact my senior who will come and take a look.
- What are you going to do now? I will ask the mother to stop baby’s feeding, call NETS immediately and my registrar.
- What is the outcome? Let me reassure that the outcome in the tertiary hospital is good. The sooner we manage the better it is.

Encopresis

Case: You are a GP and a 5-year-old boy was brought in by mother because of soiling of underpants.

Task
a. History (6 weeks ago, constipation 3 months ago with bleeding, recent teasing at school)
b. Physical examination (shy, vitals stable, growth charts, soft, non-tender, no organomegaly, + fecal masses are felt at lower
quadrants
c. Diagnosis and management

History
- Features: involuntary passage of formed or semi-formed stools into the underwear occurring repeatedly for at least one month in
children over 3-4 years of age who previously controlled his motions in the absence of any organic causes.
- Reasons: constipation, emotion, diet

Differential Diagnosis
- Encopresis
- Hirschsprung
- Drugs
- Anal fissure
- Hypothyroidism

History
- I understand your concern. Since when did you notice fecal soiling? How often? Any blood or mucus? Any tummy pain or pain
while defecating? Was he toilet-trained before? Any history of constipation or hard stool or difficulty passing stool? How is his
weight and appetite? Any problem with waterworks? Is he dry at night? How about school? Is he happy going to school? How
about his school progress? How is the situation at home? Are you a happy family? Any other medical illness such as thyroid? Is
he taking any medications? BINDS. Can you please describe his diet to me? Does he drink water?

Physical Examination
- General appearance
- Vital signs
- ENT/Chest and heart
- Abdomen
- PR: Perianal area if with fissures or excoriations; on rectal exam, rectum is packed with firm feces

Diagnosis and Management


- Your child has a condition called encopresis which is the involuntary passage of fecal material in underpants. The most common
cause for this is either constipation or dietary factors like if the intake of fiber or water is less and emotional factors such as fear
of pain during defecation.
- What happens is in chronic fecal retention, the large intestine gets stretched and increasingly becomes less sensitive to nervous
stimulation and child does not know when to go to toilet. As a result, liquid feces from small intestine may leak around the harder
feces in the large bowel and onto the underwear. Don’t worry it is a manageable condition.
- At this stage, first of all we need to empty the rectum by giving enema. The next day we will give rectal suppositories (dulcolax)
and on the third and succeeding days, fecal softeners and laxatives which we will continue for some time.
- Also, it is very important to toilet-train him regularly. The child should go to the toilet 2-3x ideally after breakfast and dinner
because our bowels tend to contract after we eat. Ask him to sit on the toilet for 5-10 minutes.
- The third factor is the diet. Make sure he drinks a lot of water, eat vegetables and fruits, and other high-fiber foods.
- Please do not scold the child and praise him when necessary. Talk to teachers about bullying. Star chart.
- I will give you reading materials and review.
- Red flags: vomiting, distention, situation getting worse, diarrhea, etc..

Pinworm
48

Case: Your next patient in GP practice is a 3-year-old girl who came in with mother who complains that she is always scratching her
bottom.

Task
a. History (worse at night, not eating well, irritable, decreased appetite)
b. Diagnosis and management

Differential
- Pinworm
- Vulvovaginitis
- Eczema
- Foreign body

History
- I understand that your daughter is scratching her bottom? Is it the first time? Where exactly is she scratching (anal or vagina)?
Did you notice any rash? Are there scratchmarks? Is there anytime when the itch is really bad? Any diarrhea or vomiting? Fever?
Does she go to childcare? Any similar episodes from childcare? Family member with similar problem? Waterworks? Did you
notice any worm in the stool? Rash?
- BINDS , medications and allergy
- FHx of asthma or eczema?

Diagnosis
- Check anal area at night 1 hour after going to sleep
- Scotch tape test early in the morning to be sent to lab

Management
- Your daughter has infestation of a pinworm. These are tiny white worms common in this age group. The eggs enter the body in
contaminated food, hatches into a worm in the gut. The adult female worm lays eggs in the anus at night which provokes
irritation and itching. During scratching, eggs go under the nails of the children and touching food with unwashed hands causes
transmission. The diagnosis is clinical but to confirm it, you can put an adhesive tape on the anus at night and in the morning,
remove the tape and send it to the laboratory to demonstrate the eggs. Also, 1 hour after sleep, you can take a look at the
backpassage of the child and you may be able to see some worms.
- Do not worry. It is a manageable condition but hygiene is the key to solving the problem
- Self help:
o Hands thoroughly washed after going to toilet, after handling a pet and before touching and food
o Fingernails should be clipped short
o Wear pajamas rather than nightgowns
o Shower each morning
o Bed linen, nightwear, underwear changed and washed in hot water
o Rooms should be vacuumed daily
o Clean toilet seat regularly with disinfectant
- Medical treatment
o Pyrantel, albendazole or mebendazole 50mg if <10kg or 100mg if >10kg SD (repeated after 2 to 3 weeks).
o If unsuccessful treat all family members
o Ointment to relieve anal irritation
- Could it be from pets? usually from human to human but you can have your dogs checked by a veterinarian
- Reading material. Review after 2 weeks.

Post-Splenectomy Counseling

Case: A 10-year-old boy injured his spleen and removed in the hospital. His father wants to discuss his son’s condition.

Task
a. Explain immediate risk of condition
b. Explain long-term risk and further management

Management
- Immediate risks: Bleeding, infection, damage to adjacent organs, clot formation in porta vein, lung collapse, and thrombocytosis
- Long-term risks
o Overwhelming Postsplenectomy Sepsis haemophilus influenza, streptococcus, meningococcus
o Amoxicillin prophlaxis x 2 years
o Immunizations: PCV 5-yearly, meningococcemia 3-5-yearly, influenza yearly, Hib once
o Emergency/Standby antibiotics: if with fever or showing signs of infection
- Education and medical alert bracelet
- Travel: malaria and Babesiosis (tick-born), animal bites
49

Counseling
- The spleen is an organ present on the left side of the abdomen under the rib cage near the stomach. It is an important part of the
body’s defense or immune system and helps to fight infections. It also acts as a blood filter to remove unneeded materials such
as old or damaged RBC and bugs. One of the functions of the spleen is to produce RBC and certain type of antibodies if our
bone marrow is unable to.
- One of the most common causes of removal of spleen is after abdominal injury when it is not possible to preserve it. Your son
can live an active life without the spleen, but there is an increased chance to contract serious and life-threatening infections. I will
be talking about long-term and immediate complications and how to address them. If you have any questions or concerns do not
hesitate to stop me.
- Immediate: John will stay in the hospital until his condition improves. It takes less than a week and can take 4-6 weeks to heal.
He will be given IV fluids and pain medications to ease the discomfort and his condition will be monitored on a regular basis. The
things that can happen include bleeding or infection or injury to nearby organs and lung collapse. Others are blood clot in portal
vein that carries blood to liver and thrombocytosis which is increased platelets for 2-3 weeks with an increased risk of
thromboembolism.
- The long term is overwhelming infection especially with streptococcus in 50% of cases. Others include meningococcemia and
haemophilus influenza. This is called overwhelming post-splenectomy infections which is common in the first 2 years after
splenectomy in young children. The incidence can be reduced by preventive measures. The first is long-term antibiotic
(amoxicillin 250mg OD or phenoxymethylpenicillin 250 mg BID or roxithromycin/erythromycin if allergic). The duration is
controversial. It can be lifelong or according to the needs or wishes of the patient especially if incapable of taking it. In this case,
we give for at least 2 years and have standby antibiotics if with signs of infection (Amoxicillin, co-amoxiclav). In case of
unexplained fever, tiredness, weakness, patient should take antibiotic and consult with GP ASAP.
- Ask about previous immunization. Then given vaccination advice: PCV 5-yearly, meningococcemia 3-5-yearly, influenza yearly,
Hib once if non-immunized.
- Further strategies: education about risks and early recognition of infection. Ideally, the patient should carry a medical alert
bracelet. With your consent, your GP should be informed and given detailed information.
- Travel risk: increased risk of serious malaria that needs special care when traveling to endemic areas. Active malaria
prophylaxis, mosquito repellents, use of barrier precautions and avoid bushwalking, etc… should be advised.
- If a patient is traveling to a region where he could get an unusual infection from ticks that is called babesiosis which can be fatal
because spleen is an important organ to remove it, warn him before travel. For this reason, contact an infectious disease
consultant or expert travel advisor before travel.
- If there is probability of travel to endemic areas where there is a high chance of meningococcal infection, patient should receive
vaccine regardless of previous vaccination status.
- If patient has animal bites, then he should have antibiotic coverage like amoxicillin or augmentin for at least 5 days.

- Reading materials.
- Red flags: increased redness, pain, discharge for incision site, increased swelling or bleeding, fever >101C (39), chills, N/V,
cough, SOB, diarrhea, immediately come to the hospital
- Arrange regular review and follow-ups.
- Other complication: hernia at incision site.

HEMATOLOGY AND ONCOLOGY

Immune Thrombocytopenic Purpura

Case: You are an HMO in a hospital and a 3-year-old Robert came with his mother complaining of a widespread bruising of skin for 2 days
and nosebleed yesterday for one time. He had flu-like symptoms 2 weeks ago. On examination, child is alert and numerous fresh bruises
on trunk and limbs are noted. Vital signs are normal. No hepatomegaly and no lymphadenopathy. Urgent FBE was done and Hgb, WBC,
Differentials (normal) and Platelet 15,000.

Task
a. Outline Provisional Diagnosis
b. Management

Case: Lily four years of age is brought to your GP clinic by her Mom Stephanie. She is worried about Lily as she had devolved rash all over
her body since last evening. Lily is well otherwise and is playing around. Lily started kinder this year and had cold/cough few days ago like
few other children in the school but now seems to be recovering. Lily lives with her parents and is fully immunized. She had no significant
medical or surgical problems in the past.

Task
a. Focused history
b. Examination and investigation from examiner (non-blanching purpuric rash)
50

c. Diagnosis and treatment advise

Differential Diagnosis
- ITP
- Meningococcemia
- Child abuse
- Acute Lymphoblastic Leukemia
- HSP
- Aplastic anemia

Management
- Your child has a condition called immune thrombocytopenic purpura. It is an immune reaction as Robert had a viral infection 2
weeks ago. His body produced a substance called antibodies to fight against the infection. Unfortunately, these antibodies are
attacking his tissues which are platelets. Platelets help our blood to clot. Now, as the number of platelets is reduced, it is causing
bleeding.
- I would like to reassure you that it is usually a self-limiting condition and most of the children recover within 4-8 weeks. At this
stage, I need to admit your child. Immediate treatment is generally not required because the probability of spontaneous
remission is very high but if the progress is not satisfactory, we will give oral/IV steroids if your child starts bleeding. In case he is
unresponsive to steroids, we will consider giving immunoglobulins. Usual indication: is severe acute bleeding and platelet is <10-
20,000.
- If Ig does not work, we will give thrombopoeitic agents to stimulate the bone marrow.

- If bleeding still continues, we might consider blood transfusion or platelet transfusion.


- Final treatment: splenectomy if not controlled
- There can be some complications of ITP: Intracranial bleed, other internal bleeding, and side effect of steroids
- Good news is prognosis is very good. More than 90% recover fully with 75% having spontaneous remissions and only 10%
develop chronic thrombocytopenia within 6 months.
- Recurrence, which is unusual, look for underlying causes like auto-immune diseases
- Advise:
o Not to play any contact sports
o Rest at home
o No IV/IM injections including immunizations
o No aspirin or NSAIDs
- I would organize pediatric registrar to come and have a look.

Pancytopenia/Acute Leukemia/High Grade Fever

Case: You are a GP and a 3-year-old boy was brought by mom because of fever (39) and tiredness. They saw a GP yesterday who
ordered some blood tests. They have returned today to discuss the results with you.

FBE: Hgb 80, WBC 2000, platelets 20,000

Task
a. History
b. Discuss results
c. Outline management

Differential Diagnosis of Pancytopenia


- Decreased production:
o Hereditary (Fanconi anemia)
o Postviral (Parvovirus B19, HIV, Hepatitis)
o Drugs (Chloramphenicol, sulphonamides, antimalarials, immunosuppresants)
o Leukemia (bone marrow exhausted of nutrition) ALL
- Increased destruction
o Hypersplenism
o Myelofibrosis
o Infections
- Megaloblastic anemia

History
- Please tell me more about his fever? When exactly did it start? What is the pattern? Did you check how high it was? Did you give
him some medications? Did it help?
- Please tell me more about his tiredness. Since when did he feel tired? Is it present at rest or with activity?
- Does he have any associated infection of the respiratory tract like cough, runny nose, discharge from the ear, earache, short of
breath? Have you noticed noisy breathing? How is his appetite? Any N/V/tummy pain? What about his bowel movements and
waterworks? Have you noticed a skin rash anywhere in the body? Did he have a fit at any time? Have you noticed bleeding from
the mouth, nose or anywhere else? Any lumps or bumps? Do you think he has lost weight recently? Do you remember any
complications during pregnancy and delivery with this child? Any FHx of bleeding disorders or blood-related cancers? Any past
medical or surgical conditions?
51

Immunization? Growth and development? Is he on any medications? Can you describe his typical daily diet to me? Was he ever
exposed to any radiation?
- May I ask, how is the home situation? Any stress? Who takes care of the child usually?

Physical examination
- General appearance: pallor, dehydration, jaundice
- Vital signs and growth chart
- Skin: bruising
- Lymph nodes
- Neck stiffness and signs of meningism
- ENT: signs of infection
- Chest and heart
- Abdomen: distention, obvious mass, spleen and liver for enlargement, bowel sounds

Investigations:
- UEC, PBS, urine for microscopy and culture, serum folate and B12, CXR, Blood culture, USD of abdomen
- Later on, the child will need to undergo BMA to confirm the diagnosis.

Management
- From history, examination and initial investigations that were provided to me, it seems like your child is suffering from quite a
serious condition we call pancytopenia. Usually in the blood, there are 3 types of cells namely RBC which carry oxygen, WBC
which fight off infection, and platelets which are responsible for the normal clotting mechanism. All these three cells are produced
by the bone marrow. For some reason, your child’s blood cells have reduced to dangerous levels so he is at risk of developing
serious infections, bleeding which may prove uncontrollable, and complications arising from a very low Hgb level.
- There can be a number of causes for pancytopenia. For example, certain drugs and inherited conditions. But what I am most
worried about is the risk of blood cancers or leukemias, which present in this way commonly. What we need to do is to send him
to hospital right away. I will liaise with the pediatrician at the hospital and they will do some more tests that include a biopsy of
the bone marrow. It is usually done under GA. With the help of a wide bore needle, they will take some tissue from the hip bone.
Don’t worry we will sedate him before hand so he won’t feel any pain.
- They will start him on some broad-spectrum antibiotics to prevent any infections and might start him on a high-dose of steroids. It
sometimes helps to bring up the level of platelets so if the counts fall below 20,000, they might need to give him blood
transfusion because of the risk of intracranial hemorrhages (bleeding from the brain).
- If proven to be leukemia on biopsy, the treatments include chemotherapy, radiotherapy, and bone marrow transplant.
- What is the outlook for my child? Once therapy is started, the 5-year-survival rate for kids is >85%.
- Don’t worry. You will have a lot of support. We will do everything possible for your child.

Henoch-Schonlein Purpura

Case: A 5-year-old boy was brought to you complaining of limping for the last 1 day.

Case 2: A 5-year-old boy was brought to you complaining of intermittent tummy pain and limping for the last 1 day. He had URTI 10 days
ago. A picture is given about the rash.

Task
a. History (no fall/trauma, fever and URTI 10 days ago, not passing urine like he used to, pain on both knees and ankles (arthralgia
of knee/ankles), rash on his legs and buttocks, abdominal pain, no pain anywhere else;)
b. Physical examination (no dehydration, palpable purpura, non-blanching from legs to buttocks, growth chart, VS normal,
tenderness in abdomen, urine dipstick ++ RBC and proteins, BSL)
c. Investigations (FBE, inflammatory markers, urine MCS and U&E/C, bleeding time and clotting profile (N))
d. Diagnosis and management

History
- Can you tell me more about it? Is he limping on one or both sides? Any trauma to the joints? Did you notice any swelling of the
joints? Any redness? How severe is the pain? Is he able to walk? Any other joints involved? Did it happen for the first time? Did
he have recent URTI? How long did he take? Any medication for it? Tummy pain? N/V? Is the rash itchy? What about his
waterworks? Is it decreased? Any burning or pain when passing urine? Any change in color of urine? Any pain his testis or
scrotum?
- How is his general health? BINDS? Medications? allergies?

Physical examination
- General appearance: pallor, dehydration, jaundice
- Vital signs: BP, growth chart
- Morphology of the rash
- ENT and Lymph nodes
- Chest and heart
- Abdomen: visible and palpable masses or tenderness
- Scrotal involvement
52

- Joint examination
- Urine dipstick and BSL

Management
- He most likely has a condition called HSP which is an inflammation or vasculitis of small vessels of his body. Most common age
of onset is 2-8 years. It usually follows an URTI which he had a week ago and the exact cause is unknown.
- I need to admit him to the hospital where he will be seen by the pediatric registrar. He will undergo some investigations such as
FBE, inflammatory markers, urine MCS and U&E/C, bleeding time and clotting profile.
- The management is largely symptomatic. There is no specific therapy except for rest, paracetamol for pain, and short course of
steroids after excluding intussusception.
- He will need followup because he has involvement of the kidneys. We will need to repeat his kidney function tests and urine
MCS as well as monitor his BP. In case of renal dysfunction, hypertension or surgical complications like intusussception, he will
be immediately referred to the specialist.
- This condition has a good prognosis and most patients recover fully in a few months
- Reading material. Review.

INFECTIOUS

Septic Workup Counseling

Case: You are an intern in HMO and a 2-month-old child has been brought in because of fever, lethargy and irritability. The pediatric
registrar has decided to do a septic workup for the child.

Task
a. Explain the component of septic workup to the mom and how you do it.

Septic Workup
- Blood Culture
- FBE, ESR/CRP
- SPA or straight catheterization for CS
- CXR
- Lumbar puncture

Counseling
- I understand from the notes that your child has fever and lethargy. At the moment, he is being seen by the specialist pediatrician.
Apparently, there is no focus of infection that is visible on examination. It is mandatory to perform a full septic workup on all kids
who present with fever without a focus. This includes taking blood samples to rule out germs or bugs in the blood, CXR to rule
out pneumonias, urine samples to rule out UTIs, and a sample of fluid circulating within the backbone to rule out possible
meningitis or “brain fever”.
- I would like to explain to you how we will take these samples and how we will minimize any pain or discomfort to your baby.
Usually, we take sample of the blood from the child’s veins to look for infection in the blood. It might cause a little bit of pain from
the needle prick but we will use a local anesthetic spray a few minutes before the procedure that will numb the area.
- The next procedure will be an xray of the chest. I understand that you might be worried about possible risk of radiation for such a
small baby. Please understand that the dose of radiation will be tailored according to his age and weight to limit overexposure.
This test is very important to look for infections or pneumonias within the lungs.
- In babies of this age group, we usually obtain a sample of urine through aspiration from the tummy to look for possible infections.
Please don’t worry it will be done by experienced hands. The nurse will give him some painkillers before the procedure. A very
small needle is passed through the skin into the bladder and the sample is withdrawn. Please don’t be alarmed if you see traces
of blood in his urine after the procedure. It is totally harmless and commonly seen after such procedure.
- The other important test that I want to talk about is a lumbar puncture where some fluid will be taken by passing a small needle
through the space between the lower spine. We will send this fluid for testing for infections. This procedure is quite safe in
experienced hands. Sometimes, there is small amount of bleeding from the area. The child may be irritable for some time, but
rest assured that it is unlikely to damage the spinal cord as the level is much lower.
- STOP after every explanation! Do not use MEDICAL JARGON
- Options: sucrose dummies, breast feeding, bottle feeding, anesthetics

Meningococcal Septicemia

Case: 2-years-old boy is brought by his upset/anxious father in a suburban hospital where you’re working as HMO. The child has become
lethargic and febrile the last 4 hours. He had mild URTI for the last 3 days and now he is uninterested in food, irritable and has a very cold
53

skin. On examination, he looks unwell and has fine non-specific macular petechial rash on the trunk and legs, and elsewhere, his skin is
cold and pale especially on the extremities. The vital signs are temperature of 40, RR 48, PR 150, BP: 90/60. There is no neck stiffness.

Task
a. Explain the diagnosis
b. Outline the management plan

- Is the patient hemodynamically stable? I would like to start with management?

Diagnosis:
- From history and PE, most likely your son has a condition called septicemia. In this condition, infection spreads through blood to
different parts of the body and it is a serious and life-threatening condition. You are in experienced hands. We will try our very
best to take care of Johnny and treat his condition.
- Because of the rash on his body, one of the possibilities to cause his condition is meningitis that is caused by a bug, N.
meningitides or H. influenza but we have to rule out the possibility of other infections also.
- Ask history to determine cause (child care, family, history of travel).
- Give prophylaxis (Rifamipicin) for close contacts
- Contact child care to find the cause
- Notify DHS

Management:
- I would like to admit him and start an IV line and take blood for culture, FBE, ESR and CRP, U/C/E, BSL
- Immediately after that I would like to give him an antiobiotic called cephalosporins. Does he have any allergies? cefotaxime q6
(50mg/kg up to 2g) or ceftriaxone (100mg/kg up to 4g) OD IV or IM or Benzylpenicillin (60 mg/kg) IV or IM.
- I would give him IV fluid to help stabilize his BP. Once this is done, I would like to contact the pediatrician to arrange his transfer
to the tertiary hospital. (If within 30 mins, travel by ambulance but >30 mins, send him by air ambulance).
- In the tertiary hospital, they will do septic screen (CXR, MSU for microscopy and culture, and LP). When the report of the blood
culture is back, the specialist might change the antibiotics depending on the result of the culture. Once he is stabilized, IV
antibiotics will be shifted to oral antibiotics and later on, the other family members will be given a preventive drug called
Rifampicin as prophylaxis.
- I will be notifying the DHS.
- Show empathy.
- Vaccination is protective but not 100%.
- Prophylaxis: Rifampicin 600mg BD x 2 days; ceftriaxone 250mg IM SD (pregnancy), Ciprofloxacin 500 mg SD (woman on OCP)

Critical errors:
a. not recognizing septicemia
b. failure to give antibiotics
c. failure to admit patient

Scarlet Fever

Case: Your next patient in GP practice is a 6-year-old girl brought by her mother due to fever and rash on the upper trunk and perioral
pallor.

Task
a. History (rough rash noted on the body x 4 days, and fever relieved by Panadol; sore throat x 2 days)
b. Physical examination (lethargic, 37.9, sandpaper like rash, strawberry tongue with white exudates)
c. Diagnosis and management

Case: You are a GP and a 5-year-old child was brought in by mother because of fever (38C), sore throat and rash.

Task
a. History
b. Physical examination (active, alert, dehydrated, T: 38.6, tachycardia, tachypnea, lymph node enlargement, no neck stiffness,
c. Investigation, Diagnosis and management

Fever + Rash + Sore throat


- Scarlet Fever (sandpaper like rash, maculopapular, strawberry tongue, perioral pallor)
- Kawasaki Disease/Mucocutaneous LN Syndrome
- Rheumatic Fever
- Measles (maculopapular, gingiva, top to botto)
- Rubella
- Mumps (flushed)
- Slapped Cheek (bilateral)
- Malignancy
- Meningococcemia (petechial, non-blanching)
- EBV/CMV (rash after use of antibiotics maculopapular; usually not involves the face or limbs)
- Child abuse (unilateral)
54

History
- When did the fever start? Did the fever and rash come together? Did you give anything for the fever? Where did the rash first
appear? Can you describe the rash? Any vomiting or diarrhea? How about the waterworks? Any burning or frequency? Any other
siblings who have the same condition? Does she go to school? Anybody in school or any family member having similar problem?
Is she eating well? Did she have any sore throat or flu-like illness before?
- How is her general health? Any lumps and bumps in the body? BINDS

Physical examination
- General appearance: ill-looking with generalized erythematous fine pink rash, rough (sandpaper-like) and blanches on pressure
with circumoral pallor.
- Vital signs
- Lymphadenopathy: enlarged and tender
- Neck stiffness
- ENT: white-coated tongue
- Chest/heart/abdomen
- Urine dipstick

Management
- Your daughter has a condition called scarlet fever. It is caused by a bug called GABHS. It can be transmitted directly or through
droplet infection. These bugs have infected your daughter’s throat and then they send the toxins through the blood to all other
parts of the body leading to rash. One of the signs that her tongue is

inflamed is known as strawberry tongue. Initially it has a white coat covering the inflamed papilla. Later on, the white coat goes
off and the red papillae are apparent. The rash has a definitive name known as sandpaper rash. Later they start to fade and the
skin gets peeled off in 7-10 days.
- I need to do some investigations to confirm: FBE, ESR/CRP, throat swab. I will start antibiotics (penicillin) 250mg BD x 10 days
(<10 years) or 500 mg BD x 10 days (>10 years).
- Is it risky? Yes it has some risks. It can affect the kidneys and cause glomerulonephritis, rheumatic fever and endocarditis, otiitis
media, septicemia, and sinusitis.
- When can she go back to school? 24-48 hours after she has started antibiotics.

NEUROLOGY

Febrile Convulsion

Case: You are working as an HMO in a metropolitan ED. The nurse asked you to see a 3-year-old boy who is in a cubicle with his mother.
The nurse hands you the triage sheet saying “3 year old presents with ambulance after having seizure at home”.

Task
a. History (watching cartoons on TV and suddenly started shaking all over his body, lasted <1 minute, was fine after that, did not
wet himself, flu for the last 2 days with temperature 38C; husband with history of seizures)
b. Physical examination (looks happy and interactive, vital signs normal except T:38.2, chest and heart normal, soft, nontender,
c. Diagnosis and Differential diagnosis
d. Management and prognosis
e. Counsel mother

Differential Diagnosis
- Febrile Convulsion
- Epilepsy
- Trauma
- Meningitis/Encephalitis
- Meningococcemia
- Hypoglycemia

Features:
- 5F’s
o Five months – Five years
o Fever
o Family history
o Focal (NOT)
o Frequent (NOT)

History
- What happened before the seizure? Fever? Sick? Rash? Headache? Vomiting? Diarrhea? URTI? Sore throat? Dysuria? LOC?
- During seizure: duration? Focal or general? Bowel and bladder control? Tongue bite?
- After seizures: drowsy/slept all day
- BINDS
- FHx of seizures/epilepsy
55

Physical examination
- General appearance and growth chart
- Rash and neck stiffness
- Vital signs and growth chart
- ENT
- Chest and heart
- Abdomen
- Neurologic examination

Questions:
- Will it happen again?
- Does this mean he has epilepsy?
- Will he be brain-damaged?
- Does he need to see a specialist?
- Don’t you need to do some tests?

Management
- Advice
o Place the child on his or her side, chest down, with head turned to one side.
o Never lie a fitting child or unconscious child on his or her back
o Do not force anything into the child’s mouth
o Undress child to their singlet and underpants to keep them cool
o Obtain medical help as soon as possible.
o Ring or go to your local doctor or to your nearest hospital. Even if the fit stops, have your child checked
o Ring ambulance if seizure lasts >5 minutes
- How to prevent another episode: undress the child down to singlet and underpants, keep the child cool and give fluids and
paracetamol mixture
- Will not cause brain damage but will have 50% chance of having recurrence
- Risk of epilepsy: 2-3%
- Reading material.

Febrile Convulsion With Speech Delay

Case: You are a GP and a 3 year-old-boy was brought into the clinic by his mother. He had a fit lasting for 2 min around 1 hour ago. On
examination, everything is normal.

Task
- Take relevant history
- Explain the diagnosis and management to the mom

History taking
- What happened? How long ago? Who saw the fits? Pattern of fits – all over the body or only some parts of the body
o Generalized – febrile convulsion
o Focal – other pathology probably infection
- Did he bite his tongue or pass urine or stool during the fits? Was it the first episode? Recent infection (sore throat, ear infection,
UTI)? Did he have a fever at that time? Sore throat or upper respiratory tract infection?
- Immunisation – up to date
- Family history of epilepsy or fits during fever
- Pregnancy, delivery, Nutrition,
- Childcare or KG? Any exposure to other sick children
- Developmental problem – speech problems? How is his behavior otherwise? How is his interaction with you, your husband and
other kids?
o Is he able to make sentences comprising 3 or more words
o How is his behavior and interaction with family and friends

o Do you think he responds well to your voice when he is called


- Any family history of speech disorder or special child in the family
- How is his appetite? Water work? Medication and allergy?

Diagnosis and Management


56

- The most likely diagnosis is febrile convulsion and it is very common in this age group. Whenever the child develops a high
temperature, the brain senses some involuntary signals in response to the high fever.
- Please do not worry and this is not epilepsy. The only problem is that there is a 30% chance that it might recur in the next 24
hours. I want you to be aware of what to do if it happens again.
1. Whenever he falls sick, check temperature.
2. Do not let temperature beyond 38C
3. Give paracetamol
4. If he develops fits, put him in left lateral position to prevent aspiration of stomach contents into the windpipe
5. Remove clothing and paracetamol per-rectal (20mg/kg)
6. The nurse outside will teach you how to do it
7. Once you recover from fits please come and see me.
8. Usually the fits last for less than 5 min
9. If they go beyond 10 min, and if they come again and again, go to hospital with ambulance
- Regarding concern about speech problem, there are many possibilities. I would like to refer him for hearing test, to the
pediatrician and speech pathologist.
- Chance of recurrence after 1st seizure is 30% and after 2 seizures is 50%
- Girls: 5 years old; boys: 6 years old
- Meningitis: if child had repeated seizures, headache, neck stiffness, if no other findings can be found
- Workup done only if no cause/focus can be found and if there is recurrence of febrile seizure

Tests
- ABR (auditory brainstem response)
- Otoacoustic emission testing

Causes of Speech delay


- Hearing defects
- Developmental disorder including Autism, Dyslexia (can’t read), Dyspraxia (unbalance orofacial muscle), ADHD
- Physical and emotional factors (child abuse, neglect, social deprivation)
- Anatomical problems – cleft palate, Cerebral palsy, MS, muscular dystrophies
- Anxiety and phobias
- Speech delay is common and more common in boys. 10% of pre-school kids suffer from some kind of speech problems. Failure
to communicate compared to other children is a sign.
- Thorough assessment will be done by speech pathologist including speech, language and motor skill assessment. The therapy
includes visual aids, toys, flash cards, exercises to strengthen the oral muscle.
- The prognosis is best if treatment is initiated before the age of 3 years and it is important that the family and the parents are
involved.

Absence Seizure/Petit mal epilepsy

Case: A 9-year-old boy is in your GP clinic, brought by his step mother, with complaint of staring blankly for a few seconds for the last
3months

Task
a. Take history from the mother (No Trauma, infection or loss of consciousness. Resume to his daily activity. His school
performance is also affected. Pregnancy and Birth is fine. Immunization is up to date. Diet ok. Water work and bowel functions
normal. Development is normal.No Fhx.)
b. Ask for physical examination findings from the examiner
c. Discuss the diagnosis and further management plan to the mother

Features
- Child stares blankly/Motion less for some time.
- Episodes happen in 5-10 seconds.
- Child can blink the eyes.
- There can be smacking the lips. Chewing. Some twitching of the muscle of eyes/face/fingers.
- No loss of consciousness. Lapse of consciousness.
- 4yrs to puberty. Later on: either settle down or have epilepsy.
- Need to exclude generalized seizure.

History
- Anything happened before or during the activity? Any shaking/ twitching/jerkey movement/tongue bite/loss of urine or bowel. Did
he have any injury, infection (meningitis)? Does he have any headaches? Any neurological deficits? Any weakness or numbness
of the limbs? Any problem with vision or hearing?
- BINDS: Any problem during the birth? Any trauma/resuscitation/medications? Growth charts. Feeding and general health all is
good. What is the home situation? School situation (His school performance has been affected). How is the financial situation?
How are you getting along with him (“She is his step mother.”)? Is he taking any medication? Any allergies? Past history of
epilepsy? Febrile convulsion? Family history of epilepsy?

Physical Examination: Nothing is positive.


57

- GA, Growth chart, V/S, Neurological deficits, rest of the exam.


- Hyperventilation can provoke the seizure episode. Ask the examiner.

Management:
- Most likely the condition he has is Absence seizure the cause is not found but can have a genetic predisposition. This is caused
by abnormal electrical discharges in nerve cells of brain. I am going to refer you to the specialist. He is going to do EEG
(Electroencephalography: 3Hz wave spike). This is a non-invasive procedure which involves placing the electrodes and
measuring the electrical activity of the brain
- Medications: Single drug for a long period of time (1year)
o 1st choice: Ethosuximide: S/E: related to the digestive symptoms, stomach upset, nausea, diarrhea, anorexia.
Dizziness or drowsy/ headache/ vision: Myopia, problem with gum. Pancytopenia/leukopenia. If you

find any infections, bleeding, problem with vision, headache please come back to the hospital.
o 2nd choice: Sodium Valproate: S/E: liver function. Monitor LFTs every 2months initially with the treatment.
- Advice the mother: Be cautious if he is bathing/ swimming/ driving. Do not close to the fire or any other dangerous activity.
Closely monitor the child. If they have generalized fits: don’t stop the fit, don’t put things in the child’s mouth. Put the child on his
side. Immediately come back to the ED if it prolong more than 10mins. I will inform the teacher also.

Provoked/Precipitated Fit

Case: A 16-year-old boy was brought to your GP clinic after having a fit at school immediately following an injury at the playground. No
relevant past medical and surgical history. His family including sisters are well. Complete examination after arrival was normal.

Task
a. Further history from mom
b. Give most appropriate investigation to examiner
c. Explain management in immediate future

Provoked seizures/Situational seizures/acute symptomatic occurs within 7 days after acute brain insult

Causes
- Structural in nature (head injury, stroke, infections such encephalitis, abscess, TB of brain, neurocysticercosis)
- Metabolic/toxic (kidney failure, liver failure, alcohol, drugs)

History
- Is the child hemodynamically stable or not?
- Please tell me more about what happened? Exactly when did it happen? were there any witnesses? Did he collide with anything,
something or someone? Do you think he tripped and fell? Did he land on his head? Any trauma to the head? Before the fit, was
he conscious? Breathing on his own? Was he confused? Any headache, N/V? Did anyone describe the fit to you? Was it over
the whole body? Or just one side of the body? Do you think the fit started in one part of the body and later involved the whole
body (Jacksonian March)? During the fit, did he wet himself? Or did he pass stools? Do you think he bit his tongue/lips? After the
fit, did he recover himself? Was he confused? Was he communicating with anyone? Is this the first time he had a fit? Any family
history of epilepsy? Regarding his health, did he have recent flu-like illness? Any recent change to his weight, or appetite? Any
problem with waterworks? Is he sleeping well? (sleep deprivation, stimulus for epilepsy) Are you aware if he might be using
some drugs, alcohol, or if he smokes? Do you think he is under a lot of stress at home, or in school? How is his school
performance? Any other concerns regarding his growth and development?

Examiner Investigations
- FBE, CRP (to r/oinfection), LFTs, KFTs (r/o liver/kidney), BSL (juvenile diabetes), urine dipstick (infection), chest xray and plain
xray of skull, drug screening with mom’s consent. Later on, CT scan might be needed.

- Guideline: CT scan/MRI is done as first line investigation

Management
- At the moment, all our examination is normal. It seems like there is nothing structurally wrong with his brain that might have
caused this kind of seizure because there is no family history and seizure was brought on by possible trauma to the head. I’m not
concerned about epilepsy. However, should he develop more fits, we will do an EEG to rule out epilepsy. Most likely, this was a
provoked fit that is commonly seen after head injury. However, there are certain metabolic as well as infectious causes of
seizures that we need to rule out, that is why I am sending him to the hospital where he will be kept under observation for a few
hours in ED. They will do some tests in the blood and urine as well as some imaging to rule out any serious causes. There is a
high chance that he might develop fit within the next 7 days. He will be observed by the specialist neurologist and if required,
they might give him some sedatives. However, antiepileptic medications are usually not required in these cases. If he loses
consciousness or with repeated fits or if the fits continue for more than 10 minutes, the specialist might start you on IV phenytoin.
- I will liaise with the ED consultant about your child’s condition and progress, please come back to me once he is discharged from
the hospital.
58

Tension headache

Case: You are a GP and your next patient is a 7-year-old girl brought by her father because of recurrent headaches

4 headache cases: tension, posterior fossa tumor, migraine, URTI (sinusitis)

Task
a. History (headache x 3 weeks – all over the head)
b. Physical examination (normal)
c. Diagnosis and management

Differential Diagnosis
- Migraine
- Tumor
- Meningitis
- Tension
- Vision problem
- Sinusitis
- Trauma
- Diabetes mellitus

History
- HPI: where exactly is the pain? When did it start? Is it going somewhere else? Is it throbbing, pinching, squeezing? Severity (1-
10)? Aggravating factors: hunger, light, movement (sinusitis), relieving factors, associated factors (early-morning vomiting, fever,
trauma)? Any lumps or bumps in the head? Any problems with vision? URTI? Cough or colds?
- HEADSSS? Are you a happy family? Do you spend time with her? Do you have financial problems? Does she go to school?
Does she like her teachers? Has she ever complained about any bullying? How is her school performance?
- BINDS – is she eating well? Is there any concern about the development? Do you have any other kids? Any major difference you
find with development of both?
- FHx: tumors, migraine, DM, etc… MA

Physical examination
- General: dehydration, pallor, jaundice, healthy
- Vital signs and growth chart; BP is important!
- ENT: FUNDOSCOPY is a must; sinusitis, tympanic membrane, neck, rigidity, LAD,
- Chest and Lungs & Heart
- Abdomen
- Neurological examination
- Urine dipstick and BSL

Child Neuro: ITPRC (inspection tone power reflexes coordination)

Diagnosis and Management


- Your child most likely has a condition called tension headache. It is important to realize that her headache is real but it is not
serious. The reason most likely is because you don’t have time for Mary and she’s probably missing that.
- I would like to arrange a family meeting. I would like to see your wife as well. For her pain, you can give her panadol. I would like
to refer her to the counselor. They are experts
- I would organize a social worker to liaise with the school (for school issues).
- Are you sure? Yes, from all the information you have given, I believe it is a tension headache.
- Can you refer me to a specialist? I am happy to refer you to a specialist for a 2nd opinion. If he needs to have some more
investigations, she may request to do so.
- Review. Reading material.
- Red flags: Vomiting, Fever, Rash
- MRI preferred in children than CT scan (less radiation exposure)

Child Having Increased Intracranial Pressure

Case: You are a GP and your next patient is a 10-year-old girl brought by her mom due to headaches for the last 6 months.

Task
a. History (6mos headache increasing in intensity; starts in occipital area then becomes generalized; associated early morning 6-
8/10 in severity with vomiting; affecting grades and skipping school, (+) FHx of migraine)
b. Physical examination (papilledema on funduscopy)
c. Diagnosis and management

History:
- Can you tell me more about it? Duration? Progressive? Type of pain (dull/steady/severe)? Severity? When does it usually
happen? How frequently does she have the headaches? Radiation (generalized)? Any particular timing in the day? Aggravating
factors: worse with coughing, sneezing or straining? Does the headache wake the child at night? Does she take any
medications? Associated symptoms: Nausea? Vomiting? Relieved by vomiting? Drowsy? Loss of consciousness?
Weakness/numbness? Diplopia? Gait problems? Problem with bright lights? Trigger factors (food, hormones)? Relieved by
sleep? Personality changes? Sinusitis, URTI? Have you ever checked her BP? School performance? Social stressors?
59

- General health? FHx (migraine, vision, tumors)

Physical Examination
- General appearance
- Vital signs: especially BP (140/90); PR: 70/minute shows Cushing’s reflex
- ENT: exclude sinusitis (tenderness of sinuses); visual acuity, PEARL, eye movements to check for diplopia, nystagmus,
FUNDOSCOPY (swelling and blurring of optic disc), visual fields, NECK STIFFNESS, gait
- Neurologic: tone, power, reflexes, gait, cerebellar signs, cranial nerves

Diagnosis and Management


- From history and PE findings, most likely your child has increased pressure on the brain which can be due to a nasty growth or
brain tumor. I do not mean to scare you but she needs to be evaluated further ASAP. I will call the ambulance and send you to
the hospital where she will be seen by a pediatric neurologist.
- To find the cause, she needs an MRI scan to see inside the brain and locate the abnormality, and if confirmed, she may need
surgery and that will be decided by the specialist and will be followed-up on a regular basis.

For migraine: Avoid triggering factors, paracetamol/NSAIDS at maximum, rest, refer to pediatric neurologist for prophylactic
treatment

Cerebral Palsy

Case: You are a GP and an 8-month-old baby is brought by his mother for routine immunization which was delayed for 2 months because
of his grandmother’s funeral. The child is arching his back and mother is concerned about this.

Task
a. History (noted to be arching back and not able to sit unsupported)
b. Physical examination
c. Diagnosis and management

Cerebral Palsy
- Cannot be diagnosed until 2 years of age
- Non-progressive but permanent disorder of movement and posture due to a defect in the developing immature brain
- Etiology: may be related to events in prenatal, perinatal, or postnatal periods; cause is unknown
o Perinatal asphyxia (<10%) cases and postnatal illnesses or injuries (10%)
o Association: LBW (<1500g) and prematurity
- Classification:
o Type of motor disorder (spasticity most common), dyskinesia,
o Distribution (hemiplegia [30%], diplegia [25%], quadriplegia [45%])
o Severity
- 70% associated disorders: visual problems, hearing impairment, communication disorders, epilepsy, intellectual disability,
specific learning disability or perceptual disorders

History
- Sorry for your loss. Do not worry, I can arrange for the immunization.
- Can you tell me more about his arching? Scissoring of his legs or crossing of legs while you hold him? Weakness or stiffness of
his limbs?

- BINDS: social smile (4mos)? Prone to supine (5mos)? Roll over (5mos) Babble (6mos)? Dada/baba (8mos)? Peek a boo
(8mos)? Vision? Hearing? Waterwork? Fever? Medical condition?
- How about your pregnancy? Was it a planned pregnancy? Did you have any infection (TORCH)? Was it a term or preterm
delivery? What about her delivery? Birth weight? About labor? Was the baby resuscitated? Complications? What about his
previous growth charts?
- Family history of any developmental disorder?

Physical examination
- General appearance
- Vital signs and growth charts
- Neurologic examination: increased tone and reflexes

Management
- Most likely John has a movement and postural disorder due to damage to an immature developing brain called cerebral palsy. It
is a group of non-progressing and permanent disorder and in his case, it most likely occurred due to the infection that you had in
pregnancy or because he was resuscitated in nursery after birth. To further confirm or diagnose his condition I will refer him to
60

neurologist pediatrician for complete neurologic and developmental examination so that we can come up with a definitive
diagnosis.
- Is it serious? Yes, it is serious and I am sorry for that. You are not alone and he will be taken cared of by a MDT:
o Specialist pediatrician
o GP
o Physiotherapist: practical advice to parents and carers about positioning, handling, and play
o Occupational therapist: developing child’s UL and self-care skills
o Speech pathologist: severe eating and drinking difficulties and communications and augmentative communication
systems
o Orthotists/prosthetists
o Dietitian
o Psychologist
o Social worker
- Any complication or health problem in the future will be dealt accordingly and I will give you the contacts of support groups and
CP clinic.
- Review. Reading materials.

Positional Plagiocephaly (Flat head)

Case: David, 9-months-old presents with his mom who is concerned about his baby’s head. She thinks that his head is oddly shaped.

Task
a. History (3months
b. Physical examination (HC growing along 25 centile)
c. Differential diagnosis
d. Management

Features
- Asymmetry of skull with normal head circumference
- “flat head”
- Most common cause of an abnormal head shape
- Congenital or acquired
- Results from infant sleeping in one position
- No impairment of cerebral development or intellect

- Treatment: changing side to which the child usually faces for sleeping then regularly changing sides and encouraging time in
prone position while awake cranial remodeling helmet (best from 4-8 months)

History
- HPI: since 3 months
- BINDS especially delivery and development
- FHx of abnormal head

Physical examination
- General appearance
- Vital signs and growth chart
- Neck for congenital torticollis
- Head for suture lines if rigid
- Dysmorphic features
- Developmental assessment
- Neurological examination

Differential Diagnosis
- Plagiocephaly or craniocynostosis (premature fusion of one or more sutures of cranial vault and base, which act as lines of
growth)
- Cranial XRay to see suture line whether patent or not (equal, symmetrical, closed at the same time) often unreliable in <3
months when low mineralization of skull is difficult to see
- Prompt referral to pediatric caraniofacial surgeon done at 5-10 months

Management
- Prevent further deformities
- Keep baby in supine position but give “tummy time” with mom observing
- Reposition baby during sleep and feeding
- Refer to craniofacial unit at RCH if failed conservative treatment up to 6 months; might use cranial remolding helmet
- External cranioplasty

ORTHOPEDICS

Pulled elbow
61

Case: You are working as an HMO in ED and a 3-year-old boy was brought by his mother. She is concerned because he is crying after
she pulled his arm while playing with him at home. She is anxious because she thinks she has fractured his arm.

Task
a. History
b. Physical examination
c. Diagnosis and manageemnt

Features
- Occurs in children under 8 years of age (2-5) when an adult supplies sudden traction to the child’s extended and pronated arm
head of radius pulled distally through the annular radioulnar ligament
- Signs and symptoms:
o Crying child
o Arms limp by side or supported in child’s lap
o Elbow is flexed slightly forearm pronated or held in mid-position
o Arm tender around elbow without bruising or deformity

Differential Diagnosis
- Fractured head of radius

History
- Is the child in pain? Give painkillers? Can you tell me more what about what happened? Is he moving the arm after that? Any
pain anywhere? Aggravating factor? Did you notice any swelling or bruising around the elbow? Deformity? Break in the skin? Is
this the first time? Any injury to other arm or any other part of the body? How is his general health?
- BINDS
- How is everything at home? Any financial problem or stress?

Physical examination
- General appearance
- Vital signs and growth chart
- Examination of arm: look at position, shoulder, wrist and elbow joints; palpation
o Inspection: deformity, ulceration, abrasions, bruising, redness and inflammation,
o Range of movements: flexion and supination causes pain

Diagnosis and Management


- Most likely your child has pulled elbow. It is not a fracture. The head of the radius has been pulled downward to the radioulnar
ligament and treatment is reduction by supination.
- Gain child’s confidence and give pain relievers
- Child stands facing doctor with parent holding non-affected arm
- Place one hand around child’s elbow to give support, pressing the thumb over the head of the radius
- With the other hand, firmly and smoothly flex the elbow and suddenly and firmly twist the forearm into full supination (+ painful
click) child will settle and resume full pain-free movements
- Recurrences possible up to 6 years
- Alternative: very gently alternate pronation and supination through a small arc as you flex the elbow
- Note: may have spontaneous resolution; send home in high sling
- If history not typical and cannot be reduced do xray to check for radial head fracture

Developmental Dysplasia of the hip

Case: You are in GP clinic. Mother brings her 8weeks old baby girl been referred to you by nurse who has found asymmetric hip crease.

Task
a. History from mom (Delivery: Breech, Feeding: Breast feeding, Immunization: Shots at 6 th week, Healthy looking, not crying)
b. Physical Examination findings (Height and Weight: on 60th percentile on Growth Chart, Left hip higher than the Right hip,
Movements: all fine, Ortolani and Barlow test (+)
c. Talk about management

History
- I can see from the notes you have been referred by the nurse. May I know what is going on? Do you have any concerns: eg any
difference between the two legs? Any problems while changing nappies? Does she move all her limps actively? Does she cry
when you move her leg or when you touch the area?
- Please tell me more about your pregnancy was it a full term pregnancy or prematurity? What was the mode of delivery (C-
section)? Why (Breech)? Any problems

during or after delivery? Did she stay in neonatal unit for a long period of time? Are you breast feeding the child? Any problems
with the feeding? Has she had her immunization?
62

- Any family history of anybody one had leg problem, limping or birth defects? Is this the first born in the family? Have you had any
concerns about her growth and development?

Physical Examination
- General Appearance: Is the child active and alert? Is she moving all 4 limbs? Pallor, jaundice, dehydration?
- Growth chart for Height, Weight, Head circumference.
- Legs: inspection: Preferably in a prong position looking for any abnormal position of legs. Any asymmetry between hip creases?
Any difference in the length of right and left leg? Any tenderness? Turn baby over:
o Barlows (Push head of the femoral back to the acetabulan when you hear a click. +ve) and
o Ortolani (fix right leg with hand than flex the left leg abducted to 90 degree is you hear a click +ve) maneuvers.

Investigations
- I would like to order U/S of both hip joints, later on we might need to do MRI or CTscan before a surgical procedure is under
taken.

Diagnosis and Management


- From history and examination and U/S seems like your child has a quit common condition called Developmental Dysplasia of the
hip. It’s a condition where the head of the thigh bone is not well formed/developed. Draw a picture. It doesn’t stay within the joint.
Usually the cause is related to mother’s hormones during pregnancy. As you know before delivery there’s secretion of certain
hormones that relax the pelvic ligament to help with the delivery. These hormones travel thru the blood reaching the baby’s
ligaments making them loose as well these ligaments they do not support the head of the thigh bone as they should. Also baby’s
position in the uterus can affect this defect eg a breech baby. This condition is more commonly seen in girls usually the first born
in the family. The severity of the disease varies in different babies. In some cases it can affect both joints this condition needs to
be treated as soon as possible to prevent permanent complication eg Shortening of leg leading to limping later on in life,
Osteoarthritis of the joint, Acetabular roof dysplasia(sometimes the pelvic part of the joint becomes affected as well) I would refer
you to the paediatric orthopaedic surgeon. The treatment depends on the severity of the disease as well as the age of the baby.
Usually they will put a special splint around the baby called Pavlik Harness (<6months). The purpose is to keep the head of thigh
bone within the joint in a neutral position until the bone matures. She will need to wear it for a few months. However if it doesn’t
improve the surgeon might decide to do either a close reduction followed by a hip spica (6-18months) from the knee to the waist
or open reduction (>18months) and reconstruction of the joint. Prognosis is generally good if treatment is started at the right time.
Your child will be frequently follow up with the specialist with repeated U/S and XRay (xray if child >9months) at every visit.

Irritable Hip/Transient Synovitis

Case: Sammy is a 4-year-old boy brought in by his father Dan. He has been limping for the last 2 days and today, he was unable to get up
from the bed. His parents are concerned because Sammy is usually very active.

Task
a. History (knee pain, refuses to bear weight, no trauma, had URTI last week)
b. Physical examination (no pallor, dehydration, jaundice, refusing to walk around your clinic, VS normal, no tenderness or
deformity,
c. Diagnosis and management

Limping cases in AMC: trauma, septic arthritis/osteomyelitis, congenital hip dislocation

Features
- Known as irritable hip
- Consequence of self-limiting synovial inflammation
- Aged 3-8 years
- Sudden onset of hip pain and a limp
- Can usually walk but with pain
- May be history of recent URTI or viral illness
- Painful limitation of movements especially abduction and rotation
- Xrays: normal may be soft tissue swelling, FBE normal, ESR mildly elevated; USD fluid in joint
- Outcome: settles to normal within 7 days without sequelae
- Treatment:
o Refer early (orthopedics)
o Bed rest
o Use of crutches and analgesics
o Follow up xray is needed 4-6 months to exclude Perthe’s disease
o Aspiration under general anesthetic may be needed to exclude septic arthritis

Differential Diagnosis
- Septic arthritis
- Perthes disease
- JCA
- Osteomyelitis
- Trauma
63

DDH TS Perthes’ SCFE Septic


arthritis
Age 0-4 4-8 4-8 10-15 Any
Limp + + + + Won’t
walk
Pain - + + + +++
Limited AB All esp. AB & IR All esp. All
movement AB & IR IR
Xray No Normal Sub- AP may Normal
value in chondral be Use
neonatal fracture normal; USD
period Dense Frog
head lateral
Pebble view
stone shows
epiphysis slip

Osgood Schlatter Disease (Tibial Tubercle Epiphysitis)

Case: Your next patient in GP practice is a 12-year-old boy who presents with pain on the right knee for the last few months when he plays
footy.

Case 2: Jason aged 15 years attends to your surgery with pain in his left leg for few weeks and finds it hard to play footy. He is captain of
his school team and due to his absence, the team had lost few matches. He denies any history of trauma, fall or injury. Pain started about
two months ago and is now severe enough to prevent him to participate in games.

a. History (knee pain for last 3 months before and after high jump, + worse while playing footie; pain on kneeling down; no hx of
URTI or trauma)
b. Physical examination (no signs of trauma, + swelling and tenderness below R knee in tibial tuberosity, no temperature difference,
joint not hot, power, ROM normal; hip and ankle normal)
c. Diagnosis and Management

Features
- Occur in adolescent age group (10-14 years)
- Usually among athletic adolescents
- Most common cause of limping
- (+)hip/knee pain after exercise
- (+) tenderness and swelling on the tibial tuberosity
- Self-limiting; usually resolves in 12 months
- Treatment: limited exercise; RICE; sometimes special strapping

Differential Diagnosis
- Osgood Schlatter
- Fracture
- Meniscal injuries
- Tumor
- SCFE

History
- For how long have you had this pain? Did you hurt yourself? Where exactly is the pain? Can you point it out with one finger?
Does it go anywhere else? Any swelling or redness over the area? Any bruise that you can see? How bad is his pain? Does it
get worse with movement? Does it get better with rest? Is this the first episode? Is it affecting your sleep or activity? Are you able
to play or walk at all? Any history of fever, recent viral illness, or cough? Are you eating and drinking well?
- Is there a family history of joint problems or arthritis? Have you noticed a skin rash along with the pain (HSP)?
- How’s your general health? Any previous medical or surgical conditions? Is your immunization up to date? SADMA?

Physical Examination
- General appearance
- Vital signs and growth chart
- Examination of BOTH knees
o Compare and measure the length of both
o Inspection: signs of trauma, bruises, swelling, insect bites, atrophy of thigh muscles, deformity, position of supine leg
o Palpation: joint lines, temperature, tenderness on tibial tuberosities. Check ankle, and hip as well
o ROM
o Sensation, power (decreased in quadriceps because of pain) and reflexes
64

Diagnosis and Management


- From history and examination, you are suffering from a condition called Osgood Schlatter but we need to do an xray of your knee
before we can confirm that.
- Xray: widening of epiphyses with bone fragmentation, no fracture or tumor.
- In adolescence, bone grows faster than the tendon such that tendon pulls on the tibial tuberosity. For example, if you put a nail
on the wall and attach a string and pull it, the string pulls on the nail. This results in pain especially after play/movement.
- It is self-limiting and not a serious condition. It will usually resolve in 12 months. However, the knee may remain uncomfortable
for around 2-3 years until the growth spurt finishes. There is continuous traction as a result of increased work of quadriceps while
playing footie resulting in inflammation of the tibial tubercle where the tendon of the thigh muscle inserts.
- The most important treatment is to rest the knee so that it heals. Apply ice packs. Once the swelling decreases, I will refer you to
the physiotherapist who will teach you stretching and strengthening exercises for your quadriceps.
- Rest for 2-3 months and slowly start your physical activities. Usually, healing will take about 3-6 months but can go up to 1 year.
Please avoid the trigger activities. After 12 months you should be fine and you can start playing footie again.
- Do I need surgery? No. At least not at this point.
- Review. Referral to physiotherapy.
- Refer to orthopedics if with avulsion.

Leg Perthes Disease (Avascular Necrosis of the Femoral Head)

Case: You are a GP and a 6-year-old child was brought in by mom because he has been limping and unable to bear weight on the left side

Task
a. History (fell down 2 days ago and at night started complaining of pain; runny nose few days ago with fever; irritable and with poor
appetite)
b. Physical examination (healthy-looking boy, irritable and in pain, afebrile, patient lying supine holding his leg on flexed position
and externally rotated; painful; no obvious swelling or point tenderness)
c. Investigation (FBE slightly elevated WBC, USG not available)
d. Diagnosis and management

Clinical features:
- 4-8 years old
- M>F
- Pain + limping (groin, hip or knee)
- Could be preceded by URTI
- Ex: decreased movement of hip joint in all directions especially INTERNAL ROTATION and ABDUCTION
- 20% bilateral
- Investigation
o Xray – femoral head is flattened with fragmented appearance
Pebblestone epiphysis
o Bone scan – useful at early age
- Treatment:
o Refer to ortho
o Stop weight bearing!
o none to prolonged bed rest wheel chair, specialized ortho support
- Complication: osteoarthritis, permanent deformity and disfigurement, shortening of leg

History
- For how long has your child been limping? Did he hurt himself? Where exactly is the pain? Can you point it out with one finger?
Does it go anywhere else? Any swelling or redness over the area? Any bruise that you can see? How bad is his pain? Does it
get worse with movement? Does it get better with rest? Is this the first episode? Is it affecting his sleep? Is he able to play or
walk at all? Any history of fever, recent viral illness, cough? Is he eating and drinking well? Is there a family history of joint
problems or arthritis? Have you noticed a skin rash along with the pain (HSP)? Who takes care of your child most of the time?
Does he go to school? Which grade? Any problems at school? How do you consider his height and weight to be? How’s his
general health? Any previous medical or surgical conditions? Is his immunization up to date? Any concern about his growth and
development? Is anyone smoking at home (passive smoking risk factor for avascular necrosis)?

Physical examination
- General appearance
- Growth chart for height and weight
- Vital signs
- Chest and heart
- Abdomen
- Lower limbs: compare and measure the length of both limbs; look for any atrophy of the thigh muscles; obvious deformity,
swelling, bruising over both hip joint, position of the leg in supine position; is it possible for me to check the child’s gait? Palpate
for any tenderness over the joint line; ROM of movement of the hip, knee and ankle joint; ROLL test (patient on supine position
with one hand resting over the thigh, with other hand, move the hip internally and abduct (+) if patient exhibits guarding or
spasm of thigh muscles

Differential Diagnosis
65

- Septic arthritis
- Osteomyelitis
- Perthes disease
- Fracture
- Slipped capital femoral epiphysis
- HSP
- Malignancies of the bone
- Juvenile rheumatoid arthritis

Investigations: FBE, ESR/CRP, Right and Left Limb xrays with Frog leg view (distortion of the contour of the head of the femur)

Diagnosis and management


- From history, PE and xrays provided to me, your child has a condition called Perthes disease also known as avascular necrosis
of the head of the femur. It is not an uncommon condition, more frequently seen in males. Because of unknown reasons, the
blood supply to the head of the thigh bone is reduced. Sometimes, it is associated with previous history of infection or trauma. It
is good that we have caught it at an early stage to prevent the risk of complications (shortening of leg, permanent deformity and
disfigurement, osteoarthritis). Sometimes other joints of the body may be involved (shoulder joint and opposite leg).
- We need to admit the child. Call in the pediatric registrar. They might decide to do further testing CT/MRI/bone scan to determine
how much damage

has been done. Your child needs strict bed rest. Don’t allow him to put weight especially on the affected leg. Provide him with
crutches. We will give him painkillers.
- Depending on the severity of the disease, the specialist might decide to put specialized orthopedic support for him. Sometimes,
this treatment needs to be continued for an extended period of time (2 years). This is important because your child is in a
growing phase and we want to prevent any abnormal, permanent positioning of the head of thigh bone.

Irritable Hip or Transient Tenosynovitis


- 3-8 years old
- History of URTI
- Complains of pain and tenderness of the calves; can arise to stand but refuses to walk; child able to walk but with limping/pain
- Child well and afebrile
- Commonest presentation to ED with lumping and pain
- Examination: pain on INTERNAL ROTATION only
- Investigations:
o Xray: normal
o USG: fluid in synovial cavity
- Diagnosis of exclusion
- Treatment:
o Rest + analgesia
o Admit if with ADHD
o Short-term steroid if no resolution
o Settles within 7 days without sequelae
o Repeat xray in 4-6 months to exclude perthes disease

Slipped Capital Femoral Epiphysis (SCFE)

Case: A 12-year-old boy is brought by his father to your GP clinic complaining of pain on his left knee and limping. He is a basketball
player. Pain started 1 week ago on his left knee while he was playing and started limping after. His movements are restricted with radiation
to thigh and groin.

Task
a. History
b. Physical examination
c. Diagnosis and management

Differential Diagnosis for a child with a limp:


a. SCFE
b. Osgood Schlatter Disease
c. Septic arthritis
d. Osteomyelitis
e. Trauma
f. SCFE
g. Perthes disease (4-8) – avascular necrosis of femoral head
h. Transient synovitis (4-8) – limping child with history of URTI but not a sick-looking with child; fluid in the joint and some soft tissue
inflammation; REST!
i. Developmental dysplasia of the hip
j. Henoch-Schonlein purpura (arthralgia, palpable purpura, nephritis, abdominal pain)
k. Adductor tendinitis

Features
- Age: 10-15 years
66

- Obesity (above 90th percentile)


- Clinical features:
o Limp + hip/knee pin
o Hip is flexed, short and externally rotated at hip

o Every movement restricted at hip especially INTERNAL ROTATION


o Bilateral in 20%
- Diagnostics: adolescent with a limp or knee pain should have xray (AP and frog view) of both hips

- Management:
o Stop weight bearing
o Refer to orthopedics
o If acute: gentle reduction (traction) pinning

History:
- Age is very important to narrow diagnosis
- Increased weight (chubby boy); no trauma; with radiation to the thigh and groin;
- Site? Onset/origin? Radiation? Type? Severity/progressive/increasing or decreasing in severity? Is he able to bear weight and
walk? Aggravating factors? Relieving factors? Associated factors? Is it for the first time (can be recurrent)? Have you taken any
medications for it? Unilateral or bilateral (bilateral in 20% of cases)?
- Fever? History of URTI? History of trauma? Able to bear weight? Crack in the skin? (In osteomyelitis – point tenderness on
bone)? Abdominal pain? Rash? Arthralgia? Past history of joint problems? Any previous family history of joint problems?
- BINDS

Physical Examination:
- General appearance: well patient; BMI 90%
- Vital signs: especially temperature
- Joints/examination (joint above and below): hip, knee and ankle: LOOK, FEEL and MOVE
o Inspection: redness? Swelling? Scars? Deformities? Rash? Hip is lying in external rotation
o Feel: for any increase in temperature and tenderness, swelling, LAD?
o Range of motion: all movements are limited especially abduction and internal rotation; knee and ankle joint normal

Investigations:
- Xray AP and frog lateral view of BOTH hips and knee
o AP view
o Frog lateral view: displaced head of the femur

Management:
- Most likely from history, PE and investigation, he is having slipped capital femoral epiphysis. Did you hear about this condition
before? In this condition, the weakness of the growth plate causes the head of the thigh bone to slip from its place at the upper
groin end of the bone.
- If left untreated, it can restrict the blood supply to the growth plate and thigh bone that can lead to death of the head of the thigh
bone and it is called avascular

necrosis of the femoral head. That’s why he shouldn’t bear weight and be on strict bed rest. He needs urgent referral to the
orthopedic specialist to review his condition. I can provide crutches if needed.
- The specialist might consider surgery to prevent further slipping and can do pinning to stabilize the bone and almost all cases
require surgery. The outcome is usually good with treatment. I will be in contact with the specialist and follow him up in the future.
- Advise him to lose weight in the future. This may help in prevent recurrence. I will give you some reading materials.
- How soon to visit the specialist? ASAP.
- Are you sure about your diagnosis? Yes. Any problem on the knee? NO.
- The other complication from this condition is osteoarthritis of the hip joint if left untreated.

Osteomyelitis

Case: You are an HMO in ED of a rural hospital and a 2-year-old girl was brought in by parent because the child refuses to stand up since
yesterday and she cries when you try to make her stand with support.
67

Task
a. History (irritable, crying a lot, did not walk the whole day, doesn’t want to touch the right leg especially the knee)
b. Examination (irritable, crying, lying on bed with right leg is flexed, 38.9, PR high, child not letting you touch the leg, painful with
minimal movement, tenderness over the knee at the tibial tubercle, swelling, redness, and tender).
c. Differential diagnosis to examiner
d. Management

Differential Diagnosis
- Osteomyelitis
- Septic arthritis
- Tibial tubercle epiphysitis
- Perthes Disease
- Fractures
- Trauma
- Tumor
- Child abuse
- Cellulitis
- Scurvy

OSTEOMYELITIS SEPTIC ARTHRITIS


Subacute onset Acute onset

Limping, decreased weight Limping, decreased weight


bearing, refuses to use limb bearing, refuses to use limb

Localized pain and tenderness Pain on movement of joint and


may or may not be present at
rest

+ Fever Almost always with fever

+ Soft tissue swelling and Joint tissue is almost always


redness swollen and red

History
- Please tell me more about her problem? Since when did she refuse to stand up? Do you think she has pain in her leg? Which
side? Is she able to walk at all? Is she bearing weight at all? Can you point out where exactly the pain is? Is there any redness or
swelling over the area? Does it feel hot to touch? Do you think it is getting worse? Does she have fever? Since when? How high
is it? Does she have lethargy? Fatigue? Is she eating and drinking well? Is she passing water normally? Any change in her bowel
habits? Do you remember if she had injury to this area recently? Did she fall down? Did she suffer from a recent viral infection?
Any other medical or surgical conditions? Any concerns about her growth and development? Immunization? SADMA? Does she
go to childcare? Who takes care of her? Problems? Stress?

Physical Examination
- General appearance: LOC, pallor, dehydration, jaundice
- Vital signs and growth chart
- Lower limb: look for any obvious redness, swelling, deformity, trauma. Observe the child’s posture; any tenderness of the bone;
feel for warmth over the area; palpate for joint effusion, crepitus, check the joint above and below looking for any evidence of
infection
- Am I able to observe the gait of the child? Is the patient able to bear weight or not?
- Urine dipstick

Investigation
- I would like to order some investigations including FBE, ESR, CRP, blood culture, urine microscopy and culture, anteroposterior
and lateral Xray of the limb, USD (in cases of joint problems); best test: bone scan
- Pathogens: staphylococcus aureus (MCC), streptococcus pneumonia, streptococcus pyogenes; fungal if immunocompromised;
tennis shoe prick (pseudomonas); sickle cell anemia (salmonella)

Diagnosis and Management


- At the moment, it seems like your child might be suffering from a condition called osteomyelitis where an infection occurs within
the bone. This infection usually arises either from the blood where the patient has a recent history of URTI. Sometimes, the
infection enters after direct trauma to that area. It usually affects the long bones in the body, especially at the ends of the long
bones because of poor blood supply to that area.
68

- The usual symptoms are pain, swelling, redness, deformity, fever, and limping.
- We need to admit your child to prevent any complications which might arise if we don’t treat it early. Complications include
pathological fractures, chronic/recurent osteomyelitis, septic arthritis, bone deformity, sinus formation, abscess formation
(brodie’s), sepsis.
- I will call in the orthopedic registrar to come and see the child. They will review all the investigations. If required, they might
aspirate the joint to rule out any concomitant infection of the joint.
- We need to give some IV antibiotics – usually flucloxacillin – that needs to be continued for the next 3 weeks followed by oral
antibiotics (if >5 flucloxacillin x 1 week then oral for a total of 4 weeks).

- He needs to take rest until all symptoms are gone. We will give him pain killers, antipyretics. We might need to splint the area to
prevent discomfort and deformity.

Septic arthritis

Case: You are the HMO at a hospital. A father brought his 3-year-old daughter complaining of fever for 1-2 days. She’s not walking for few
days, ill-looking, pain on her right knee, not eating.

Task
a. History
b. Physical examination
c. Diagnosis and management

Differential diagnosis
- Osteomyelitis
- Septic arthritis
- Trauma
- Transient synovitis
- Bleeding disorders (hemarthrosis)
- Henoch schonlein purpura

History:
- How long has she been limping? Is it sudden or gradual? Fever? Chills? How long has she been having fever? Any swelling?
Severity of pain (SORTSARA)? Any history of URTI during the previous days? Any bleeding disorders? Any rash? Waterworks?
Bowel?
- BINDS?

Physical examination:
- General: daughter is clinging to father, irritable, and looks ill; rash? LAD?
- Vital signs: temperature: 39, RR, PR, BP normal
- ENT, lungs, cardiac, and abdomen
- Knee examination:
o Inspection: R knee swollen, red; any signs of injury? Discharge? Any pathology on the other side? Anything on the
ankle or hip?
o Palpation: warm and tender to touch; point tenderness???
o Gait?
- Remember: FEVER + RASH + LN

Diagnosis and Management:


- Your child is suffering from a condition called septic arthritis. Do you know anything about this? I would like to admit your child,
take blood for FBE, ESR/CRP, and culture/sensitivity. Secure IV line. We have to start antibiotics (IV flucloxacillin) because the
most likely organism is staph aureus. Give analgesia for the pain.
- I would organize pediatric orthopedic referral to review your case and they may consider doing aspiration of the knee joint and
plain Xray or (ultrasound/bone scan if not sure about diagnosis).
- Reassure! But it can be serious this is why we have to admit and treat him as soon as possible.
- Septic arthritis – give 3-5 days of IV antibiotics and 2 weeks of oral;
- Osteomyelitis – 2 weeks IV antibiotics and 2-3 weeks of oral

Short Stature (Constitutional Delay)

Case: An 8 year old child was brought to your GP clinic by her mother because she is concerned about her height as she is the shortest in
her class. Otherwise, she is well and healthy.
69

Task
a. History (38 weeks, well-developed, no developmental delay, height 165-70)
b. Physical examination (Height above 3rd percentile, weight 15th percentile)
c. Diagnosis and Differential diagnosis
d. Talk to the mother

History
- When exactly did you notice that she is short? I would like to inquire more about your pregnancy with this child. Were there any
problems to yourself or any abnormality in baby? Is it a planned pregnancy? Is she your only child? Did you have any previous
pregnancies or miscarriages? What was GA of delivery? Any complications during labor? Any problem such as cord prolapse or
prolonged delivery? Any abnormal CTG or low oxygen level (hypoxia leads to hypothalamic, hypopituitary problems)? After
labor, any detected abnormality during the first checkup (Apgar score? Hypoglycemia (leads to reduced growth hormone)? Heel-
prick test? CDH? COA)? Resuscitation? What was the BW? Height?
- Is she developing well since then (milestones)? Since birth, were there any detected problems with her general health or chronic
disease problem (celiac disease, crohn disease, chronic kidney disease, asthma)? What is your height? Your husband? What is
your social background? Any detected familial trend of short stature? Anyone with grandparents who are short? Any similar
sibling problems? What is the age of puberty for both parents? Do you or your husband have chronic problems? Does your child
receive any treatment with steroids? Is your child using any medications? How would you describe her performance in school?
How will you describe the intelligence of your child? Were you able to detect any abnormal features? How is this affecting her life
in general?

Physical examination
- General appearance: stigmata of turner syndrome or any other chronic disease; dysmorphic features
- Anthropometry: Growth chart for weight and height, GV, upper flow segment ratio, compare with percentile charts
- Ask for height and check if proportionate or disproportionate (ratio of upper segment and lower segment)
- Secondary sexual characteristics
- Visual acuity and fields (pituitary tumors)

Investigation
- FBE, U&E, LFT, TFTs, ESR, RFTs, celiac disease tests, chromosomes in all girls (regardless of appearance; karyotype Turner
45O if GV <25 percentile for bone age
- Xray of Left wrist
- Growth hormone studies – 1 exercise test and glucagon stimulation test

Differential Diagnosis
- Constitutional delay (BA<CA)
- Endocrine: Hypothyroidism, Panhypopituitarism, Cushing Syndrome
- Genetic: Turner Syndrome
- Chronic Infections (Asthma)
- Chronic disease (Coeliac disease, Cystic fibrosis)
- Familial short stature (BA=CA)

Management:
- Reassurance
- Referral for turner syndrome karyotyping
- Parents should be considerate and should not criticize because height is not the only thing that matters
- Review. Referral for further assessment.

PULMONOLOGY

Bronchiolitis

Case: Sara age 8 months is brought to your GP clinic by her Mom Margaret. She had cold for couple of days with fever and runny nose.
Today she is woken up with cough and rapid breathing. Yesterday, she didn’t eat as well as she does usually. She refused to eat breakfast
this morning and she only drank half a bottle of formula milk. She had normal number of wet nappies. Sara’s brother who is three years of
age had cold for last one week and his asthma played up needing salbutamol regularly.

Task
a. Further focused history
b. Further examination and investigation
c. Probable diagnosis and treatment advise

Features

General appearance: pallor, jaundice, dehydration, BMI/growth chart


- Usually starts with cough and coryza then becomes more severe after 3-5 days.
- Wheezy breathing, tachypnea, hyperinflated barrel chest with subcostal retractions
70

- Auscultation: widespread inspiratory crackles with expiratory wheezes


- Xray: hyperinflation

Risk factors for severe bronchiolitis


- Young (<6 weeks)
- Extremely premature infants
- CHD
- Neurologic conditions
- Chronic respiratory illness
- Pulmonary hypertension

Assessment of severity
Mild Moderate Severe
Behavior N Some/ Irritable;
intermittent lethargy;
fatigue
RR N Increased; Marked
tracheal tug increase or
Nasal flaring decrease
Tracheal tug
Nasal flaring
Accessory None or Moderate Marked chest
muscle use minimal chest wall wall retraction
retraction
Feeding N May have Reluctant;
difficulty unable to feed
feeding or
reduced
feeding
Oxygen None required Mild Hypoxemia;
SaO2 >93% hypoxemia may not be
corrected by corrected by
oxygen oxygen
(SaO2 90- (SaO2<90%)
93%)
Apneic None May have brief Frequent or
episodes episodes prolonged

Management: Bronchiolitis is an infection caused by a virus known as RSV. It is common among infants between 2 weeks and 9 months
(up to 12 months);
- Mild: treat as OP; hydrate and oxygenate (small frequent feeding)
- Admit to hospital because of risk of dehydration and hypoxia
- Observe: color, pulse, respiration, oxygen saturation
- Oxygen: by nasal prongs to maintain O2 above 90%
- Fluids IV preferably or by NGT if unable to feed orally
- Antibiotics – not indicated unless with secondary infection
- No evidence for use of nebulized adrenaline, bronchodilators or corticosteroids
- Increased incidence of asthma
- Advise that coughing can continue up to 4 weeks (post-bronchiolitis cough)

Pertussis (Whooping Cough)

Case: You are an intern in the ED. A 10 months-old-boy was brought in by mom complaining of cough.
71

Task
a. History - cough and fever x 5 days with vomiting after cough; not feeling well with decreased nappies;
b. Physical examination- irritable, dehydrated and coughing; not in obvious distress;
c. Management

Case 2: 4-months-old baby coughing for the last 2 weeks (cough cough cough cough vomit); mom is also sick; no cyanosis; lungs clear;

Case 3: Stephanie brings her 3-year-old daughter Sally to your GP clinic with history of persistent cough for the last 2 weeks. Stephanie is
concerned because last night Sally could not stop coughing. The cough started with cold, runny nose and mild fever. She was off her food
for 1-2 days but recovered well

except for the cough. Sally goes to kinder twice a week and rest of the days Stephanie looks after her. She is otherwise healthy and is not
on any regular medications.

Task
a. Further history (dry hacking cough, occurs in bouts, and continuous, SOB, when cough settles looks alright, no smoker at home,
no history of atopy or asthma, + symptoms at childcare, bowel works and waterworks normal, FT, immunization up-to-date,
normal growth and development, no problems socially)
b. Physical examination and investigation findings from examiner (comfortable, pallor, dehydration, jaundice, VS T37, HR 96, RR:
18, Oxygen 98%; chest is symmetric, chest is clear and no added sounds and equally reactive, heart and abdomen normal;
growth chart 50 centile; FBE increased in lymphocyte count, CRP 15, CXR normal; nasopharyngeal aspirate for PCR)
c. Diagnosis and management

Differential Diagnosis
- Pertussis
- Croup
- Bronchitis
- Postviral cough

History
- May I know when it started? Can you describe the nature of the cough? Does he vomit in the end? Does he produce sound at
the end or turns blue? Do you think his breathing is noisy? Are there fits? Do you think he is short of breath? Any fever? Since
when? Is he eating or drinking well? Any change in his bowel habits? Change in number of wet nappies? How is his general
health? PMHx? Immunization? Any problems at birth? Did he require a long resuscitation? Is this the first time for him to develop
this condition? Whom does he live with at home? Anybody else sick in the family? Does he go to childcare? Any chance of
exposure to a sick person in childcare? Anyone smoking at home? Is he on any medications?

Examination
- General appearance: dehydration? Pallor? Cyanosis? Any obvious respiratory distress? Can I observe the pattern of cough?
- VS: including oxygen saturation
- ENT: for inflammation and LAD
- Chest examination preferably when child is not crying; auscultate for crackles, wheezes,
- Palpate abdomen
- urine dipstick

Management
- Most likely your child has a condition called whooping cough or pertussis. It is an infection of the airways caused by a bug
(Bordetella pertussis). It's usually acquired by droplets. Most probably his dad is suffering from it as well. We can do some tests
for him as well.
- At the moment we need to admit him because: a. his diet is not well, b. we need to do some tests as well (nasopharyngeal
aspirates to look for lymphocytes and FBE for lymphocytosis), and c. give him some medications
- If child is not feeding well, then put NGT
- We need to start him on antibiotics, preferably clarithromycin for 7 days

- All those who came into contact with the child during the past week should also get clarithromycin (azithromycin 5d/,
erythromycin 7d/cotrimoxazole – if allergic).
- Inform DHS as pertussis is a highly infectious disease
- You also have to inform childcare
- Once antibiotics are started, he can go to childcare or other public areas because his secretions are already free from bugs
- We will keep him in the hospital for a few hours only until he is well rehydrated
- Complications: asphyxia, hypoxia, apnea, hernia, cerebral hemorrhages, convulsions, rupture of diaphragm, otitis media,
pneumonia, conjunctival hemorrhages, pneumothorax
- Please understand that he may have this cough for a long period of time. Also known as "100 day cough"

RCH
72

- Dry cough and nasal discharge approximately 1 week, followed by a more pronounced cough occur in spells or paroxysms and
vomiting
- If <6mos: admit (greatest risk of complications: apnea, severe pneumonia, encephalopathy and death)
- Immunization if unimmunized
- Treatment of contacts: infants <12 mos of age without 3 documented doses of vaccine, unvaccinated or partially vaccinated
person with chronic CR illness and any woman in the last month of pregnancy

Croup (Laryngotracheobronchitis)

Case: You are in a GP clinic and your next patient is 9-month-old John who was brought by his father saying that he’s having noisy
breathing.

Task
a. History (started night PTC noisy and harsh, worse when crying, runny nose 2 days back, decreased appetite)
b. Examination findings (conscious, harsh inspiratory stridor, vital signs accessory muscle use, T: 38, RR:50, HR:110 saturation
93%, stuffy nose, subcostal retraction, bronchial (transmitted) sounds, inspiratory stridor)
c. Diagnosis and management

Differential Diagnosis
- Croup
- Epiglotittis
- Bacterial Tracheitis
- Foreign Body
- Bronchiolitis
- Asthma
- Anaphylaxis

Clinical features:
- Respiratory syncytial virus
- Clinical features: barking cough, inspiratory stridor, fever, but no signs of toxicity
- Risk factors for severe croup:
o Down syndrome
o Sublottic stensosis (congenital or secondary to prolonged neonatal ventilation)

Features Mild Moderate Severe


Stridor + Exertion/ At rest
crying/ playing
Barking cough + + +
RR Normal Tachypnea Marked
tracheal tug Tachypnea
Nasal flaring tracheal tug
Nasal flaring
Behavior alert Irritable drowsy
Work of None/minimal Moderate Severe
breathing retraction retraction
Oxygen None None Hypoxemia
(late sign)
Treatment Home
Steroids:
prednisolone

- Mild: May happen again Give steroids (one dose) worse or Stridor at rest hospital
- Moderate: hospital calm him oxygen and steroid
o Mild-moderate: prednisolone 1mg/kg then 2nd dose next evening or oral dexamethasone 0.15mg/kg SD
- Severe: admit
o Severe: nebulize with adrenaline (1ml of 1% adrenaline diluted to 4 ml w/ NSS or 5ml adrenalin 1:1000) + 0.6mg/kg
(max 12mg) IM/IV dexamethasone

Indications for admission


- Drowsy child or decreased LOC
- Not feeding well
- Respiratory distress
- Stridor at rest
- Apnea
73

- Oxygen saturation <93%


- Age <6mos
- Tracheal tug
- If mother can’t cope

History
- Is my patient hemodynamically stable?
- Can you describe more about the noisy breathing? Is it the first episode? Is it continuous? Is there a certain position that relieves
the noisy breathing? What are the aggravating factors? Do you think he has swallowed anything? Are there other symptoms
such as cough, fever, runny nose, drooling of saliva? Is he turning blue? Is he working hard to breathe? Vomiting? Any decrease
in number of nappies? Any change in color of urine? Does he cry when he pee? Does he have rash in the body?
- BINDS (smoking? Similar condition in childcare/sibling/school)
- FHx of allergies, atopy or eczema

Physical examination
- General appearance and level of consciousness, decubitus, drooling, stridor at rest, sign of respiratory distress, ill-looking,
dehydration
- Vital signs and growth chart
- ENT (swollen tongue)
- Neck: Lymphadenopathy
- Chest: accessory muscle use, subcostal recession, tracheal tug, asymmetry in breathing, auscultation for added sounds, air
entry, wheezes

Diagnosis and Management


- Most likely your child is suffering from a condition called croup. Have you heard about it? It is a viral infection of the upper airway
caused by parainfluenza virus. It affects the larynx, trachea, and upper part of the bronchus, producing inflammation and swelling
that makes it narrow causing noisy breathing. It is a common condition in children aged 6mos to 3 years.
- Is it risky? If left untreated, it can be risky but if treated it has good prognosis. It is a self-limiting condition.
- Are you comfortable to take care of the baby at home? You can take the baby home and I will give him steroids. If it persists, you
can give him another dose of steroid at home but if he develops noisy breathing at rest or if he becomes drowsy and is not
feeding properly, then please go to hospital right away.
- For moderate to severe: I will need to call ambulance to arrange for his admission. He needs to have oxygen and if severe
enough, he needs to be nebulized with adrenaline.

Primary Tuberculosis

Case: You are a GP and an 18-month-old boy born in Australia. The parents are of Asian descent and visit their country frequently. The
baby has fever, chronic cough and repeated chest infections. You ordered a CXR which showed whitish, shiny opacities in the lower lobe
of right lung/opacities in hilar LN on both sides. PPD Mantoux test is 20mm

Task
a. Explain XRay result
b. Further management

History
- Did the child have BCG in the past? Did you have any similar symptoms before or in the past? Did he have contact with a person
with active tuberculosis?

Counseling
- From history and examination findings, the child has primary tuberculosis. The PPD Mantoux test is a screening test for
tuberculosis. Usually, if it is >10mm it is positive for latent or active tuberculosis. It is a communicable infection caused by a bug
called Mycobacterium tuberculosis. It spreads from person to person through air when somebody with active TB coughs, laughs
or sneezes. It typically affects the lungs but can spread to any part of the body.
- There are two types: active and latent. I need to send your child to the hospital/TB clinic where he will be seen by the infectious
disease specialist who will order further investigations and start the treatment. They will do sputum for AFB staining, culture that
takes 6-8 weeks and Quantiferon GOLD assay (simple and results within 24 hours). Consider HIV screening after consent or LN
biopsy if indicated.
- Antimicrobial treatment with 3 or 4 drugs (Isoniazid, pyrazinamide, ethambutol, and rifampicins) daily for 2 months then isoniazid
and rifampicin for 4 months. The drugs can be given daily or 3x per week.
- He will be isolated for the first 2 weeks of the treatment although children with TB rarely transmit the infection. Compliance with
the treatment is very important because failure to comply might lead to development of drug resistance.
74

- TB can be cured in almost all cases by taking the medications for the full course of treatment as prescribed by the specialist. The
side effects include loss of appetite, stomach upset, extremely low chance of liver toxicity with isoniazid, orange discoloration of
urine, sweat, or saliva that is harmless with rifampicin, and eye problems with ethambutol. Others include itchiness and skin rash.
- It is a notifiable disease. The contacts also needed to be traced and screened so you will also be further investigated.
- Where did he get this infection? Overseas travel and contact with an infected person is a possibility.
- Are medications safe for my son? Yes. They will be prescribed in safe doses and he will be frequently reviewed for effectiveness
and side effects.
- What about recurrence? Will it come back again? It is rare. However, with re-exposure and low immune system, it is possible.

Additional features:
- Transmission less likely in children because:
o Usually have no cavitation
o Low bacterial load
o Less post-tussive force
- Contacts: screened with PPD and CXR;
- Latent tuberculosis: prophylaxis x 6 months; add pyridoxine if immunosuppressed/immunocompromised
- Notify DHHS

Obstructive Sleep Apnea

Case: A mother brought her 3-year-old son to your GP clinic because of snoring and mouth breathing. He is known to you because you
have treated him for recurrent otitis media for the last 12 months. He also had flu-like symptoms 3 days ago.

Task
a. History (child is snoring and mouth breathing at night; tiredness during the day; concerned that he is not putting on weight,
snoring; has problem with hearing)
b. Physical examination (growth chart normal, clear discharge on the nose, no polyps, no deviated nasal septum, + discharge in
ear, tympanic membrane is opaque, + enlarged tonsils)
c. Diagnosis and management

History
- Can you tell me a bit more about it? Since when? Did you notice any pauses in breathing/stop breathing? Daytime sleepiness?
Lethargic or tired? Playful or active? Headaches? Restlessness? Sweating?
- Otitis media? How frequent? Any treatment given? Ear pain? One or both ears? Discharge? Offensive or purulent? Bleeding?
Hearing problems? Speech problems/delay? Learning difficulties? Behavioral problems? How is it affecting school performance?
Sore throat? URTI in a year (6-12/year)? Any allergy? Coughing or sneezing? Eye irritation? Weight of the child?
- BINDS? Anyone smoking at home? Any previous surgeries (esp repair of cleft lip/palate)? FHx of obstructive sleep apnea or
sleep disorders?

Physical Examination
- General appearance and craniofacial abnormalities
- Vital signs
- Growth chart
- ENT: perforation, discharge, nose (polyps/discharge/swollen turbinates) and throat (enlargement of tonsils)

Management
- From history and examination, most likely your child is having OSA due to enlargement of the tonsils at the back of the throat. It
is a condition in which children have breathing difficulties when they are sleeping. When children sleep, their muscles are relaxed
which can lead to floppy upper airway which can become partially or totally blocked by adenoids or tonsils when the child is
asleep. I will refer him to the EN T surgeon for tonsillectomy (Adeno). It is a surgical procedure to remove tonsils under GA.
Having adenoids and tonsils taken out cures OSA in 80-90% of cases.
- Lifestyle modification
o Healthy diet
o Lose weight if indicated
o Stop smoking
- Investigation: overnight oximetry and polysomnography (gold standard). The child sleeps overnight in a sleep laboratory with
parents to detect any apneic episodes.
- Regarding his recurrent ear infection (may be due to TM dysfunction), he can have a prolonged course of antibiotics (Amoxicillin
x 3 weeks) and or grommets. I will refer you for formal hearing tests because of his hearing problems.
- Treat Allergic rhinitis, if indicated
- If surgery delayed we can do CPAP wherein the child will wear a closed fitting mask over the nose during sleep while a small air
compressor forces air under low pressure over the upper airways to keep them open.

Recurrent Cough (Postviral cough)


75

Case: Your next patient is a 2-year-old boy brought in by father in your GP clinic because of recurrent cough and is concerned.

Task
a. History (recurrent cough x 6 months, dry cough, coughs a lot at daytime, not whooping type of cough, no hospitalization, no
problems with pregnancy or delivery, immunization up to date, development normal, on childcare, smokes outside, no FHx of
asthma)
b. Physical examination (well child and playful, VS normal, ENT normal, no LAD, equal air entry no added sounds; abdomen
normal; rest of examination normal)
c. Diagnosis and Differential Diagnosis
d. Management

Differential Diagnosis for Recurrent Cough


- Asthma
- Gastroesophageal reflux
- Postnasal drip (sinusitis, allergic rhinitis)
- Tracheomalcia (onset in infancy)
- Dry (worse at night): postviral cough, chronic non-specific cough of childhood, asthma
- Dry (paroxysmal): pertussis
- Wet (productive): suppurative lung disease (chronic suppurative bronchitis, CF, immunodeficiency, primary ciliary dyskinesia,
inhaled foreign body)

- In older child (honking/tics): psychogenic (especially in children with history of school phobia; cough does not occur during sleep
and remains unchanged with exertion or infection)
- Travel: TB
- Irritant cough: smoking, pollutants, allergies

History
- Can you describe the cough for me? Is it a barky cough or does it occur in bouts then followed by vomiting, turning blue or
whoop? Since when? Is it dry or productive? If productive, what is the amount, color, offensive smell, or blood? When does this
cough happen? Does the child have frequent URTI? Does the cough come after URTI? Any headaches or stuffy nose? Does he
have sneezing or eye irritation? Is it seasonal? Any triggers likes cold or exercise? Any snoring at night or daytime sleepiness?
Does it happen in the day or night? Does he have early morning symptoms? SOB? Chest tightness? Wheezy chest? Is he
allergic to pets, food, housemites? Any choking episode then followed by cough? Do you ever remember a time when the child
was unsupervised and you noted the cough or wheezy chest? Has he been diagnosed with any medical condition such as cystic
fibrosis, airway problems or any malignancy? Any problem with his waterworks or bowel movements like diarrhea or hard to flush
stools? Does he have any vomiting after feeding? Is the cough worse when lying down? Any weight loss? Fever? Night sweats?
Any history of travel especially to Asian countries? Any lumps or bumps? Is it associated with any tics?
- BINDS!!!!!!!
- FHx of asthma, eczema, hay fever?
- Anyone smoking at home? Anyone at home or childcare having similar symptoms? Any medications being taken?

Physical examination
- General appearance: pallor, cyanosis, dehydration
- Skin: Clubbing and eczema/rash
- Vital signs and growth chart
- ENT: nasal polyps, enlarged tonsils/adenoids
- Chest and lungs: deformity or wheeze or stridor
- Abdomen: organomegaly

Management
- The child is well and there are no findings on examination so most likely, it is a postviral cough because children can cough for
many weeks after a viral infection and usually, they can 6-12 URTI per year.
- Antibiotics and cough medications are not useful. However, I would advise you to stop smoking (even outside) because passive
smoking can increase his chances of having URTI.
- If with sinusitis/allergic rhinitis/atopy: avoid allergens
- Healthy diet
- We will have a wait-and-see approach with regular follow-ups
- If concerned, do CXR and lung function if possible
- Red flags: SOB, weight loss, wheezy chest, fever, poor feeding, problems with growth

Cystic Fibrosis

Case: You have a 10-weeks pregnant lady G2P1 in your GP clinic. Her first child Mary aged 2 years, was born prior to their arrival in
Australia. She was having recurrent URTI and recently diagnosed with cystic fibrosis. She came to discuss cystic fibrosis and chances of
having another baby with this condition.

Task
a. Explain nature of cystic fibrosis
b. Explain likely outcome and counsel regarding this pregnancy
76

Counseling
- I know you have come to see me to discuss cystic fibrosis. Can you please tell me how much you know about this condition?
Cystic fibrosis is an inherited disease which affects mucus-secreting glands in the lungs, GI tract, pancreas, liver, skin, and
reproductive system in boys. It is a serious condition often manifesting with recurrent URTI, chronic diarrhea, poor appetite, and
as a result, poor weight gain.
- When a person has CF, their mucus glands secrete very thick sticky mucus. In the lungs, the mucus clogs the tiny air passages
and traps bacteria (staphylococcus/hemophilus/pseudomonas). Repeated infections and blockages can cause irreversible lung
damage (bronchiectasis, pulmonary fibrosis, pulmonary hypertension and cor pulmonale). If disease involves the pancreas, it
presents with poor appetite and chronic diarrhea, and as a result, poor weight gain and decrease in the level of fat-soluble
vitamins.
- The management of cystic fibrosis is complex and best achieved in a specialist CF center. At present, there is no cure for CF.
Treatment aims to slow progression of the condition and includes:
o Nutrition – high fat high calorie diet; pancreatic enzyme replacement if needed. Vitamin supplementation (ADEK) to
maintain adequate growth.
o Salt supplement
o Lung care – chest physiotherapy at least once daily. The physiotherapist will teach you appropriate techniques. BAL is
done yearly until the child can cough out sputum. There is often a need for prophylactic antibiotics orally or via
nebulizer. Even viral infection may predispose a CF patient to secondary bacterial infection so antibiotic therapy is
used aggressively.
o Immunization – yearly influenza vaccine and 5-yearly pneumococcal vaccine are recommended
o Regular exercise increases physical fitness in patients with CF. Upper body exercise may increase respiratory muscle
endurance.
- Often, children with CF require frequent admission. I want to stress that during hospital stay all facilities for normal psychosocial
development are available.
- Unfortunately, even with maximum care and support this condition is life-shortening. Patients have life expectancy into their 30s.
- CF is an autosomal recessive inherited condition. There is a 25% chance at each birth for a child to have cystic fibrosis. You and
your partner need to have genetic counseling. There are two available methods to find out during the pregnancy if your second
child has CF. It is CVS and amniocentesis. CVS is done between 11 and 12 weeks and amniocentesis is done around 15-16
weeks. These procedures are commonly performed and relatively safe. However, there is a small risk of miscarriage after these
tests. CVS carries 1% and amniocentesis 0.5% risk of miscarriage, respectively.
- You do not need to decide today. I want to give you reading materials and important contact numbers for CF support groups. If
you have any financial difficulties, we can involve social worker.

NEPHROLOGY

UTI in a Child

Case: You are an HMO in ED and a 1-year-old child was brought in by mother due to fever, decreased activity and appetite. On
examination temperature is 38.3C. You did urine bag specimen and showed nitrites ++++ and leukocytes +++, No RBCs.

Task
a. Explain the diagnosis (fever x 3days, not eating, irritable and lethargic)
b. Further management

- Infants:
o Fever can be sole symptoms
o Nonspecific (lethargy, irritability, vomiting, poor feeding)
- Older children:
o Fever, dysuria, frequency, incontinence, macroscopic hematuria, abdominal pain
- Confirm diagnosis of UTI by suprapubic aspirate, midstream urine or catheter sample
- 0-12 mos: Suprapubic aspiration
- 12-3 years: Catheter sample
- >3 years: midstream urine

UTI TREATMENT
>6mos Trimethoprim 5 days
Cotrimoxazole
<6mos/sick/APN Benzylpenicillin + 5-7days
Gentamycin 10-14 days (APN)
Prophylaxis Trimethoprim 3-6 mos
Cotrimoxazole
Causative organisms: E. coli (80%), Proteus mirabilis, Klebsiella oxytica, Other gram negative, Enterococcus (10%)
- 40% have underlying UT abnormality: VUR (40%), reflux associated nephropathy (renal scarring: 12%), pelvic-ureteric junction
(PUJ) or vesico-ureteric junction (VUJ) obstruction (4-6%), other
- Children <3mos with UTI should also have lumbar puncture.
- Subsequent investigations:
o USD: all children <6mos old presenting with first UTI; consider in children >6mos presenting with first UTI
o Micturating cystourethrogram: performed to detect VUR; provides good anatomic detail of the bladder and urethra;
o DMSA scan: done at least 6 months after UTI to look for renal parenchymal defects
o Siblings of children w/ VUR: 50% will also have VUR; consider USD in such cases
77

Management
- DRAW DIAGRAM of URINARY SYSTEM
- I will explain to you the possible cause of your child’s condition. Please feel free to ask me if you have any questions or if
anything is unclear to you. Your child most likely is suffering from UTI. It is usually caused by a bug coming from the child’s
backpassage when child defecates or from skin. Is the first episode? Any workup done? Structural abnormality (VUR posterior
urethral valve)? Family history?
- He needs to be admitted and will be referred to pediatric registrar for further investigation and management. The first
investigation to be done is

suprapubic aspiration to confirm the diagnosis because the sample from the urine bag is not reliable since it might be
contaminated. This is a procedure of collecting urine by the use of a small needle and anesthesia and it will be sent for culture
and sensitivity to identify bugs in the urine. First ultrasound is done to detect any residue of urine (should be 20ml). Under sterile
condition, a needle attached to a syringe is introduced into the bladder perpendicular to the abdominal wall in the midline. Any
organism detected confirms UTI.
- Is it really necessary? Yes, because it is important to know the organism causing the infection and we cannot rely on urine bag
specimen to detect UTI because of contamination.
- The other investigations that will be done are FBE, ESR/CRP, blood culture, and UEC. The registrar will also start the child on
antibiotics (Benzylpenicillin and Gentamycin) and panadol for the fever. Before the child is going to be discharged, an ultrasound
will be done to detect structural abnormalities of the urinary tract, and once the infection is settled a MCU is done. It is a test to
detect VUR or other structural abnormality. The technique consists of catheterizing the baby and filling the bladder with
radiocontrast agent. Radiograph is taken and if contrast moves into the ureter and kidney, VUR is diagnosed. Test ends when
the child passes the urine as the bladder is under pressure and chances are increased to pick up any structural abnormality like
VUR. If both investigations are negative, no further investigation is required. If any of them is abnormal, he will be followed up by
specialist.
- Another test is the DMSA scan. It is a radionuclide scan done at least 6 months after infection to detect renal scarring. VUR can
cause reflux nephropathy and can damage the kidney. Once the infection is settled, the child will be followed up by repeat
microscopy and culture to make sure the infection is cleared.
- Advise of hygiene and wipe from front-back.
- If VUR is suspected: do USD in siblings because of increased risk of VUR (50%).

Post-streptococcal Glomerulonephritis

Case: A father comes to your GP practice with his 5-year-old son as he passed dark urine for 2 days. He also brought a jar of the urine to
show you.

Task
a. History (dark-colored urine x 2 days, first time, noticed puffiness of face; skin infection 2 weeks before where GP gave antibiotics
and topical creams; no sore throat)
b. Physical examination (alert, marked periorbital edema, 150/90, urine dipstic +++ RBC, ++ protein; )
c. Diagnosis and management

Features
- Mainly in children after sorethroat or impetigo or scarlet fever
- Gap: at least 7-10 days
- Clinical features: irritable, lethargic, sick, hematuria, “coke-colored” urine, periorbital edema, maybe in legs or scrotum as well;
rapid weight gain; scanty urine output; hypertension indicates complication
- Diagnosis:
o GABHS antigen +
o BUN and creatinine
o C3 and C4 reduced

o ASO titer increased


o DNAse B increased

History
- I understand that your son has dark-colored urine. Can you tell me when it started? Have you noticed that the urine output has
decreased? Is it smoky? Did he have any sorethroat a week or two before? Did he have any skin infections 1-2 weeks ago? Do
you know what it was? Did you have it checked? Were medications given? Have you noticed any weight gain? Any changes in
the urine such as burning sensation when urinating or frequency? Any injury to the loin region? Is it before, throughout or after
urine? Any problems with the flow? Do you know whether he has taken any large amounts of beetroot or jellies or lollies?
- How is his general health? Any significant medical illness? How about his weight and appetite? Any fever or joint pains? Any
bleeding disorders in the child or in the family? FHx of kidney problems?
- BINDS

Physical Examination
- General appearance
- Vital signs
78

- ENT
- Chest and heart
- Abdomen
- Genital exam
- Peripheries
- Urine dipstick

Diagnosis and Management


- From history and examination, most likely, your son has a condition called PSGN following the skin infection. Do you know what
that is? Because he had a bacterial skin infection, the body produced substances called antibodies against those bacteria
forming immune complexes which deposit in the kidney causing diffuse inflammation and damage in the glomeruli leading to the
release of protein and blood in the urine.
- I would like to arrange a referral for admission. At the hospital, he will be seen by a pediatric neprhologist and will do further
investigations to confirm the diagnosis such as urine MCS, FBE, ESR/CRP, RFT, U&E, ASO titer, complement levels, GABHS
antigen. Once confirmed, they will start him on antibiotics to destroy the remaining bacteria. They will strictly maintain fluid chart,
weigh him daily, fluid restriction, low protein, low salt and, high carb diet.
- Because he is having high blood pressure, the specialist might consider giving him antihypertensives such as nifedipine and
diuretics for edema.
- Don’t worry, you are in safe hands
- Is it risky? It usually resolves spontaneously after several weeks. Although, in minority of cases, it can progress to chronic renal
failure. The prognosis is excellent. More than 95% recover completely.
- Regular monitoring of BP and kidney function and urine protein and blood.

Nephrotic Syndrome
Case: Your next patient in GP practice is a 5-year-old Betty brought by father because of puffy face for the last 3 days.

Task
a. History
b. Physical Examination

c. Investigation
d. Diagnosis and management

Differential Diagnosis
- Cellulitis
- Allergy
- Child abuse
- Kidney problem
- Heart problem
- Hypothyroidism
- Liver failure
- Malnutrition

Abdul’s D/D when he sees puffy face of child:


- Cardiac Failure
- Liver problem
- Kidney problem Nephrotic syndrome

History
- Since when? Is it getting worse or is it the same? Is there any swelling anywhere else on the body (Legs and hands also)? Is the
child having SOB? (pleural effusion)? Any abdominal pain or distention (Fluid in the abdominal cavity)? Did you notice any
change in the swelling of the private part? Did you notice any change in the weight? Cardiac problem? Any palpitation? Chest
pain? Cardiac problem in the past? Liver problem? Kidney problem? What about the child’s urine output (it was slightly
decreased)? By any chance, was the child ever diagnosed with UTI or DM? Is the child on any medications eg. NSAIDs? Any
fever, nausea, vomiting? Any swelling of the eyes? Any redness? Any trauma? (Cellulitis) Any allergy? Bee sting? Food allergy?
BINDS
- Family History of kidney problem, liver or cardiac problem

Physical examination:
- General Appearance: puffy on the face, hands and the legs.
- Vital signs: BP:85/45
- Rest of the examination was normal.
- Chest: look for fluid: dullness on percussion.
- Abdomen: any ascites, tenderness
- Private part: aggravation of fluid? “normal”

Investigations:
- Urine dip stick, BSL
- What do you want to know for urine dip stick? Proteinuria: 3+ (>3.5g/day),RBC: -ve, Leucocytes: -ve,
- FBE: ESR, CRP. Renal function test: Normal, LFTs: Proteins? “Albumin is low”, Cholesterol: High >5.5, C3, C4 level: Normal
“low in PSGN”, ASO titres: Normal
79

Who is likely to have NS?


- Patient having DM. Hypertensive, malignancy: Multiple myeloma.

Nephrotic Syndrome:
- Minimal change disease: 80-85% Steroid responsive
- Focal segmental glomerulosclerosis
- Membranoproliferative GN

Pathophysiology
- When the kidney is damaged basement membrane damaged lose albumin in urine Low oncotic pressure Fluid from
the ICF to EEF, Fluid in the vessels are less hypovolemia.

Complications:
- NEED to be careful about Hypovolemia.
- Change in immune response: Infection/sepsis
- Kidneys are not perfused: Acute renal failure.
- Loss of Anti-thrombin III, Protein C and Protein S: lead to thrombosis
- Spontaneous Bacterial Peritonitis

Nephrotic Syndrome PSGN


No hematuria Hematuria
BP normal High BP
ASO titre and C3 C4 NORMAL ASO titre high C3 C4 low

Management
- Your child has a condition called Nephrotic Syndrome. This might have been the result of the infection your child had a couple of
weeks ago which resulted in the deposition of immune complexes in the kidney causing protein to leak out.
- I need to refer the child for admission because she needs to be seen by a Paediatric nephrologist. In the hospital, her fluid
Balance (intake/output) will be monitored and her vital signs including BP, skin and pulse (dehydrated? cold/clammy skin?
Increase skin turgor?) will also be checked periodically. They will pass an IV line and give fluid and she will be weighed daily to
see if the treatment has been effective.
- The specialist might give some drugs including: steroids, Antibiotics: Penicillin prophylactic, Low dose aspirin
- Diet: low salt intake and normal protein diet
- Albumin: Always given when patient has anuria, acute renal failure, circulatory collapse, hypervolemia, low BP, kidney
compromised. When intracellular fluid goes back after a few hours give furosemide.
- Cyclosporine/cyclophosphamide (not necessarily need to mention.)
- 75% chance of relapse in the 1st year.
- Education: Do the urine dip stick at home. Follow up with renal function test and urine dip stick.

Enuresis

Case: A 6-year-old boy is bedwetting at night. He is dry during the day since 2 years old. The urine microscopy and culture is negative.
Task
a. Relevant history
b. Physical Examination
c. Diagnosis and outline management

Features
- Bedwetting in a child >5 years old and start treatment if persistent >6 years.
- Primary: child never dry at night for at least 6 months
- Secondary: child achieved dryness for 6 months then started bedwetting; most common cause is psychosocial (regression)
- Autosomal dominant with variable penetrance
- Management
o Desmopressin – given when patient has camps or sleepover; advised fluid restriction 1 hour before and 8 hours after
desmopressin

NOCTURNAL ENURESIS
Educate parents
Remove blame
Stop fluid restriction
Treat constipation
Urine microscopy and culture
80

History
- Is he dry during the day? Has he ever been dry at night? For at least 6 months? Is it getting worse? How frequent does he wet
his bed? Is there any change in color or frequency of urine or painful urination? Polydypsia or polyuria? Constipation? Stress in
the family or school? How do you react to his bedwetting? Does he try to jiggle? Whenever he goes to the washroom does he
lose urine? Anyone else in the family having similar condition?
- BINDS

Physical examination
- General appearance
- Vital signs and growth chart
- Neurologic examination
- Genitalia for hypospadias
- Abdomen for hernia orifices
- BSL & dipstick

Diagnosis and management


- Your child has a condition called nocturnal enuresis. This is the involuntary passage of urine during sleep in the absence of any
physical activity. It could be caused by a bladder defect when the bladder is not mature, genetic predisposition (father), or
insufficiency of a hormone called ADH which helps regulate water. Another would be psychosocial reasons.
- At this stage, I would advise you not to blame or punish him. Encourage him by doing a star chart. He should have a normal
lifestyle. I understand that it is difficult and embarrassing and you are washing a lot of linen, but let me reassure that this is a
manageable condition. I would advise for you to start the alarm clock method. You can hire it or buy it from the pharmacy. They
are very sensitive and even 1 drop of urine can start the alarm.
- Can he go for sleepovers or camping? Yes, but to avoid the embarrassment, we can prescribe a drug called desmopressin (ADH
nasal spray). When using it, it is advisable to not to drink fluids one hour before and 8 hours after the spray (2 puffs per night).
- We will try it for a few weeks but if it’s not working, I will refer you to the pediatrician.
- Reading material. Review in 6 weeks. Offer support.

UROLOGY

Vulvovaginitis

Case: Your next patient is young girl who came in because of painful urination.

Task
a. History
b. Physical examination
c. Diagnosis and management

Differential Diagnosis
- Child abuse
- UTI
- Foreign body
- Vulvovaginitis
- Threadworms

History
- Has she had this episode previously? Increase in frequency (She is well active and playful)? Is there any discharge (There’s
81

some discharge)? Since when is this happening? Complaining of pain during urination? Did she have fever? Is she lethargic? Is
the urine smelly? Any previous history of urinary tract infection? Discharge: Can you describe the discharge for me (“Yellowish
color discharge, some stains on underwear)? What’s the amount? When? Is it offensive/smelly? Does it contain blood (foreign
body/child abuse)? Any irritation/inflammation in that area? How’s situation at home? Any time you leave her unsupervised?
Does she go to childcare? How the relationship with you and your partner (child abuse)? Any signs of trauma? Any worms from
back passage? Risk factors: What is she wearing (Tight clothes/jeans)? Any use of soaps, bubble baths? Any change of
cosmetics use? Does she go to swimming (moisture)? Is she toilet trained? Is she obese?( not able to sit on toilet)

Physical Examination
- General Appearance: well active
- Vital Signs: all normal temperature is normal
- Pelvic: If there’s any inflammation/irritation, Inspection of the vulva: redness/scratch marks, no signs of trauma. Is there any
discharge (Discharge it was yellow) Are there signs of trauma (No)?
- Urine dipstick: Normal

Management
- From history and examination, your child has a condition called vulvovaginitis. Have you ever heard about it? Basically, the skin
over there is thin because of the lack of estrogen in young children. This can lead to the infection caused by organism from the
back passage causing discharge and painful urination.
- Advice: Wear cotton clothes. Not to leave moisture to allow organism to grow in that area.
- You can use Creams: Zinc oxide + caster oil to relieve the pain. You can also use vinegar in warm water/NaHCO3 in the water
for the child to sit in.
- If by any chance she develops fever, is lethargic, or bloody/smelly discharge you need to bring her to hospital.
- I’m going to follow her up in a few days to see the progress.
- If offensive discharge: take swab from the labial area.

Umbilical-Inguinal hernia with Cryptorchidism

Case: A 7-month old baby was brought to your GP clinic by his mother because she noticed a lump in his groin area and another at the
belly button. You diagnosed it as umbilical and inguinal hernia and you also noticed that the left sided testicle is undescended.

Task
a. Discuss the condition with the mother
b. Advise further management
c. Answer question

Counselling
- Do you have any particular concern before I explain anything about your son’s condition? This happens when part of the gut
goes out into the umbilical or inguinal area.
- May ask: reducible/painful/vomiting/fever?
- Any thyroid disorders (hypothyroidism)?
- Family history of hernia?
- Severe cough or constipation? Abdominal operation?
- Term/preterm?

Counselling

Umbilical
hernia

Inguinal
hernia

- For umbilical hernias, no treatment is necessary. They will usually disappear by 1 year of age. Some larger ones may recede at
4 years of age. If they don’t disappear by age 4, then refer to specialist; putting a coin increases the risk of strangulation
- Inguinal hernias: this hernia is the one that I’m concerned about. It carries risk of incarceration and strangulation. Incarceration is
the type wherein it goes out but it cannot be pulled back, while strangulation occurs in an incarcerated hernia which is an
emergency. If at any time you notice the child always crying, vomiting, or you see that the lump cannot go back, then you need to
go to the emergency department.
- 6x2: at birth – 6 weeks: within 2 days; 6weeks-6months: within 2 weeks; 6months onwards: 2 months
- Refer to specialist.

Inguinal 2/3

High scrotal
Mid-scrotal
82

- For cryptorchidism: in 90% it usually goes with indirect inguinal hernia. It is common in preterm patients (20%). If it is found at
birth then we can review for 3 months. Referral if still not descended. Right now, since he’s 7 months old, I will need to refer him
to a surgeon to repair because this is the optimal time for surgery to take place (6-12 months). But it is up to 2 years. What
happens if it doesn’t get fixed? Decreased fertility, increased cancer (5-10x) and increased risk of trauma.
- What is a hernia? It is a condition in which part of the intestines or other tissues like fat in the abdominal cavity protrude or come
out through a weak spot in the abdominal wall. Is it congenital? Any abdominal operations? Family history? Premature birth?
Constipation? Coughing?
- Umbilical hernia is easily reducible. Usually disappears gradually and most of them disappear by 1 year of age. Larger ones
disappear by 4 years of age. Any history of hypothyroidism? There is no treatment required. We only refer to the specialist if it
doesn’t disappear by 4 years of age. Do not put a tape or strap because it can cause strangulation.
- Inguinal hernia is more common in premature infants and it can cause intermittent pain and discomfort. The 2 things that we are
worried about incarceration and strangulation. Incarceration is when the hernia cannot be pushed back into its place. RED
FLAGS: If a child cries a lot, if the area is painful to touch, if it is irreducible or the child has fever, nausea or vomiting, then
immediately bring the child to ED. Strangulation is constriction of blood vessels that can arrest the flow of blood to a tissue. He
will have severe pain with change of the skin to blue or blackish. Refer to surgery asap.
- Undescended testis: 20% of premature males, 2-4% in term males; more than 2/3 is in the groin region. If nonpalpable at birth
review in 3 months then refer; surgery is orchidopexy which is to fix the testis to its place. Optimal time is 6-12 months and
satisfactory as long as testis is within scrotum at the age of 2 years so as not to affect sperm production.

Problem of non-descent
- Testicular dysplasia
- Trauma
- Risk of malignant change (5-10x)

Optimal time of surgery is 6-12 months. There is decreased risk of affecting fertility if testis is repaired by 2 years.

Advantages of early orchidopexy


- Provides optimal chance of fertility
- Corrects indirect inguinal hernias (coexists in 80%)
- Reduces risk of trauma
- Reduces risk of torsion
- Reduces psychological consequences
- Lessens risk of malignancy

Testicular Torsion

Case: A 10-years old boy is brought into your GP clinic by his father with complaints of pain in his right groin area.

Task
a. Focused history (biking then sudden pain on the scrotum x 1 hour groin area lower tummy; (+) previous episode, 9/10,
nausea and vomiting, fever)

b. Physical examination (unwell and distressed; tachycardia; Right side of the scrotum inflamed, red, and swollen with tenderness)
c. Diagnosis and management
- Anyone <18 years old with sudden onset pain is TESTICULAR TORSION until proven otherwise.

Differential Diagnosis
- Strangulated hernia
- Trauma
- Epididymo-orchitis
- Renal stones

History
83

- Can you tell me more about what happened? Severity? (Give painkiller)? Sudden onset or gradually? Is it constant or increasing
in intensity? Did it happen for the first time? Aggravating factors? Did he fall off the bike or is there history of trauma? Associated
symptoms: Any nausea/vomiting? Fever? Trauma? Lumps before? Is there any recent swelling of his glands near his ears due to
mumps? Any lump in the lower tummy or groin that is difficult to push back? (Epididymo-orchitis) UTI or STI; any stones?
Surgery in the private area? Previous hospitalization due to serious medical illness? How is his general health
- BINDS
- Prehn sign: elevation of the scrotum relieves the pain;

Physical examination
- General appearance: dehydration
- Vital signs: pulse and temperature (might be increased)
- Abdomen: tenderness, lump (strangulated/incarcerated hernia)
- Genital: check for redness, swelling, and acute tenderness of scrotum; horizontal testis drawn up in the groin and thick tender
spermatic cord; Prehn sign

Diagnosis and Management


- Draw diagram
- Most likely he has a condition called testicular torsion in which testis is twisted around leading to severe pain. it is an emergency
condition that needs immediate referral to the hospital and I am calling for an ambulance now.
- He will be seen by a pediatric surgeon ASAP who will perform a surgery to untwist it. It is called orchidopexy in which both the
testes will be fixed at their places to avoid any recurrences in the future.
- Do you need any investigations? NO investigations are unnecessary because it can cause undue delay for treatment. The
optimal time to operate this condition is within 4-6 hours after onset of the pain and any delay can cause loss of testicle due to its
compromised blood supply. In that case, it is removed rather than fixed back.
- Will it affect his fertility? No. because the other testes will still be functioning.

For strangulated hernia: mixed with intusussception

NEONATAL AND INFANT CONDITIONS

Erythema Toxicum Neonatorum

Case: David, 5-days old presented to your GP clinic with skin rash

Task
a. History
b. Physical examination
c. Differential diagnosis
d. Plan

History of rash
- Since when? Where? Does it spread? What makes it increase or decrease? Have you had any past treatment or history? contact
with a person with similar symptoms? Is it itchy?

Differential Diagnosis
- Miliaria
- Erythema toxicum neonatorum
- Neonatal sepsis
- Impetigo
- Listeria
- Varicella
- Staphylococcal infe3ction

Features
- Very common condition
- Underlying pathophysiology is unknown
- Benign, asymptomatic, Self-limiting condition with no complication
- Common in term babies between day 3 to the 2nd weeks
- 90% of cases peak at 2-3 days of age
- Resolve by first week of life but occasionally runs a fluctuating course over a few weeks
- Broad erythematous base up to 2-3cm diameter with a 1-2mm papule or pustule
- Fluid examination: eosinophils and no organisms

Irritable Baby

Case: Your next patient is a young mother who came in with her 6-weeks-old baby because she says he is crying a lot. She visited another
clinic 2 weeks ago and was consoled that her baby was normal. She has come for a second opinion.

Task
a. History
84

b. Physical examination
c. Diagnosis and management

Features:
- Baby 2-16 weeks old especially 10 weeks
- More than 3 hours a day for more than 3 days a week for 3 weeks
- Crying during late and early evening
- Child flexing legs and clenching fists because of stomach ache.
- Include: temporal association with feeds, variation with contextual or environmental factors, parental response (both affective and
practical)
- Management
o Reassurance and Explanation to parents
o Pacifying methods
o If severe: trial of cow’s milk protein elimination and mother must exclude all cow’s milk products from their diets and
take calcium supplements or use extensively hydrolyzed formula if bottlefed.
o Medications: antispasmodics should not be used and OTC preparations are not helpful.

Differential Diagnosis
- Infantile colic
- GOR
- Cow’s milk intolerance
- Not enough milk
- Intusussception
- Galactose intolerance
- Pain from otitis media, UTI, bowel obstruction, or other infections

History
- I understand that you are concerned because your baby is crying. I know that you have seen another GP and he says that it is
normal. May I ask you a few more questions? How often does he cry? Could you please describe how many hours a day he
cries? Do you think that the crying is connected to food? Is the baby breastfeeding or bottlefeeding? Does he vomit after feed?
Any change in bowel motion? Is there mucus or did he have diarrhea? Is he hungry all the time? Does the baby feed for a longer
time? Are your breasts engorged before feeding? Does he get pale while crying? Is he drawing up his legs while crying? How
about his waterworks? Is he wetting his nappies as usual? Is it smelly or was there any change in color? Does he cry while
urinating? Any fever, cough, noisy breathing or ear discharge? Any lumps and bumps in the body? Are the bowels frothy? BINDS
- Do you have enough support? How is your mood? How is your energy level? How are you coping up with is crying? Do you take
a lot of coffee?

Physical examination
- General appearance: pallor, dehydration, jaundice
- Vital signs and growth chart
- ENT and neck
- Chest and heart
- Abdomen: masses, tenderness, hernial orifices
- Genitalia
- Anus: excoriations

Diagnosis and Management


- Sarah, from the history and examination, let me reassure you that your baby is healthy and I could not find any abnormality. Most
likely, he is suffering from a condition called infantile colic. It usually occurs at the age of 6-8 weeks. It is a common condition.
The reason why it happens is because some babies are extra sensitive to environmental stimuli or internal stimuli. For example,
if baby is hungry or tired, if there are external noises or overheating of room. It is completely harmless which will most likely
disappear by the age of 3-4 months. It is also related to high caffeine intake of mom.
- It is advisable to maintain a cry diary and see if it is related to food or other factors. You can minimize environmental stimulation
by having low-level background noise, soft lighting and comfortable ambient temperature.
- You can do the 5S Techniques
o Swaddling
o Side or stomach position
o Shushing
o Swinging
o Suckling
- You can also take him for a ride in the car because it helps.
- It is okay for the child to cry for some time. It is called the 5-minute rule. Don’t worry your milk is adequate.
85

Please continue with breastfeeding. Changing to formula will not make a difference. But if you’re too tired or exhausted, we can
consider formula to give you a break
- I will organize a social worker for help.
- Family meeting

Direct Hyperbilirubinemia (Jaundice in a 3-day-old baby)

Case: You are working as an HMO in Pediatrics and David, 3-days-old was brought to you by his mom because of jaundice.

- Investigation: total bilirubin (elevated); unconjugated – normal; conjugated – high

Task
a. History (jaundice started D3, with frequent sleeping and decreased appetite; mother A+)
b. Examination findings (moderate jaundice, BW:3kg, growth chart 25%)
c. Investigation (FBE, MSU, Total bilirubin/B1/B2, USG of abdomen, UEC, BSL)
d. Differential diagnosis and diagnosis
e. Management

Differential Diagnosis for Jaundice


- Direct/Conjugated
o Galactosemia (urine for reducing sugar/substances)
o Neonatal hepatitis/infection
o Biliary atresia

- Indirect/Unconjugated
o ABO, Rh incompatibility
o Infection/sepsis
o G6PD/Pyruvate Kinase deficiency
o Hypothyroidism
o Hereditary spherocytosis
o Breastmilk/Breastfeeding jaundice
o Physiologic jaundice

History
- When did the jaundice start? Any fever? Vomiting? Skin rash? Change of color of stool or urine? Any medications? Infection
during pregnancy? Premature rupture of membrane? Maternal fever? Birth: preterm? Blood group of mom, partner, and child?
Birth trauma? Feeding? Wet nappies?

Examination
- General appearance: dysmorphic features, sleepy, crying, active, dehydration, pallor, jaundice (up to where – check level and
depth)?
- Vital signs and percentile
- Chest and heart
- Abdomen
- Genital examination

Investigation
- FBE, Blood film
- Blood group for mom and Baby
- Coomb’s test (direct and indirect)
- Total bilirubin (direct and indirect)
- MSU
- TFTs
- RFTs reflects that of mom

Diagnosis and Management


- Your son has conjugated hyerpbilirubinemia which is an increase in a certain type of pigment in the blood. This is not an
uncommon condition. There are several rare conditions which can cause these: biliary atresia, neonatal hepatitis and
galactosemia. The drainage in the liver can be blocked in the case of biliary atresia which is why we need to do ultrasound. A
metabolic disease may also be the cause so we need to check the urine for reducing sugar.
- I would like to refer him to the pediatric registrar and consultant who will take his history and who can manage this case better
than I can.
- If bilirubin is high: can do phototherapy but if high or increasing then may do exchange transfusion

Physiologic Jaundice (unconjugated) No case

Features
86

- End of 2nd day to 10-14 days in preterm


- End of 3rd day to end of 1st week of life in term
- Pregnancy history: TORCH
- Delivery: instrumental delivery cephalic hematoma
- APGAR
- Birth weight
- Infection such as in urine, ENT, chest
- Neonatal screening test: hypothyroidism (large tongue, umbilical hernia), PKU, galactosemia
- Change in urine color
- Change in bowel color
- Baby putting on weight, happy baby, feeding normally
- Investigation: normal except jaundice
- FBE normal; PE normal except jaundice
- Rule out shape of RBC: hereditary spherocytosis (unconjugated) and check enzyme defects especially G6PD
- N/V/fever, no problem with urine
- Treatment: phototherapy

ABO Incompatibility

Case 1: You are asked to see an infant Jessica born 24 hours ago with jaundice. She is the first child of a mother whose pregnancy was
normal. Delivery was at term by midwife and was uneventful. Infant’s weight was 3700g. Jaundice was noticed soon after birth, <24 hours.
Infant has been sucking well at the breast. Mom wants to go home ASAP. On examination, she has clinical jaundice, otherwise well and
active. No hepatosplenomegaly or other signs.

Task
a. Investigation
b. Advise parents on diagnosis
c. Further management

AMC cases
- Pathological jaundice
- Conjugated hyperbiliribuinemia
- Breast jaundice
- Physiological (never out before)

Features
- 1st 24 hours = pathological jaundice
o Rh incompatibility: rare
o ABO incompatibility

MOM O Baby A or B
Blood Group O A has A and B antigen
No antigen, Antibody A/B B has B and A antigen
- Moms’ antibody cross placenta and reacts against fetal RBC direct coomb’s test strongly positive
- RBC Hb + globin biliverdin bilirubin (unconjugated)
- FBE: Hb to detect anemia (<110) exchange transfusion
- Investigations: Mom and baby’s blood group, direct coombs test (detect antibody bound to surface of RBC when they cross
placenta), total bilirubin (indirect and direct), FBE, LFTs, TFTs,
- Management
o Refer to specialist
o Phototherapy: isomerizes bilirubin I bilirubin D
o Stool color change to black/green
o Monitor Hb, bilirubin and condition of baby
o Exchange transfusion: if phototherapy does not work; antibody removed and anemia treated
o Unconjugated can cause kernicterus brain damage and nerve deafness (refer for audiology + BST)

Investigation
- Total bilirubin 250mmol/L
- Conjugated 6 mmol/L
- Unconjugated 244 mmol/L
- Hb: 170 mg/dl
- Blood type: Mom O+, baby A+

Management
- I would like to explain investigation results
87

- The baby is having a condition called hemolytic disease of newborn because of ABO incompatibility. The baby’s blood group is
A+ and mom is O+. There is a reaction between the antigen on the baby’s RBC and the mother’s antibody. Because of this
reaction, antibody has crossed placenta causing breakdown of fetal RBC. This process of breakdown is called hemolysis called
hemolytic disease of newborn
- Can we go home? I understand you are undergoing a stressful situation but the baby needs to stay in the hospital for further
treatment until the condition settles. The product formed by the breakdown of baby’s cell is called bilirubin. One of the
components of bilirubin is high and that needs treatment under light called phototherapy. If we do not treat her, the level of
bilirubin can further increase and can cause damage to the brain. It is called kernicterus and can affect hearing of the baby by
affecting hearing nerve causing deafness. I am not scaring you but let me assure you that with treatment, this condition is easily
treatable. She will be placed under light and under continuous monitoring. Her eyes will be covered to protect them and she will
only be under light when she is sleeping and not feeding. Her bowel motion may be loose green/black in color when under light
so don’t be concerned about that. This treatment can be given in the room where you are staying now.
- Are there any side effects? No. this treatment has excellent results. However, if level of bilirubin rises further despite treatment,
we have the option of exchange transfusion. This procedure involves slowly removing patient’s blood and replacing it with fresh
donor blood.

- Can I continue breastfeeding? Yes you can.


- What happens after treatment? Follow up developmental assessment and referral to audiologist for audiometry to check hearing.
- Check bilirubin frequently during the first 24 hours.

Critical error
- Failure to recognize hemolytic disease of newborn
- Failure to give phototherapy

Breastmilk Jaundice

Case 2: Baby Helen was brought to see you in Gp clinic as mom is concerned about continuing jaundice. She is now 2 weeks old and was
born at term by easy vaginal delivery. Her weight was 3700g, APGAR 9,10 (1,5 minutes). she became jaundiced on day 3. There was no
blood groups incompatibility, no RBC, structural/enzyme abnormalities. She was treated with phototherapy for 2 days. Since discharged
from the hospital at day 8 of age, jaundice has persisted and mom is concerned. Baby is feeding well from the breast. Current weight is
3900g. On examination, the baby is active, and clinically normal apart from jaundice.

Task
a. History
b. Investigation
c. Explain diagnosis
d. Management

Features
- >15 days
- Similar to physiologic jaundice
- Breastfreeding +
- Treatment: reassurance, if concerned stop 24-48 hours to see improvement/confirmation
- Red flags: lethargy, fever, drowsy, infection
- Frequent followup

History
- Is she your first baby? How is her general health besides jaundice? Did she have any fever after birth/infection? Any change in
color of urine? Is she crying a lot while changing nappy? Is the urine smelly? Is there any change in bowel motion? Any nausea,
vomiting or fever? Any ENT or chest infection? Any problems establishing breastfeeding? Is she gaining weight? Is she active
and playful? Did she have neonatal screening tests?
- Mother: How is your general health? Are you on any medications? Do you smoke or drink alcohol?

Investigations:
- Serum bilirubin level unconjugated bilirubin (high), FBE normal, Hb normal, WCC normal, TFTs normal, urine MCS normal

Management
- Most likely your child has a condition called breastmilk jaundice. This occurs in approximately 10% of breastfed infants. Let me
assure you that it is not a serious condition. It is a benign self-limiting condition and requires no treatment.
- Is there anything wrong with my milk? NO. There is nothing wrong with your milk. Although it is caused by certain factors in the
breastmilk, it doesn’t require treatment and you can continue breastfeeding.
- Does she need phototherapy again? No.
88

- For how long will it persist? It may persist up to 3 months of age, but baby remains well and gains weight. However, it may
disappear in just a few days.
- The diagnosis can be confirmed by suspending breastfeeding for 24-48 hours which will result in the fall of bilirubin level. After
which, breastfeeding can be continued. This is acceptable but not necessary.
- Please express your milk in order to maintain lactation if temporary suspension is advised.
- Red flags: fever, jaundice worsening, drowsy or decreased LOC. Review.

Pathological Obstructive Jaundice (Conjugated Hyperbilirubinemia Biliary Atresia)

Case 3: A mother brings her 2 weeks old baby boy who is having jaundice since he was 3 days old in your GP clinic. His BW was 3.5kg.
The mother’s blood group is O+ and baby is also O+. The serum bilirubin level is given which shows significant conjugated
hyperbiliribunemia.

Case: Tom aged 35 days old was brought by her mother Nicole to your GP surgery. She is concerned about Tom’s stool color which she
noted clay this morning. Tom was born by spontaneous NSVD at 38 weeks in a local hospital and was discharged home the next day.
Nicole had been breastfeeding tom but she finds tom is not actively sucking since this AM. Tom is the first baby to Nicole and she is very
excited about the prospects of parenthood.

Task
a. Further history
b. Further management (no jaundice, organomegaly, baby at 30 centile, hernia orifice, chest and lungs normal; investigations FBE
Hg 130, WBC 13000, Platelets 465, Bilirubin 178 mmol/L, unconjugated 68mmol/L, conjugated 110mmol/L should be less
than 20%)

Pathological (Obstructive) Jaundice


- Bilirubin: >25mmol/L or 25% of total bilirubin
- Occurs early in life
- Level of jaundice variable
- 3-4 weeks (4-6 weeks) onset/worsens
- On examination
o Baby normal
o Growth normal
o Putting on weight
- Biliary atresia: bile duct obliterated bile duct cannot enter 2nd part of duodenum to be excreted accumulates in the liver
damage and cirrhosis
o has to be treated within 70 days of life
o surgery: liver transplantation
- choledochal cyst: cyst in the bile duct obstruction
- neonatal hepatitis: LFTs abnormal, history of contact (viral or autoimmune)
- galactosemia: deficiency of enzymes that cause breakdown of galacotose accumulation damage of eyes, kidneys, liver
galactose spills out in urine
o investigation: reducing substances in urine; urea, creatinine and electrolytes
- Sepsis and hypothyroidism FBE, TFT
- History: pregnancy, delivery, after
- Ask about change in color of urine and stools

History
- Can you tell me more about it? Is the jaundice getting better or worse? Did he have any treatment for it? Is John your first baby?
Was he delivered full-term or

preterm? What was the mode of delivery? Any complications (cephalhematoma)? Any instrumental delivery? For how long did
he stay in the hospital?
- Did you have any infection in pregnancy? Did he have his neonatal screening test? Did you notice any fever after birth? Any
lethargy, drowsiness or excessive crying? Any episode of vomiting? Is he breastfeeding or bottlefeeding? Any problem with
feeding? Any problem with establishing breastfeeding? Is he putting on weight (dropped weight in week 1 then putting on after
that)? Did you notice any change in color of his urine (Yes, it is dark)? Is it smelly? Does he cry a lot when changing nappies?
Any change in bowel motions (Yes. It is pale white/pale grey)? Did you notice any mass in his tummy (splenomegaly)? Tongue
enlargement? Any recent contact with anybody who has hepatitis? Is he taking any medications?
- FHx of liver disease or metabolic disorder like galactosemia?
- Immunization up to date?

Management
- From history, I’m quite concerned about your son’s prolonged jaundice because the rise in conjugated bilirubin means
obstructive jaundice. I would like to arrange his admission to the hospital ASAP where he will be seen by a pediatrician. He
needs further investigations to rule out some serious conditions most importantly, biliary atresia as a cause of increased bilirubin
level.
- The further investigations include FBE (Hb and WBC), LFTs, USD of the abdomen and further liver biopsy can be done to
confirm the diagnosis of biliary atresia or choledochal cyst, urine for reducing substances, UC&E, TFTs.
- Biliary atresia is the obliteration of the extrahepatic biliary system resulting in obstruction to bile flow. It cannot be transported into
the intestines so it is retained in the liver causing jaundice and further damage. The cause is unknown and treatment is surgery –
liver transplantation – within 70 days of age.
89

SIDS (Book case: Condition 5)

Case: You work in GP and are counseling the family of a 4-month-old male infant who was rused to ED of local hospital the day before but
was dead on arrival. The provisional diagnosis is SIDS and the baby is to have a biopsy. You had seen him for the first time 2 months
previously with his single mother when he was thriving and developing normally and had commenced imunisations. Two days before his
death you saw him again, this time with mild URTI which were causing minor difficulties with breastfeeding. However, over the next 2 days
he apparently improved, and his mother had advised you that he appeared normal and fed well from breast just prior to his death. You are
unaware of any suspicious circumstances surrounding the death. The family members have attended to seek details of why the baby died
and why an autopsy is necessary. The spokesperson for the group is the mother’s sister, the aunt of the infant. The mother is also present,
but is too distressed to ask any questions herself.

Task
a. Answer questions of aunt relating to the death of the infant
b. Counsel accordingly

- I am SO SORRY to hear about Andrew’s death. Please take my deepest condolences.


- We can’t understand why Andrew has died. The most likely cause is SIDS also known as a “cot death”. Sadly, it is a major cause
of death under the age of 1 year. The peak incidence occurs at around 4 months of age. The certain cause is unknown, but
many theories exist. Usually, there is a need to exclude other possible causes such as severe infection, which is unlikely in
Andrew’s case because his snuffles were not warning signs to think about overwhelming infection. Other conditions such as
metabolic and genetic causes also need to be excluded. That is why, in case of any sudden or unexplained death, police and a
coroner must be notified.
- Why did the police have to be involved? Do they think my sister killed her baby? I can see that you are going very difficult and
dramatic time. The main purpose of this management is to try and find out what causes SIDS and exclude other possible causes.
The police officers will interview you and me and doctors in the hospital, to assist the coroner. All police officers who work with
SIDS cases are specially trained to conduct their work in a manner to provide comfort to the family. Nobody suspects your sister.
- There is a need for autopsy. It is a MEDICAL examination of a body and the internal organs after a person has died. It is usually
performed on the next working day after the child’s death. It is done like a surgical operation by a very experienced pathologist IN
AN ATTEMPT TO FIND OUT WHAT CAUSES SIDS and EXCLUDE POSSIBLE CAUSES OF DEATH. During this procedure,
tiny tissue samples will be removed for further examination under microscope. They might also be sent for chemical analysis and
microbiological culture. Usually, the full result is available 6 weeks after autopsy procedure. However, I can try to find out about
initial findings. Once the autopsy is complete, you can arrange for a funeral. A social worker can help you with a funeral
arrangement. If you wish to see or hold Andrew or take some locks of hair, I will try to arrange this.
- For now, I want to give you important contacts of SIDS support group which can help you get through the stages of grief. I am
here to help you too. Please don’t blame yourself. You are not responsible for Andrew’s death. It is not related to his mild
infection and medical treatment will not change the outcome.
- The SIDS support groups often hold meetings with other bereaved parents. Perhaps, eventually, you can think about it.

SIDS Counseling

Case: You have a 32-weeks pregnant lady at your GP clinic. She wants to know about SIDS because her neighbor just lost the baby.

Task
a. Counsel your patient

- I understand you have come to see me because you are concerned about SIDS. I’m so sorry to hear about your neighbor’s loss.
How much do you know about this condition?

- I would like to ask you a few questions regarding your pregnancy. Do you have regular checkups? Any concerns about blood
tests, USD, sweet test, and BP? Do you have any complaints at the moment? Do you feel the baby’s movements? How’s your
general health?
- Do you or your partner smoke? Do you or your partner drink alcohol? Have you or your partner ever tried recreational drugs?
Who do you live with? Do you have enough support? Do you organize nursery or you plan to place cot in your bedroom? Do you
plan to breastfeed? What is your POV regarding immunization? FHx of SIDS
- Let me tell you about SIDS, risk factors, and preventive measures.
- SIDS also known as “cot death” is the major cause of death under the age of 1year. The incidence is around 1:1000 but
improving since the introduction of the government campaign.
- The causes are unknown but risk factors have been identified.
o Sleeping position: Sleeping position is important. Always place baby to sleep on its back with your baby’s feet at the
bottom of the cot.
90

o The cot environment. Infant’s dying from SIDS were found to be wrapped more warmly, wore hats, used quilts or
dooners, had cover over their heads, or were wrapped loosely. That is why always use firm, well-fitting mattress. Don’t
use cot bumpers. Keep soft toys out of the cot. Tuck in the bed clothes securely and ensure the head is uncovered.
Make sure your baby is not overheated. Sweating around the head and neck indicates the baby is too hot. Avoid
wearing hats at sleeptime.
o Tobacco smoke: Daily exposure of infant to smoke is highly significant and dose related. Make your house a smoke-
free environment at all times.
o Bed sharing: Bed sharing is a risk factor if parents have been smoking, using alcohol, or recreational drugs. Baby
should sleep in the cot. However, you might consider room sharing because there is evidence which suggests sharing
a room with the baby for the first 6 months may be beneficial.
o I will teach you how to recognize significant illness in your child. Spike a fever, not feeding well or decreased nappies,
etc..
o Statistics show that immunized children are at lower risk of SIDS than those who are immunized.
o Breastfeeding: Breastfeeding is a protective factor.
o There are some risk factors which we cannot influence such as prematurity, low birth weight, low apgar scores,
Boys>girls.
- Investigation or blood tests? There is no investigation that can predict SIDS.
- I head about Home Apneic Monitoring (HAM), should I get one? HAM is not routinely recommended as no study has
demonstrated that this program reduces the incidence of SIDS.
- Does previous SIDS increase risk of having another child with SIDS? Slightly. There is 1.6x higher risk. In this case, home apnea
monitoring can be considered mainly for parental support.

OTHERS

Immunization Advice

Case: You are a GP and a 6-weeks-old baby boy was brought in by mom. He is the first child of the family. The child has been breastfed
and is gaining weight. All examination up to now is normal. The mother wants to know how and when the child is going to be immunized.

Task
a. Outline the current immunization protocol
b. Explain what diseases are covered by it

- As you know, immunization is a very important aspect of preventive medicine. It works in two ways: first of all, it stimulates the
immune system of the child to produce cells that defend the body. Secondly, we inject the child with the same bugs as that of the
disease, but these bugs have been weakened by certain techniques so they can’t produce the disease.
- Immunization is offered at certain times starting at birth and then at 2, 4, 6, 12 and 18 months. Later doses are usually at
preschool age. Usually, more than one dose is required for complete protection. With the development of immunization program
in majority of the countries of the world, a number of serious and lethal disease have been eradicated. That is why, immunization
is recommended for all children all over Australia. Within the government’s program, the diseases that are covered are
chickenpox, rotavirus that produces diarrhea, polio, infections like measles, mumps, and rubella, hepatitis B, pneumococcal
vaccine that prevents respiratory and brain infections, and DPT vaccine that prevents against whooping cough, tetanus, and
diphtheria or gray membrane infection of the throat.
- As you know, all medications have side effects. Majority of vaccines have a few insignificant side effects like local skin reaction
(pain, redness, and swelling of the skin), sometimes especially with DTPa the child can develop high-grade fever, but we usually
give antipyretics half an hour before the vaccine to prevent that. This side effect is sometimes accompanied by excessive,
inconsolable high pitched crying (because of pertussis component). With the arrival of acellular pertussis vaccine, these side
effects have been minimized.
- There are some contraindications for these vaccines especially the live vaccines such as MMR, chickenpox, OPV:
o Absolute Contraindications:
History of anaphylaxis
Encephalopathy within 1 week following injection of DTPa
immunodeficiency states (child with HIV, on chemotherapy, on treatment with high-dose steroids >2mg/kg
for more than 2 weeks)
o Relative Contraindications where we will delay the vaccination:
Fever >38.5
If child has been on chemotherapy previously – delay for 6 months after stopping
- My friend’s child had an egg allergy and it was not given to her child. Is this true? Previously, kids who had history of egg allergy
were not given MMR but

now it is recommended to give the vaccine in a controlled manner where all equipment for resuscitation is available.
- I have heard a lot about homeopathic vaccination. Up till now, there has been no evidence within the medical literature that
supports efficacy of homeopathic vaccination. However, the decision is still yours.
- What if I travel in between and my son misses a dose? There is a special catch-up schedule for children who have missed their
doses or who come to Australia from overseas.
- I am going to give you written material that will tell you exactly when to bring the child for each vaccination.
91

- It is important to maintain a record for your child (blue/yellow book).


- MMR and autism? There is no literature up to now that supports it.

Down Syndrome

Case: You are an HMO in a hospital and a mother of two just delivered a baby. The pediatrician suspects that the baby has down
syndrome. The first child is normal. The pregnancy and delivery of this child is normal.

Task
a. Ask examiner for typical physical features of down syndrome
b. Counsel

Counsel
- I know from the notes that your child has been suspected of having a child with Down syndrome? Do you have a special concern
at this moment? I would like to ask the examiner about certain features specific to down syndrome.

Physical features of Down Syndrome:


- Floppiness at birth (reduced muscle tone) - most important sign 90%
- Microcephaly
- Additional features that may or may not be present: flat occiput, moon-shaped face, short neck, flat nose, wide nasal bridge,
epicanthal fold medially and upward slanted eyes, small low-set ears, small mandible, prominent tongue, brush field spots on iris
(depigmented, whitish spots), cleft lip or palate, single palmar crease
- associated diseases: duodenal atresias, hirschsprung disease
- cardiac problems are difficult to diagnose at birth

Counseling
- From the examination findings, I highly suspect that your baby does have Down syndrome. I understand that this is a big shock
for you. Are you alright to continue? Do you know anything about Down syndrome? It is the leading cause of cognitive
impairment all over the world. It is associated with a degree of learning impairment as well as developmental delay. Basically, it
is a genetic abnormality that can happen in about 1 in 800 pregnancies. It has been associated with advancing maternal age and
certain genetic defects within mom or dad. We still need to confirm the diagnosis through gene study. We will take some blood
from baby and send it to the genetic clinic. I would suggest that you bring your partner when the results are being discussed.

- At the moment, we need to do some tests to find out if your son has some of the associated defects i.e. Heart disease (VSD),
hypothyroidism, cataracts, hearing problems, spinal or backbone defects (atlanto-axial instability), and abdominal ultrasound
(duodenal atresia) We will do TFTs, echo, USG of abdomen, and spinal xray. Later on, we will continue to monitor him for the
development of any intellectual disability. Usually, these kids have limited intellectual ability but they are usually very compliant,
cheerful and happy kids. With the latest advancements, the average life span for a down syndrome baby is >55 years.

- You will have a lot of support from centerlink, social worker, respite care, child psychologist, down syndrome association of
australia.
- causes: non-disjunction or translocation

Failure to Thrive (Non-organic Cause)

Case: You are a GP and an 18-month-old boy is brought to your clinic because his mother says he is not eating well. His growth charts
show that his weight dropped from 50th to 3rd percentile. His mom is worried.

Task
a. History (preterm at 35 weeks; not feeding x 6mos; 19 years old single mom with no job)
b. Physical examination (pale)
c. Investigation
d. Diagnosis and management

Failure to thrive – drop of more than 2 percentile of weight on a growth chart; majority between 25-75 percentile

Causes of Failure to Thrive


- Reduced calorie intake
o Cleft palate
o Persistent vomiting
o Anorexia of chronic diseases
o Improper breastfeeding technique
o Inadequate provision of food
- Reduced absorption of Food
o Cystic Fibrosis
o Coeliac disease
o Chronic diarrhea
o Chronic liver disease
- Increased number of calories used
o Congenital heart disease
o Diabetes
92

o Hyperthyroidism
o Recurrent UTIs
o Cystic fibrosis
o Inborn errors of metabolism
- Psychosocial issues
o Neglect
o Poverty
o Parental depression
o Behavioral disorder in the child (ADHD, autism)
o Coercive/Forceful feeding

History
- Can you describe your child’s typical daily diet to me? What type of milk does he take (breast milk, cow’s milk, formula milk)?
How much milk does he take? Have you introduced solids? Does he eat meat? Does he eat with the family at the table? Do you
think his appetite is okay? Does he have any other problems, like diarrhea, constipation, vomiting? Any

change in the number of wet nappies? Did he suffer from frequent infections since birth? How was your pregnancy with this
child? Was it a planned pregnancy? Any problems at delivery? Did he have the regular screening tests that are done at birth
(heel prick test: cystic fibrosis, galactosemia, hypothyroidism, phenylketonuria)? How has his health been since birth? Any
problems regarding growth and development? Vaccination?
- May I ask what the home situation is? Are you supported by your partner, family and friends? What do you do for a living? Any
financial problems? Any help from centerlink? Do you own a house or are you renting? Do you smoke or drink? Have you ever
tried recreational drugs (relevant because it might interfere with your child’s growth)?
- Any family history of anybody on a special diet? Cystic fibrosis?

Vital signs
- General appearance
- Vital signs and growth chart for height, weight and head circumference; immunization status
- Chest and Lungs/Cardiac
- Abdomen: distention, organomegaly, bowel sounds
- Muscle wasting especially over the buttocks, lack of fat in the cheeks and temporal area

Investigations:
- FBE, Urine MSC, LFTs, UEC, TFTs, Iron studies, fecal microscopy and culture, stool for fat and fatty acid crystals, celiac
microscopy, chloride sweat test

Management
- Your child has failure to thrive because of malnutrition. It means that there is no serious medical condition that can be diagnosed.
The actual problem is the quantity and quality of food that is provided to your child. We need to improve his feeding habits. I will
give you some written material about proper dietary habits. It is important to give him a balanced diet containing fruits,
vegetables, meat and milk to prevent any nutritional deficiencies.
- I have also found out that you need some help in the form of social and financial support. I will contact a social worker who will
help you find support from centerlink and also to look for a job.
- I will give you some contact address of support groups for single moms where you can talk about your problems.
- For the anemia, you can try some iron-fortified cereals to improve his hemoglobin level. Please remember a healthy balanced
diet is the most important factor for your child’s growth and development.
- Referral to dietitian and review.

Refugee Assessment

Case: You are a GP and your next patient is Majuk who is 18-months old attending your practice with his Dad John as they have a letter
from the community nurse who requested check for Majuk. They are refugees from Sudan and their family had a very stressful time as
they spent the last 5 years in a refugee camp.

Task
a. History
b. Examination findings
c. Address the problem
d. Investigation and management

- Sensitivity for the hardship and trauma they endure


- Need a lot of reassurance from GP to help them and not from the authority.
- I am not from the government or immigration. I am here to help you and your son and if you are happy for me to do that, I will let
your family doctor know everything we have done and any treatment we need to start but I will not report this to anyone else
without discussing with you first.
- Communication problem language problem interpreter service (1300655820) or have appointment with interpreter;
- History
93

o Any problems? How is he? Any concern about his hearing? Vision? Sleep? Behavior?
o BINDS: How was his delivery? Any problems with your wife’s pregnancy? Was he delivered term or preterm? Any
immunization? Is he breastfed? Until now? When was solid introduced? Any plans to stop breastfeeding? Any problem
with his development? Social history?
o Serious illnesses in the family?
o How is everything with you dad? How is your mood? Do you feel low? How’s your sleep and appetite?
- Assessment:
o Growth assessment
o Nutritional assessment (vitamin D)
o Developmental assessment
o Vaccination status (do you have any written record) BCG scar in deltoid region or lateral aspect of elbow joint in
African patient!!!
o Dental assessment
o Mental health assessment of the family
o Financial and housing assessment
o English classes
o Tuberculosis: CXR and PPD Mantoux test; Quantiferon gold test (tuberculosis interferon assay) helps differentiate
between positive mantoux due to BCG vaccine and latent TB
- Investigations
o Routine: FBE,TFTs, LFTs, RFTs, BSL
o Infections: Hepatitis B serology, CXR, schistosomiasis (urine/serology),
o Nutritional test: iron studies, vitamin D levels, calcium,
- Management
o Refer to dietitian
o Monitor growth and development
o Time table to re-immunize. Contact camp to track
o Dental issues
o Mental health assessment refer to psychologist
o Paperwork for centerlink
o Advise for English class

You might also like